Board Review Express® February 6-9, 2020 Virtual Course geor Day 2 Friday, February 7

8:00 – 8:30 am Emergency I – Robert Dachs, MD, FAAFP

8:30 – 9:00 am Common ENT Problems – Teresa Holt, MD

9:00 – 9:30 am Emergency Medicine II – Robert Dachs, MD, FAAFP

9:30 – 10:00 am Challenging Issues in Hematology – Teresa Holt, MD

10:00 – 10:15 am Q&A

10:15 – 10:30 am Break

10:30 – 11:00 am Common Neurological Disorders – Teresa Holt, MD

11:00 – 11:30 am Peripheral Vascular Disease – Robert Dachs, MD, FAAFP

11:30 am – 12:00 pm The Major Arthritides – Benjamin Gilmer, MD, MS

12:00 – 12:15 pm Q&A

12:15 – 1:15 pm Break

1:15 – 1:45 pm Acute CVA & TIA – Robert Dachs, MD, FAAFP

1:45 – 2:15 pm Urologic Problems – Benjamin Gilmer, MD, MS

2:15 – 2:45 pm Maternity Care I – David Weismiller, MD, ScM, FAAFP

2:45 – 3:15 pm Endocrine Diseases – Benjamin Gilmer, MD, MS

3:15 – 3:30 pm Q&A

3:30 – 3:45 pm Break

3:45 – 4:15 pm Maternity Care II – David Weismiller, MD, ScM, FAAFP

4:15 – 4:45 pm The Surgical – Robert Dachs, MD, FAAFP

4:45 – 5:30 pm Selected Issues in Women’s Health – David Weismiller, MD, ScM, FAAFP

5:30 – 5:45 pm Q&A

Emergency Medicine: Part 1

Robert Dachs, MD, FAAFP Clinical Associate Professor Ellis Hospital Family Medicine Residency Program Albany Medical College, Albany, New York Disclosure Statement

It is the policy of the AAFP that all individuals in a position to control content disclose any relationships with commercial interests upon nomination/invitation of participation. Disclosure documents are reviewed for potential conflicts of interest. If conflicts are identified, they are resolved prior to confirmation of participation. Only participants who have no conflict of interest or who agree to an identified resolution process prior to their participation were involved in this CME activity.

All individuals in a position to control content for this session have indicated they have no relevant financial relationships to disclose. Learning Objectives

1. Be familiar with current BLS and ACLS algorithms. 2. Understand new principles of sepsis recognition and management. 3. Recognize common injuries associated with child abuse. 4. Evaluate and triage complicated extremity trauma. 5. Develop wound care strategies for common lacerations. 6. Be familiar with Good Samaritan Laws. Part 1

•Resuscitation • Specific injuries and emergencies - BLS - Head injury - ACLS - Eye and ENT emergencies • Cardiac arrest - Chest and Abdominal trauma • Arrhythmias - Muscle and extremity injury - Good Samaritan Laws - Acute laceration and burn care •Shock •Trauma – Prevention Strategies • Child Abuse Question #1: You are on a plane ride when a passenger suddenly becomes unresponsive. Which statement below is true?

A. You should immediately start chest compressions B. You should check a pulse, if none present, open the airway, give 2 rescue breaths and start chest compressions C. You should check a pulse, if none present, immediately start chest compressions D. You are aware that Good Samaritan laws protect you in all 50 states, but do not get involved because such laws do not exist during airflight A. Nontraumatic Cardiac Arrest: BLS and ACLS: Basics

Step 1: Initiate emergency response Notes 1) No pulse check – public Step 2: Start BLS *Pulse check <10sec – healthcare provider 2) Think “CAB”

“Shockable” (VF/VT) Step 3: Monitor vs. “nonshockable” (asystole/PEA) Nontraumatic Cardiac Arrest: “Nonshockable” Rhythm: Asystole & PEA

Asystole Note Step 1: Continue CPR for 2 minutes Atropine is out

PEA: The 5T’s and 6H’s

Step 2: Epinephrine 1mg IV/IO 1. Toxins 1. H+ (acidosis) q3-5 minutes 2. Tamponade 2. Hypothermia 3. Tension pneumo 3. Hypokalemia 4. Thrombosis (cardiac) 4. Hyperkalemia 5. Thrombosis (PE) 5. Hypovolemia 6. Hypoxia Nontraumatic Cardiac Arrest: “Shockable” Rhythm: V fib/V tach Defibrillate (200J) x 1 I 5 cycles of CPR, if shockable rhythm 1. Defibrillate again (x1) and 2. Give vasopressors (epinephrine 1 mg q3-5min) I 5 cycles of CPR, if shockable rhythm 1. Defibrillate again (x1) and 2. Give antiarrhythmic (amiodarone 300mg IV/IO) Electrical Cardioversion

Narrow regular: 50-100 J SVT and atrial flutter

Narrow irregular: 120-200 J atrial fibrillation

Wide regular: 100 J; monomorphic VT

Wide irregular: defibrillation dose polymorphic VT (not synchronized) All cases courtesy of ecgweekly.com

9 Good Samaritan Laws • Present in all 50 states - And on airlines (Aviation Medical Assistance Act, 1998) • General principles - No legal obligation to provide aid* - Immunity from malpractice suit if aid were provided, except for gross, willful or wanton negligence or lack of “good faith” - Restriction to application outside of hospitals* - Withdrawal of legal immunity if the doctor accepted payment for aid - Recipient of aid must not object to aid rendered. Shock

Cardiogenic Septic Hypovolemic Neurogenic Sepsis: CMS Definitions

•Sepsis=➔ SIRS: >2 or more - SIRS + source of infection ▪ T <36° or >38° •Severe Sepsis=➔ ▪ RR>20 ▪ P>90 -Sepsis + a) lactate > 2 or ▪ WBC <4,000 or >12,000 b) organ dysfunction or >10% immature cells •Septic Shock=➔ Severe sepsis + a) lactate >4 or b) Hypotension despite fluid resuscitation Septic Shock Management

To be completed within three hours of time of presentation: • Measure lactate level • Obtain blood cultures prior to administration of antibiotics • Administer broad spectrum or approved combination of antibiotics within 1 hour of sepsis recognition • Administer 30 mL/kg crystalloid for hypotension or lactate ≥4 mmol/L ------* If hypotensive (MAP <65) after fluid challenge, start norepinephrine Trauma Prevention

• 27.6 million people were treated in an ED for injuries in 2015 • 2.8 million people were hospitalized due to injuries in 2015 • Each year 214,000 people die from injury – 1 person every 3 min - #1: MVA – 33,804/yr (2013) Trauma Prevention

• 27.6 million people were treated in an ED for injuries in 2015 • 2.8 million people were hospitalized due to injuries in 2015 • Each year 214,000 people die from injury – 1 person every 3 min - #1: MVA – 33,804/yr (2013) 37,461 (2016) Trauma Prevention

• 27.6 million people were treated in an ED for injuries in 2015 • 2.8 million people were hospitalized due to injuries in 2015 • Each year 214,000 people die from injury – 1 person every 3 min - #2 #1: MVA – 33,804/yr (2013) 37,461 (2016) Trauma Prevention

• 27.6 million people were treated in an ED for injuries in 2015 • 2.8 million people were hospitalized due to injuries in 2015 • Each year 214,000 people die from injury – 1 person every 3 min - #2 #1: MVA – 33,804/yr (2013) 37,461 (2016) - #1 : Firearms 39,773 (2017) Trauma Prevention Children (age 1-19)

• 2016 statistics # of deaths % of deaths - All causes 20,360 - Injury-related 12,336 60.6% #1: Motor vehicle crash 4,074 20.0% #2: Firearm 3,143 15.4% (suffocation, drowning, burns make up the rest of trauma deaths) #3 Malignant neoplasm 1,853 9.1%

Cunningham RM, et al. NEJM, December 20, 2018 Trauma Prevention Children (age 1-19)

• 2016 statistics # of deaths % of deaths - All causes 20,360 - Injury-related 12,336 60.6% #1: Motor vehicle crash 4,074 20.0%

➢ Seat belts reduce serious crash-related injuries and deaths by 50% ➢ Seat belts saved almost 14,000 lives in 2015 ➢Air bags provide added protection but are not a substitute for seat belts. ➢ Air bags plus seat belts provide the greatest protection for adults. Child Safety Seat Recommendation

Note: NHTSA and AAP (August 2018): revised recommendation from age-based to Weight-height recommendations of manufacturer

Courtesy of CDC, accessed @cdc.gov (motor vehicle safety page) Trauma Prevention: Drowning

• 4,000 deaths/yr in U.S. - >6 minutes submersion is associated with poor outcome - Water temperature has no correlation with outcome - No difference in fresh vs. saltwater aspiration • #1 cause of death children ages 1-4 yrs of age

Fencing: Odds ratio for drowning in a fenced vs. unfenced pool = 0.27 (95%CI, 0.16-0.47) 4 sided fencing completely surrounding pool (not attached to house on one side

Thompson, DC et al. Cochrane Review 1998 accessed June 22, 2018 Trauma Prevention: Suffocation

• Most common cause of unintentional deaths in infants • Counsel parents to - Remove soft bedding and toys from sleeping area - Use the cribs original mattress size - Position newborns on their backs for sleeping

https://www1.nichd.nih.gov/sts/about/risk/Pages/reduce.aspx Head Injury-Prevention: Helmets

• Motorcycle helmets: decrease head injury (69%) and death (42%) Cochrane Library 2008, Issue 3 • Bicycle helmets decrease head injury (63%), brain injury (88%) and facial injuries (65%) Cochrane Library 2008, Issue 3 • Skiing helmets decrease head injury (60%) Sulheim, S. et al. JAMA 295:919, Feb 22, 2006 Head Trauma A. Intracerebral hemorrhage Often not seen on initial CT (delayed presentation) B. Epidural hematoma 80% due to rupture middle meningeal artery Rare in the elderly, associated with skull fx Lucid interval (“talk and deteriorate”) C. Subdural hematoma Tear of bridging veins between dura and arachnoid Common in elderly, alcoholics D. Concussion – see sports medicine lecture Epidural Hematoma Subdural Hematoma

Convex Concave, crescent shape Question #2: 65 y/o with a fib on warfarin with acute intracerebral hemorrhage after a fall. The ACCP and AHA/ASA recommends:

A. Start FFP and Vitamin K 10mg IV B. Start FFP and Vitamin K 10mg IM C. Start Activated Factor VII + Vitamin K 10 mg IV D. Start a 4-factor Prothrombin Complex Concentrate (KCentra) + Vitamin K 10mg IV Question #2: 65 y/o with a fib on warfarin with acute intracerebral hemorrhage after a fall. The ACCP and AHA/ASA recommends:

A. Start FFP and Vitamin K 10mg IV B. Start FFP and Vitamin K 10mg IM C. Start Activated Factor VII + Vitamin K 10 mg IV D. Start a 4-factor Prothrombin Complex Concentrate (KCentra) + Vitamin K 10mg IV Prothrombin Complex FFP and Vitamin K Concentrate (PCC) FFP - normalize INR 13-48 hours • Pooled human plasma −Need ABO testing • Stored as a powder −30-60 minutes to thaw −ABO testing not required −15ml/kg - approx 1L (4units) for 70kg −Volume <100ml • May need 30ml/kg • Contains II, IX, X, varying amounts Vit K - normalize INR 12-24 hours VII and Protein C and S −10mg IV (over 30min) • Rapidly reverse INR: 3-15 minutes − 3/100,000 anaphylaxis Suggested Protocol

• A. In life-threatening bleeding, on warfarin... Vitamin K 10mg IV + 4 factor PCC (Kcentra) INR 2-4, give 25U/kg INR 4-6, give 35U/kg INR> 6, give 50U/kg • B. If hx of HIT: Profilnine 50IU/kg • C. If blood product contraindicated.. Consider rFVIIA 1-2mg (off-label) Eye Emergencies: Trauma-related • Corneal abrasion Level of evidence −Topical NSAID’s offer effective pain relief from B and may result in earlier return to work. −Topical cycloplegics and mydriatics do not relieve pain B and are not recommended. −Eye patch does not improve pain and can delay healing A −Topical antibiotics may be prescribed to prevent C bacterial superinfection in corneal abrasions. If associated with contact lens – a) Do NOT reinsert lens b) use antipseudomonal antibiotic

30 Eye Emergencies: Burns

• Chemical Burns − Alkali worse than acid − Irrigate, irrigate, irrigate (2L)

• Radiation keratitis −Flourescein demonstrates multiple punctate lesions −Pain relief!!! −Encourage prevention!

31 Eye Emergencies: “I got punched in the eye”

• Orbital fracture −Check EOM • Most common= restricted upward gaze − Anesthesia distribution infraorbital nerve • Traumatic iritis − Inflammation of iris and ciliary muscle − Provide Analgesia • Mydriatics alleviate the pain associated with ciliary spasm (light sensitivity) • Hyphema: blood in anterior chamber − Elevate head of bed 30 degrees at rest − Avoid ASA/NSAID’s, ophtho consult

32 Eye Emergencies: Subconjunctival hemorrhage It’s not an emergency!!

• DO NOTHING!!!!!! (Reassure) • Avoid straining

33 The Eye Emergencies

Trauma Non- trauma

Painful Painless • Direct blow • Foreign body Does proparacaine relieve pain? • Chemical Yes: No: • U/V keratitis -Conjunctivitis -Keratitis Iritis/Uveitis -Corneal ulcer Acute glaucoma The Eye Emergencies

Trauma Non- trauma

Painful Painless • Direct blow • Foreign body Visual change • Chemical - Posterior vitreous detachment - Retinal detachment • U/V keratitis - Retinal artery/vein occlusion - Ischemic optic neuropathy (includes temporal arteritis) ENT Emergencies

• See ENT and Pediatrics lecture • See Supplemental Slides (end of slide set) (Blunt) Chest Trauma

• Chest wall Rib fracture: #4-9 most common – painful! Flail chest: 3 ribs with >2 fractures each • Lungs/pleura Hemo/pneumothorax Pulmonary contusion • Heart/great vessels

Image reprinted with permission from Michael AJ Sawyer, MD, published by Medscape Reference (http://emedicine.medscape.com/), 2014, available at: http://emedicine.medscape.com/article/428723-overview. (Blunt) Chest Trauma

• Chest wall – Rib fracture: #4-9 most common – painful! – Flail chest: 3 ribs with >2 fractures each • Lungs/pleura – Hemo/pneumothorax – Pulmonary contusion • Heart/great vessels – Aortic tear – Cardiac tamponade (Blunt) Chest Trauma: Commotio Cordis

• 216 cases in registry • Blunt trauma (e.g., ball) • Strikes before peak of T wave V Fib (Blunt) Abdominal Trauma • #1: Spleen • At risk: • Mononucleosis, malaria, hematologic disease • Kehr’s sign: blood irritates diaphragm=> L shoulder pain • Post- splenectomy vaccines • Pneumococcal >14 days before/ • HIB • Meningococcal after splenectomy • #2: Recommended Vaccinations for Asplenic Patients.

Rubin LG, Schaffner W. N Engl J Med July 24, 2014;371:349-356. 3. A 31 yo male is brought to the ED after a steel scaffold struck and pinned his left lower extremity for 2 hrs before extrication. On arrival in ED: VS: stable, ABCs are normal. Left leg: hip to ankle is swollen, ecchymotic and tender. Sensation intact. 2+ DP, PT present Labs: H/H=13/37, Urine dipstick: (+) blood Urinalysis: 0-2 RBCs, 0-2 WBCs/hpf This patient should be admitted for the management of: A. Rhabdomyolysis B. Compartment syndrome C. Renal contusion D. Hemorrhagic shock Rhabdomyolysis

• First described in WWII London bombings • Seen in: Trauma, seizures, burns, drug overdose, exertion, toxin/drug induced • Urine dipstick positive 50% (myoglobinuria) • Elevated CPK (>2-3x reference) • Complication: Acute renal failure 30-40% • Treatment: • Crystalloid 500 cc/hr => Urine output 200-300 cc/hr Compartment Syndrome

• Can occur in anywhere perfusion pressure falls below tissue pressure in any anatomic space • >30 mm Hg*** • Classic: extremities, but any compartment susceptible • Classic: trauma to extremity – but can occur with exercise • Clues: severe pain, decreased sensation, pain on passive stretch, tense extremities • 4-6 hours before irreversible damage • Do NOT wait for pallor, pulselessness • Caution: open fractures are NOT immune from developing compartment syndromes Wound Care 4. A 37 y/o male in good health presents with a laceration of the 5th finger due to broken glass. Which one of the following statements is true? A. Sterile gloves during wound repair reduce the rate of subsequent wound infection compared to the use of clean, non-sterile gloves. B. Local anesthetic containing epinephrine is not to be used in fingers, toes, tip of the nose and penis in healthy patients. C. Wound irrigation with tap water has been shown to have comparable (if not lower) rates of wound infection than sterile saline. D. X-rays are only indicated if the glass is known to contain lead. Wound Care: Tetanus

A. Tetanus Clostridium tetani – anaerobic Gr. (+), ubiquitous, soil 233 cases in US from 2001-2008 (approx. 30 cases/yr) Elderly, immigrants

A. Tetanus Prophylaxis Clean wound >10 years since last dose Dirty wound >5 years since last dose Safe in pregnancy

Substitute Tdap for Td once, then Td booster every 10 years Can I Irrigate With Tap Water? Cochrane Review: 2012 •Meta analysis of 5 studies tap water vs. normal saline ***No difference in rate of wound infection with tap water vs. saline Adults: RR 0.66, 95% CI, 0.42 – 1.04 Trauma: Wound Care Principles • Irrigate, irrigate, irrigate Level of evidence - Pearl: Tap water appears better than saline A • Clean nonsterile gloves do not increase infection rate A

• Non-infected wounds caused by clean objects can be B repaired up to 18 hrs, face/scalp up to 24 hours • If no concern for vascular compromise, epinephrine is B safe in digits • X-ray if you think any glass is possible - Will identify any glass 2 mm or greater

Whom do you place on prophylactic antibiotics? High-risk site – (hand, foot), High-risk mechanism – (bites) High-risk patients (immunocompromised, prosthetic valve) ...and Know the Neurologic Innervation to the Hand!!!

•Radial nerve •Median nerve See Supplemental Slides for description of each •Ulnar nerve Burns See Dermatology lecture and Supplemental Slides

nd 1st degree burn arm Superficial 2nd degree burn of abdomen Deep partial 2 degree of thorax and arm

All photos courtesy of American Family Physician, 85: Jan 1, 2012 5. Child abuse should be suspected in all of the following EXCEPT:

A. 7-month-old with diffuse cerebral and retinal hemorrhages B. 2-year-old with 3 rib fractures after a fall C. 6-month-old with multiple lower ext bruises from falls D. 20-month-old with spiral fracture of distal tibia Child Abuse: Wide spectrum of injuries - Includes: Burns, contusions, fractures, head injury Shaken Baby syndrome: Diffuse cerebral injury with edema, +/- intracerebral bleed, retinal hemorrhages

3 Pearls 1. “If they don’t cruise, they don’t bruise” 2. Rib fractures – < age 3, 82% are abuse 62 children, Univ. Colorado, Barsness, KA,et al. J Trauma 2003 3. Undiagnosed==> > 25% mortality in 2 yrs Toddler’s Fracture – Is Not “Abuse”

• Most common fx in age 9 mos – 3 yrs that present with a limp (29 of 100) • Spiral fracture of distal tibia • Best seen on oblique view • May be occult • Below knee walking cast x 3 weeks Thank you! Answers

1. C 2. D 3. A 4. C 5. D Supplementary Slides B. Arrhythmias: Bradycardia: •Definition: Heart rate < 50

Evaluate potential causes (MI, heart block, drug overdoses…)

Stable: Monitor Unstable: Atropine

• Transcutaneous pacing • Dopamine

57 B. Arrhythmias: Tachycardia:

Stable: Unstable Cardioversion (see next slide) Narrow QRS Wide QRS <0.12 sec > 0.12 sec

Regular rhythm Irregular rhythm Monomorphic (PSVT) (rapid atrial fibrillation)

A fib with BBB Stable Vent tachycardia • Vagal maneuver • Adenosine • B-Blocker • B-Blocker • Antiarrhythmic agent • Ca+ Channel blocker • Ca+ Channel blocker • Expert Consultation

58 “Shockable” rhythm: V fib/V tach: Amiodarone vs. lidocaine vs. placebo •Methods: Randomized, double-blind trial of adults with out-of- hospital refractory V fib/V tach

•Results: amiodarone 300mg lidocaine120mg placebo (n= 970) (n=993) (n=1056) survive to hospital discharge: 24.4% 23.7% 21.0% (95% CI, -0.4 – 7.0,; p=0.08) Secondary outcome: Neurologic outcome in survivors: No difference

Kudenchuk PJ, et al. NEJM May 5, 2016 Sepsis: New Definition (SCCM 2016)

•“life-threatening organ dysfunction caused by” … infection”

SOFA score: > 2 Clinical suspicion • Severe Sepsis

• Septic Shock: Sepsis + a) lactate > 2 AND b) vasopressor needed to maintain MAP > 65 JAMA Feb 23, 2016 Sepsis: New Definition (SCCM 2016)

•“life-threatening organ dysfunction caused by” … infection”

SOFA score: > 2 Clinical suspicion

Early Recognition: qSOFA score >2 or more • RR > 22 • Altered mental status • BPsys < 100

Trauma Prevention • 27.6 million people were treated in an ED for injuries in 2015 • 2.8 million people were hospitalized due to injuries in 2015 • Each year 214,000 people die from injury—1 person every 3 min - #2 #1: MVA – 33,804/yr (2013) 37,461 (2016) - #1 : Firearms - 39,773 (2017)

Statistics from CDC, December 2018 ENT Emergencies: Ears

• Otitis media and Otitis Externa – (covered in Special Sensory)

• Foreign bodies −Watch out for button batteries – Need urgent consultation!

• Auricular hematoma − Needs I&D, w/o results in cauliflower ear. “Scrum cap” ENT Emergencies: Nose

• Epistaxis: − Anterior: packing, silver nitrate sticks, electrocautery, topical tranexamic acid (TXA) − Posterior – balloon packing • Foreign bodies −Watch out for button batteries Am Fam Physician. 2005 Jan 15;71(2):305-311 • Nasal fracture −No x-rays needed −Check for septal hematoma • Needs I&D, w/o results in “saddle deformity”

Am Fam Physician. 2004 Oct 1;70(7):1315-1320 ENT Emergencies: Dental Trauma •Tooth fracture −Ellis I – enamel only −Ellis II – enamel + dentin −Ellis III – includes pulp

•Tooth avulsion

−Immediate replace/reinsert adult tooth www.wikipedia.org • Gently cleansed, do not scrub −Do not replace/reinsert baby tooth Courtesy of SlideShare Author: Rea Corpuz PECARN-2009 • 42,412 children eval for head injury (10,718 age < 2 years) • 14,969 got CT: 376 (0.9%) with TBI, 60 (0.1%) underwent neurosurgery • Derived rule (33,785 pts) and validated (8,627 pts)

Age < 2 yrs: Age > 2 yrs: - Normal mental status - Normal mental status - No scalp hematoma except frontal - No LOC - No LOC or < 5 sec - No vomiting - Non-severe mechanism of injury - Non-severe injury mechanism - No palpable skull fracture - No signs of skull fracture - Acting normal per parents - No severe headache Derived: 98.6% sensitive (95%CI,92-99%) 96.7% sens (95%CI,93-98% Validated: 100% sensitive (95%CI,86-100%) 96.8% sens (95%CI,89-99%) Refer to mdcalc.com ENT Emergencies: Throat • Parapharyngeal infection (Lemierre’s Disease) − Septic thrombophelbitis of the Internal jugular vein − Typical age group 15-30 − Fusobacterium necrophorum: Gram Neg(-) anaerobe

• Peritonsillar abscess −Most common 20-40 yrs of age • Retropharyngeal abscess −Most common 2-4 yrs of age • Epiglottitis −More common in adults than children Adult Head Injury: Is There Any Clinical Decision Rule You Can Rely On?

•Answer: No!! •Many have tried… none are reliable enough!!!! •- Canadian rules (Lancet, 2001) •- Scandinavian rules (J Trauma, 2000) •- New Orleans rules (JAMA, 2005) •- NICE (2004) •- NEXUS II (J Trauma-Injury Inf & Crit Care,2005) Can I Irrigate with Tap Water? Weiss EA, et al. BMJ Open 2013 •Methods: double-blind, randomized 625 subjects to:

•Results: 500ml sterile saline vs. 500ml tap water Rate of infection: 6.4% 3.5% no difference (95%CI, -0.4 – 5.5%) Nerve Function: Hand Radial Nerve • Sensory: Posterior hand – thumb to radial 1/2 of ring finger

• Motor: Wrist and finger extension Test against resistance Nerve Function: Hand Median Nerve • Sensory: Palmer surface, thumb to radial 1/2 of ring finger • Motor: Flexion of wrist and fingers • Best test: Make “OK” sign

“OK” Deficit Nerve Function: Hand Ulnar Nerve

• Sensory: little finger and ulnar 1/2 of ring finger • Motor: innervates interosseous muscles (intrinsics) Test: Abduction of fingers Trauma Prevention: Burns • 500,000+ cases/yr in US ED’s • Scalding accounts for 80% of burns in young children - Flame related injuries are more common in older children • Maintain a functional smoke alarm - 2/3 of homes in which children are injured or killed by fire have no functional smoke detectors Burn Identification

•1st degree – involve only epidermis – a bad sunburn •2nd degree - Superficial partial thickness- epidermis plus upper layers of papillary dermis - Deep partial thickness - - involves deeper layers of dermis (reticular dermis) •3rd degree (full-thickness) – down to and including subcutaneous fat (requires skin grafting) •4th degree – includes muscle, tendon, bone Burn: Care • Minor thermal burns should be treated immediately with cool running water (Level of evidence: C) • The blisters…. in general, leave alone • Superficial burns can be treated with topical application of lotion, honey, aloe vera or antibiotic ointment. (Level of evidence: B)

Silver sulfadiazine: - “the largest volume of evidence suggests that topical silver sulfasdiazine is associated with a significant increase in rates of burn wound infection and increased hospital LOS…” (Cochrane Library, 2013) - Compared to honey (4 trials, low quality) – wounds healed quicker with honey (Cochrane Library, 2015) - Compared to biosynthetic dressings – wounds healed quicker with dressings (DuoDerm, OpSite, etc) ( Cochrane Library, 2013) © 2020 American Academy of Family Physicians. All rights reserved.

All materials/content herein are protected by copyright and are for the sole, personal use of the user.

No part of the materials/content may be copied, duplicated, distributed or retransmitted

in any form or medium without the prior permission of the applicable copyright owner.

Common ENT Problems

Teresa Holt, M.D. Associate Program Director Grant Family Medicine Residency

Nanette Lacuesta, M.D Associate Program Director Riverside Methodist Hospital Family Medicine Residency OhioHealth, Columbus, Ohio Disclosure Statement

It is the policy of the AAFP that all individuals in a position to control content disclose any relationships with commercial interests upon nomination/invitation of participation. Disclosure documents are reviewed for potential conflicts of interest. If conflicts are identified, they are resolved prior to confirmation of participation. Only participants who have no conflict of interest or who agree to an identified resolution process prior to their participation were involved in this CME activity.

All individuals in a position to control content for this session have indicated they have no relevant financial relationships to disclose. Learning Objectives

• Recognize the diagnosis, management and ancillary testing for the following: − Acute and chronic otitis media − Tinnitus and hearing loss − Acute and chronic sinusitis − Allergic rhinitis and chronic cough − Laryngitis − ENT Emergencies − Vertigo − Bells Palsy 1. A 12-month-old male patient is brought into your office by his mother for fever, cough and pulling on his left ear. He is afebrile in your office, playful and interactive. When you examine his ear, the tympanic membranes are slightly erythematous, but he is uncooperative for pneumatic otoscopy. Your diagnosis is?

A. URI, acute otitis media uncertain B. Otitis media with effusion C. Acute otitis media D. Ramsay Hunt syndrome Diagnosis of Acute Otitis Media

• Must use stringent criteria ‾ Ensure appropriate treatment ‾ Avoid overuse of antibiotics • Three required elements to make the diagnosis ‾ Acute onset of symptoms of otalgia ‾ Presence of a middle ear effusion ‾ Acute signs of middle ear inflammation

http://pediatrics.aappublications.org/content/early/2013/02/20/peds.2012-3488 SORT B Symptoms of Acute Otitis Media

• Onset within 48 hours • “Severe” symptoms Moderate to severe pain Fever >39oC (102.2oF) Non-verbal child (<6 months old) • Holding, tugging, or rubbing of ear • Poor eating, irritability

http://pediatrics.aappublications.org/content/early/2013/02/20/peds.2012-3488 SORT B Diagnosis of Middle Ear Effusion • Decreased or absent mobility of TM ‾ Do not diagnose without demonstrating MEE ‾ Pneumatic otoscopy – standard of care ‾ Better sensitivity and specificity than tympanometry and Weber Test ‾ Tympanometry

http://pediatrics.aappublications.org/content/early/2013/02/20/peds.2012-3488 SORT B Diagnosis of Middle Ear Inflammation

• Bulging and fullness of the TM • Bullae on the TM • Acute purulent otorrhea (not from otitis externa) • Marked erythema, BUT… ‾ An erythematous TM in the absence of bulging or immobility of the TM has only a 15% PPV for AOM ‾ An erythematous TM can be caused by fever, crying, and URI Otitis Media With Effusion (OME)

• Signs of middle ear fluid ‾ Impaired TM mobility ‾ Air fluid levels ‾ Bubbles ‾ Amber or blue color • Absence of signs of acute inflammation • Symptoms- slower onset, less severe, no fever Acute Otitis Media Otitis Media with Effusion TM position Bulging Retracted or Neutral TM color Red or Yellow Amber or Blue Signs/Symptoms Pus, Otorrhea, or Bullae Air fluid levels or Bubbles

(c) B. Welleschik (c) B. Welleschik https://commons.wikimedia.org/wiki/File:Otitis_media_entdifferenziert2.jpg Tympanometry

• Quantifies pneumatic otoscopy • Considered if otoscopy indeterminant or OME uncertain • Not for kids <7mo (canals too compliant) • Measures: ‾ Ear canal volume (cm3) ‾ Compliance (cm3) ‾ Pressures Type A curve Type B curve—always abnormal Type C curve ‾ Normal compliance ‾ Decreased compliance Less helpful — low Sn, Sp and pressures ‾ MEE ‾ Negative pressures ‾ Stiff TM from ‾ Retracted TM • Scarring ‾ Eustachian tube • Tympanosclerosis dysfunction • Cholesteatoma • Tumor 2. A 22-month-old male patient is brought to you crying and in obvious acute distress from right ear pain. He has a fever of 103.6oF, has an immobile, bulging, erythematous right TM. The best treatment option would be? A. Do not use antibiotics because this is probably a viral illness B. Have the parents observe for 24-48 hours and treat with antibiotics if the child does not improve C. Treat with antihistamines/decongestants alone D. Start antibiotics immediately Treatment of AOM

• Pain control ‾ Use ibuprofen or acetaminophen • Decongestants/Antihistamines ‾ Not proven to help ‾ Increased side effects ‾ May relieve nasal congestion in older children Treatment of AOM • Three bacterial pathogens ‾ S. pneumoniae ‾ H. influenzae ‾ M. catarrhalis • Antibiotics or Observation? ‾ First studies out of Europe ‾ Concerns about over usage of antibiotics ‾ Several meta-analyses suggest most children do well without ATBs • 61% resolve symptoms within 24 hours Treatment of AOM 2013 AAP/AAFP Guideline

Age < 6 months 6-23 months >/= 24 months Observation N/A if suspect AOM Unilateral Uni-/Bilateral AND Symptoms are AND Symptoms are NOT severe NOT severe

Antibiotics Suspect AOM Bilateral Uni-/Bilateral AND Symptoms are AND Symptoms are NOT severe Severe

OR Uni/Bilateral AND Symptoms are severe

http://pediatrics.aappublications.org/content/early/2013/02/20/peds.2012-3488 SORT B Observation of AOM 2013 AAP/AAFP Guideline

• Observation if symptoms worsen or persist • Based on shared decision-making with parents • Is only appropriate if follow-up can be ensured in 48-72 hours • Reexamination • Phone call • Prescription can be filled

Little P, Moore M, Kelly J, et al.; PIPS Investigators. Delayed antibiotic prescribing strategies for respiratory tract infections in primary care: pragmatic, randomised controlled trial. BMJ. 2014;348:g1606. "Antibiotics for ear infections in children" 3.You diagnose AOM with severe symptoms in a 22- month-old male patient who weighs 20 kg (44 lb). What is the most appropriate treatment?

A. Amoxicillin 900 mg/day B. Amoxicillin 1800 mg/day C. Amoxicillin-clavulanate 900 mg/day D. Amoxicillin-clavulanate 1800 mg/day Antibiotics for AOM 2013 AAP/AAFP Guideline

• Amoxicillin 80-90 mg/kg per day • Amoxicillin-clavulanate for those: ‾ Treated with ATBs in last 30 days ‾ With concurrent conjunctivitis (H. influenzae) ‾ Taking prophylactic amoxicillin for recurrent AOM (no longer recommended) Antibiotics for AOM 2013 AAP/AAFP Guideline • Penicillin allergy – No urticaria or anaphylaxis (Non-type 1) • Cephalosporins ‾ Cefdinir (Omnicef) 14 mg/kg per day ‾ Cefpodoxime 10 mg/kg per day ‾ Cefuroxime (Ceftin) 30 mg/kg per day ‾ Ceftriaxone 50 mg/kg IM/IV Antibiotics for AOM 2013 AAP/AAFP Guideline • Penicillin allergy with urticaria or anaphylaxis (Type 1) ‾ Macrolides • Erythromycin + sulfisoxazole • Azithromycin, clarithromycin ‾ Clindamycin • Not recommended due to resistance ‾ Trimethoprim-sulfamethoxazole ‾ Levofloxacin Antibiotics for AOM 2013 AAP/AAFP Guideline • Duration of treatment • 10-day course of antibiotics ‾ Reduced course of antibiotics showed less favorable outcomes ‾ Neither adverse effects or emergence of drug resistance was lower with reduced length of treatment

http://www.nejm.org/doi/pdf/10.1056/NEJMoa1606043 4. You treated a child with AOM with antibiotics and see him back 4 weeks later. The child is asymptomatic, but you determine he has a middle ear effusion. Your recommendation would be? A. Reassurance and reevaluate in 2 months B. Oral antihistamine for 30 days C. Re-treat with amoxicillin-clavulanate D. Oral low-dose steroids for 30 days E. ENT referral Follow-up for AOM • Monitor middle ear effusion (MEE) • Does not mean treatment failure ‾ 70% had MEE after two weeks ‾ 40% after one month ‾ 20% after two months ‾ 10% after three months • Follow-up recommended at 8-12 weeks • Monitor for hearing, language and learning problems

http://www.uptodate.com/online/content/topic.do?topicKey=pedi_id/10593&selectedTitle=1~150&source=search_result SORT C Persistent AOM

• No improvement in 48-72 hours • Reassess to confirm diagnosis • Switch to second-line antibiotic – assume resistant bacteria (penicillin resistant S. pneumoniae) • Amoxicillin/Clavulanate • Cephalosporin • Macrolide • Clindamycin Recurrent AOM

• Antibiotic prophylaxis ₋ Specifically NOT recommended ₋ 2013 AAP/AAFP Guidelines • Minimize risk factors ₋ Exposure to cigarette smoke ₋ Pacifier, bottle feeding ₋ Daycare attendance Treatment of Otitis Media With Effusion

• Watchful waiting for three months • Test for hearing loss at three months • Tympanostomy tubes is preferred procedure <4 years • Tympanostomy tubes, adenoidectomy or both >4 years • Do NOT use antibiotics, antihistamines, decongestants or steroids • Tubes now controversial- AAFP POEMs 2018- 16 RCTs no hearing benefit Reproduced with permission from at 12-24 months Otolaryngology Houston www.ghorayeb.com http://www.houstonoto.com/OtitisMedia PEtube.html

Otolaryngol Head Neck Surg. February 2016;154(suppl 1):S1–S41. Am Fam Physician. 2016 Nov 1;94(9):747-749. SORT A/B Tinnitus •Perception of noise in the absence of an acoustic stimulus •Most commonly idiopathic, then sensorineural hearing loss •Other causes: cerumen impaction, effusion, cholesteatoma, HTN, TMJ, vascular, neoplastic, neurologic, meds •Meds: NSAIDs, Antimicrobials, antineoplastics, loop diuretics, others. All work up other than history and physical is SORT C Hearing loss or vertigo: eval for Meniere’s dz with audiometry, electronystagmography, MRI Pulsatile: neurovascular imaging Unilateral: eval for vestibular schwannoma with audiometry, MRI Diagnostic Approach to Patients with Tinnitus. Yew, KS. Am Fam Physician. 2014 Jan 15;89(2):106-113. Sudden Sensorineural Hearing Loss

•30dB loss at 3 consecutive frequencies with normal exam otherwise

•Idiopathic 85-90%

•Treatment- oral corticosteroids 1mg/kg/d- max of 60mg x 10-14 d −Equivocal benefit but low risk, best benefit if started right away (first 2 weeks of sx) −No benefit for: antivirals, vasodilators (calcium channel blockers), anti-platelets, or antibiotics unless evidence of AOM on exam

• Otolaryngol Head Neck Surg 2012;146(3 Suppl):S1-S35. 5. A 43-year-old male presents to your office with five days of nasal congestion and headache. His temperature is 100.8oF, he has purulent rhinorrhea, and minimal tenderness to palpation over the frontal and maxillary sinuses. Your next step would be?

A. Transilluminate the sinuses B. Get sinus x-rays C. Treat with decongestants/mucolytics D. Treat with amoxicillin-clavulanate Acute Rhinosinusitis

Symptoms <4 weeks −Purulent nasal drainage −Nasal obstruction −Facial pain, pressure or fullness Etiology −Viral is most common etiology −Bacterial rhinosinusitis – 0.5-2% Indicators of Bacterial Rhinosinusitis

• Duration of symptoms >7 days • Worsening of symptoms • Moderate to severe pain and fever >101oF • Bimodal illness – worsening of symptoms after initial improvement

Am Fam Physician. 2016 Jul 15;94(2):97-105 SORT C Non-helpful Tests • Trans-illumination of sinuses • Viral or bacterial cultures (except endoscopic) • Diagnostic imaging is not recommended unless a complication or alternative diagnosis is suspected • Sinus x-rays • Sinus CT scans • MRI or ultrasound

Am Fam Physician. 2016 Jul 15;94(2):97-105 SORT C Treatment of Viral Rhinosinusitis • Self-limited disease – treatment does not shorten the course. • Supportive care: SORT A • Analgesics (NSAIDs, acetaminophen) • Saline nasal sprays (irrigation) • Intranasal corticosteroids • Consider Ipratropium- NIH data for rhinorrhea only, not pain or congestion Topical decongestants: do not use longer than 72 hours, risk of rebound congestion • Not recommended • Oral decongestants: no RCT evaluating effectiveness • Antihistamines: worsen congestion by over-drying mucosa; not recommended unless history of allergy • Zinc- no benefit for sinusitis (mixed data for URIs) Antibiotic Treatment of Bacterial Rhinosinusitis

• In acute, uncomplicated watchful waiting without abx is an appropriate initial management if assurance of follow up SORT A • Observation should include supportive treatment for seven days – 70% resolve by 1 week • Antibiotics recommended if sx persist > 7 days without clinical improvement or if worsening SORT C

Am Fam Physician. 2016 Jul 15;94(2):97-105 Avoid prescribing antibiotics in ED for uncomplicated sinusitis

Do not routinely obtain radiographic imaging for pts who meet diagnostic criteria for uncomplicated acute rhinosinusitis

Do not routinely prescribe antibiotics for acute, mild to moderate sinusitis unless sx (purulent nasal secretions, maxillary pain, or facial/dental tenderness to percussion) last at least 7 days or xs worsen after initial clinical improvement Antibiotics for Bacterial Rhinosinusitis Start empiric antibiotics if • Onset with severe symptoms or signs of high fever >102oF and purulent nasal discharge or facial pain lasting at least 3-4 days • Onset with persistent symptoms lasting 10 days without improvement • Onset with worsening symptoms, new onset of symptoms lasting 5-6 days after initially improving (“double sickening”)

IDSA Clinical Practice Guideline. Clinical Infectious Diseases 2012;54(8):1041–5 Antibiotics for Bacterial Rhinosinusitis (IDSA)

Amoxicillin-clavulanate now recommended as empiric choice due to antibiotic resistance

Duration: 7-10 days adults, 10-14 days for kids

High dose (1 g BID or 90 mg/kg/day) if: >10% penicillin resistant S. pneumonia Recent hospitalization Abx use within past month Daycare attendance <2 yo or >65 yo Severe infections Immunocompromised IDSA Clinical Practice Guideline. Clinical Infectious Diseases 2012;54(8):1041–5 SORT C Antibiotics for Bacterial Rhinosinusitis (IDSA)

• Adults: Fluoroquinolones and doxycycline No longer recommended due to antimicrobial resistance: ‾ Macrolides ‾ Trimethoprim-sulfamethoxazole ‾ Cephalosporins • Kids: cephalosporins

IDSA Clinical Practice Guideline. Clinical Infectious Diseases 2012;54(8):1041–5 Treatment of Bacterial Rhinosinusitis

•Same adjunctive therapy as for viral rhinosinusitis (NSAIDs, saline spray, topical ipratropium, mucolytics) • What about nasal steroids? Reduces inflammation and swelling of nasal mucosa Meta-analysis showed increased symptom response, especially milder disease

http://onlinelibrary.wiley.com/o/cochrane/clsysrev/articles/CD005149/frame.html http://www.ncbi.nlm.nih.gov/pubmed?term=18056902 SORT B Bacterial Rhinosinusitis in Children

AAP guidelines in 2013 made several changes • Added worsening of symptoms after initial improvement to diagnostic criteria • Added observation period of three days after 10-day illness • Amoxicillin (80-90 mg/kg) is the drug of choice for children • Amoxicillin-clavulanate or ceftriaxone are alternatives • Recommend against any imaging studies

http://pediatrics.aappublications.org/content/early/2013/06/19/peds.2013-1071.full.pdf SORT C Chronic Rhinosinusitis

• 12 weeks of symptoms ‾ Nasal obstruction ‾ Facial pain ‾ Mucopurulent drainage ‾ Decreased sense of smell • Most patients cannot be cured – control or reduce symptoms • Complex inflammatory changes rather than persistent bacterial infection Chronic Rhinosinusitis

• Saline nasal sprays/irrigations – low pressure/ high volume Neti pots better than sprays (SOR B) • Intranasal steroids – sprays and instillations • Oral steroids • Leukotriene antagonists • Referral to ENT specialist Topical and oral antimicrobials if endoscopic culture positive

Am Fam Physician. 2017 Oct 15;96(8):500-506 Allergic Rhinitis •IgE mediated rhinitis •Persistent: >4 d/wk for >4 weeks −Tx: allergen avoidance, intranasal steroids •Intermittent- less than 4 d per week −Tx: nasal saline spray

•Second Line tx −Antihistamines (SOR B)- increased side effects and less effectiveness •Not Helpful for either: −Pillow/ mattress covers are not helpful (SOR A) −Imaging: Choose Wisely Campaign for ENT

Am Fam Physician. 2015 Dec 1;92(11):985-992 Chronic Cough

•Cough >8 weeks •Eval: exam, CXR, check for ACE •Most common causes: −GERD: empiric high dose PPI for 2-3 months, consider pH probe or EGD −Postnasal drip: nasal steroid −Asthma: trial inhalers, consider spirometry

•If above treatments fail, consider CT chest or bronchoscopy

N Engl J Med 2016;375(16):1544-1551 Laryngitis

•Acute- hoarseness for <3-4weeks −Causes: URI, vocal strain (due to vocal fold edema), allergic rhinitis −Tx: complete voice rest NOT: whispering, clearing throat, ABX, inhaled steroids, nebulized saline •Chronic: >3-4weeks −Causes: GERD, smoking, allergic rhinitis, vocal fold polyps, head/ neck , neurologic −Eval: Laryngoscopy (SOR C)- if >3 months of sx if >2 weeks if high risk (smoker, heavy EtOH, hemoptysis) −Treatment: tx the underlying cause (PPI, polypectomy, etc.) voice therapy (SOR A)

Am Fam Physician 2017;96(11):720-728. ENT Emergencies: Ears

• Otitis media and Otitis Externa – (covered in Special Sensory)

• Foreign bodies −Watch out for button batteries – Need urgent consultation!

• Auricular hematoma − Needs I&D, w/o results in cauliflower ear. “Scrum cap” ENT Emergencies: Nose

• Epistaxis − Anterior: packing, silver nitrate sticks, electrocautery, topical tranexamic acid (TXA) − Posterior – balloon packing • Foreign bodies −Watch out for button batteries Am Fam Physician. 2005 Jan 15;71(2):305-311 • Nasal fracture −No x-rays needed −Check for septal hematoma • Needs I&D, w/o results in “saddle deformity”

Am Fam Physician. 2004 Oct 1;70(7):1315-1320 ENT Emergencies: Dental Trauma •Tooth fracture −Ellis I – enamel only −Ellis II – enamel + dentin −Ellis III – includes pulp

•Tooth avulsion

−Immediate replace/reinsert adult tooth www.wikipedia.org • Gently cleansed, do not scrub

Courtesy of SlideShare −Do not replace/reinsert baby tooth Author: Rea Corpuz 6. A 57-year-old female patient of yours presents with dizziness and a sensation that she is spinning. It occurs when she turns in bed or lifts her head to look in an upper cabinet. Episodes are brief but are becoming more frequent. She has no tinnitus or hearing loss. The most likely diagnosis would be?

A. Meniere’s disease B. Benign paroxysmal positional vertigo C. Vestibular neuronitis D. Acoustic neuroma Vertigo Illusion of movement ‾ Spinning, tilting, swaying ‾ Subjective or objective (patient or environment) Must be distinguished from presyncopal faintness and dysequilibrium Central and peripheral causes

Peripheral Vertigo Central Vertigo Imbalance Mild-Moderate Severe; cannot stand Hearing Loss Yes No Tinnitus Yes No Other Neuro Symptoms No Yes Peripheral Vertigo • Benign paroxysmal positional vertigo ‾ Canalithiasis ‾ Torsional up-beating nystagmus (85-95% in posterior semicircular canal) ‾ Brief spinning spells (seconds) when head moved ‾ , but rarely ‾ No hearing loss, ear pain or tinnitus

• Vestibular Neuronitis ‾ Viral or post-viral inflammation of labyrinth ‾ Saccade, horizontal nystagmus ‾ With unilateral hearing loss, it is called “labyrinthitis” ‾ Lasts 1-2 days before resolution

Am Fam Physician. 2017 Feb 1;95(3):154-162. Peripheral Vertigo • Meniere’s Disease ‾ Endolymphatic Hydrops ‾ Associated with tinnitus, hearing loss and ear fullness

• Acoustic neuroma ‾ Vertigo is minor, tinnitus and hearing loss are main complaints

• Herpes Zoster Oticus (Ramsay Hunt Syndrome) Courtesy: Dr. Christopher Chang, Fauquier ENT Hearing loss, vertigo, taste loss, facial paralysis

Am Fam Physician. 2017 Feb 1;95(3):154-162. Central Vertigo

• Migrainous vertigo

• Wallenberg’s syndrome ‾ Infarction of lateral medulla

• Cerebellar hemorrhage or infarction ‾ Sudden intense vertigo and vomiting ‾ Markedly impaired gait – falls to the side of the lesion Evaluation of Vertigo

• Careful history • Neurological exam • Lab tests help <1% of the time • MRI: if other neuro signs, hearing loss • Dix-Hallpike Maneuver

http://www.aafp.org/afp/2017/0201/p154.html http://www.nejm.org/doi/full/10.1056/NEJMcp1309481 Treatment of Vertigo

• Medications ‾ Most useful for vertigo that lasts hours or days – not BPPV ‾ Lots of sedation as well as risk of falls and urinary retention in older patients ‾ Anticholinergics – scopolamine ‾ Antihistamines – meclizine, dimenhydrinate ‾ Phenothiazines – promethazine, metoclopramide ‾ Benzodiazepines – diazepam, lorazepam Treatment of Vertigo

• Vestibular Rehabilitation (PT) • CNS compensation for peripheral vestibular injury – ? Central? • When started early, balance and function are improved compared with controls • Home exercises also effective

http://www.uptodate.com/online/content/topic.do?topicKey=genneuro/5875&selectedTitle=1~150&source=search_result SORT B Treatment of Vertigo

•Benign Paroxysmal Positional Vertigo ‾ Medications generally not helpful ‾ Canalith repositioning – Epley Maneuver

http://www.aafp.org/afp/20050315/1115.html Bells Palsy

•Idiopathic facial paralysis •Early recognition and treatment decreases risk of chronic partial paralysis and pain. •DDx: stroke and other central pathology of facial nerve spare the forehead •Antivirals/corticosteroids (NNT 15) more effective than either agent alone. •Do not use antivirals alone to treat (SORT A)

Antiviral Therapy for Bells Palsy. Seehusen, DA and Egitto, EA. Am Fam Physician. 2016 May 1;93(9):742-743. Albers, JR, Tamang S. Common Questions about Bell Palsy. Am Fam Physician. 2014 Feb 1;89(3):209-212. Board Frequent Fliers

•AOM dx- need all 3: sx (F>102, pain onset 48 hrs), effusion, inflammation •AOM ABX if: <6mo, bilateral/ severe 6mo-23mo, severe 24+ mo •AOM ABX = Amox 80-90mg/kg/d •Tinnitus- causes: idiopathic, then sensorineural hearing loss •Acute sinusitis ABX if: >7d, worsening sx, bimodal, F >101- use augmentin adults, augmentin or amox in kids •Chronic sinusitis- no ABX •Laryngitis- tx total voice rest •BPPV- torsional up-beating nystagmus •Bells palsy- tx antiviral and steroids Answers

1. A 2. D 3. B 4. A 5. C 6. B © 2020 American Academy of Family Physicians. All rights reserved.

All materials/content herein are protected by copyright and are for the sole, personal use of the user.

No part of the materials/content may be copied, duplicated, distributed or retransmitted

in any form or medium without the prior permission of the applicable copyright owner.

Emergency Medicine: Part 2

Robert Dachs, MD, FAAFP Clinical Associate Professor Ellis Hospital Family Medicine Residency Program Albany Medical College, Albany, New York Disclosure Statement

It is the policy of the AAFP that all individuals in a position to control content disclose any relationships with commercial interests upon nomination/invitation of participation. Disclosure documents are reviewed for potential conflicts of interest. If conflicts are identified, they are resolved prior to confirmation of participation. Only participants who have no conflict of interest or who agree to an identified resolution process prior to their participation were involved in this CME activity.

All individuals in a position to control content for this session have indicated they have no relevant financial relationships to disclose. Learning Objectives

1. Appropriately manage a variety of soft tissue infections, including animal bites. 2. Appropriately manage tick-borne illness. 3. Recognize and manage acute allergic reactions. 4. Recognize and determine which toxicology emergencies require specific antidotes. Part 2

•Soft tissue infection • Acute allergic reactions - Cellulitis vs. CA-MRSA - Acute management • Animal Bites - ACE-induced angioedema - Cat • Environmental injuries - Dog - Cold-related - Human - Heat-related - Other • Toxicology •Tick-related illness • Gastric decontamination - Lyme disease • Antidotes - Other tick-related • Common toxidromes Cellulitis

Non-purulent (+) purulent

• Strep pyogenes • CA-MRSA • MSSA Staph. aureus

MSSA MRSA

HA- MRSA CA- MRSA

USA 100, 200, USA 300, 400 500, 600, 700, 800

Panton-Valentine Leukocidin How do they present? exotoxin CA- MRSA: Presentation

“I think I got a spider bite” Cellulitis Treatment Non-purulent (+) purulent

• Strep pyogenes • CA-MRSA • MSSA Drug Adult Dose Incision and Drainage Dicloxacillin 500mg QID [Nafcillin] 1-2g q4h Drug Adult Dose Cephalexin 500mg QID TMP-SMX 1-2 tab BID* [Cefazolin] 1g q8hrs Doxycycline 100mg BID Doxycycline* 100mg BID Clindamycin* 300-450mgTID Clindamycin* 300-450mgTID Linezolid* 600mg BID Linezolid 600mg BID Animal Bites

1. Which of the following bites has the highest risk of infection?

A. Cat bite to the hand B. Human bite to the face C. Dog bite to the thigh D. Spider bite to the arm A. Cat Bite

• 5-18% of all reported bites • Puncture wounds • 80% of bites become infected - 53-80% with Pasteurella multocida - Watch for bone and joint infection

RX: Amoxicillin-clavulanate B. Human Bite • Watch for closed fist injury • High rate of infection, 26-83% polymicrobial

Viridans Streptococci,100% S. aureus, 29%

Bacteroides species, 82% Peptostreptococcus, 26%

S. epidermidis, 53% Eikenella species, 15% Corynebacterium species, 41%

• Copious irrigation, avoid closure RX: Amoxicillin-clavulanate x 5 days C. Dog Bite • 80-90% of all reported bites (#1) • Most common on extremities • Only 5% of bites develop infection - Higher rate in hands, deep puncture, older pts.

Pasteurella multocida S. aureus, 29% Bacteroides species - Fusobacterium Fusobacterium species EF-4 bacteria Eikenella species, 15% DF-2 bacteria (Capnocytophaga sp)

• Primary closure - OK • +/- Amoxicillin-clavulanate The Long List of Infections .Cat bite Pasteurella multocida Amoxicillin-clavulanate Dog bite Pasteurella multocida, Capnocytophaga sp Amoxicillin-clavulanate Fusobacterium, EF-4, S. aureus, Eikenella Human bite Viridans Streptococci, Bacteroides sp, Amoxicillin-clavulanate Animal hides, Anthrax (Bacillus ciprofloxacin, doxycycline, carcasses anthracis) (skin and lung PCN infection) Fresh water injury Aeromonas sp. TMP-SMX, FQ

Saltwater injury Vibrio fulnificus (skin and GI infection) 3rd generation cephalosporin + mino/doxycycline or FQ Fish tank exposure Mycobacterium marinum Hot compresses, minocycline, clarithromycin Hot tub exposure Pseudomonas aeruginosa Self-limiting, acetic acid compresses, ciprofloxacin Cat Scratch Disease

• 22,000 cases/year, 2,000 hospitalizations/year • Regional lymphadenopathy, 10% suppurative - Axillary/epitrochlear nodes 46% - Cervical 26%, inguinal 17% • Bartonella (Rochalimaea) henselae - small Gr (-) rod • Diagnosis: Cat scratch and serologic testing (IFA) • Self-limiting disease, 1 to 2 months Do NOT I & D!!! Cat Scratch Disease

• Treating with antibiotics • Immunocompromised patients: YES • Immunocompetent patients ?????? • Some say “No” • Some say “Yes” (including Up-to-date) – azithromycin 500mg qd x 5 days *Earlier resolution seen, based on a 1998 study of 29 patients Next Case...

2. A 24 yo male presents with a 3-day history of a rash that is increasing in size. - It is not (-) painful or tender. - It is flat, oval, 14 cm x 7 cm in size, has central clearing and has no fluctuance. - No associated fever/chills or systemic symptoms. - No new medications. No recall of any insect bite. - He recently vacationed on Martha’s Vineyard, MA.

See photo - next page

2. A 24 yo male presents with a 3-day history of a rash that is continuing to increase in size. The rash is not painful or tender. It is flat, oval, 14 cm x 7 cm in size, has central clearing and has no fluctuance. No associated fever/chills or systemic symptoms. No new medications. No recall of any insect bite. He recently vacationed on Martha’s Vineyard, MA.

In this case, you would: A. Prescribe azithromycin 500 mg qd x 5 days B. Prescribe doxycycline 100 mg BID x 10 days C. Obtain a Lyme titer; if positive treat with ceftriaxone. D. Obtain a Lyme titer; if positive, confirm with a Western blot study and if positive treat with ceftriaxone. Lyme Disease • Due to spirochete: Borrelia burgdorferi • Transmitted by: Deer tick –Ixodes scapularis and Ixodes pacificus From left to right, an Ixodes scapularis larva, nymph, adult male tick, and adult female tick.

Wormser G P et al. Clin Infect Dis. 2006; 43:1089-1134 FIGURE 2. Average annual number of confirmed Lyme disease cases, by county of residence* — United States, 2008–2015†

MMWR Surveill Summ. 2017 66 (22): 1-12 Courtesy of the CDC Lyme Disease: Stages

• Early Lyme Disease –Erythema migrans: present 50-70% of cases

Courtesy of Wikipedia Courtesy of the CDC/James Gathany Lyme Disease: Stages

• Early Lyme Disease –Erythema migrans: present 50-70% of cases

–Neurologic disease Cranial neuropathy (7th nerve), radiculopathy, lymphocytic meningitis

–Cardiac disease: think A-V block Lyme Disease: Stages • Late Lyme disease –Lyme arthritis •Large joints, typically knees –Neurologic disease •Encephalopathy •Peripheral neuropathy

• Post-Lyme disease syndromes –“…unexplained chronic subjective symptoms following treatment…”

IDSA guideline, 2006 Lyme Disease: Treatment

Stage 1: Do not test, just treat

Adults • Doxycycline 100 mg BID x 10 days • Amoxicillin 500 mg TID x 14 days • Cefuroxime 500 mg BID x 14 days • Azithromycin 500 mg qd x 7-10 days

Medical Letter, May 9, 2016 Lyme Disease: Treatment

Stage 1: Do not test, just treat

Children • Doxycycline: > age 8, 2 mg/kg BID • Amoxicillin: 50 mg/kg/d divided TID • Cefuroxime 30 mg/kg/d divided BID • Azithromycin 10 mg/kg/d qd

Medical Letter, May 9, 2016 Lyme Disease: Testing

•Stage 1: Do not test, just treat

So whom do you test???? For unexplained symptoms… Lyme Disease: Treatment •Stage 1: Erythema migrans •Doxycycline, Amoxicillin, Cefuroxime, Azithromycin •Early Neurologic Disease −7th cranial nerve palsy ------> oral regimen (14-21days) −Meningitis/radiculopathy------> parenteral (10-28 days) •Lyme carditis ------> oral or parenteral (21-28 days) •Lyme arthritis –------→ oral 28 days Lyme Disease Treatment

• Watch for Jarisch-Herxheimer reaction –Fever, chills, myalgias, headache Remember syphilis… –Treat symptomatically, do not d/c or switch antibiotic Lyme Disease Prophylaxis???

• Routine Abx prophylaxis or serologic testing is NOT recommended (E-III). • A single dose of doxycycline may be offered to adult patients (200 mg) and children ⩾8 years (B-I) if all of the following • attached tick can be reliably identified as an adult or nymphal I. scapularis tick and • that is estimated to have been attached for ⩾36 h and • prophylaxis can be started within 72 h of the time that the tick was removed and • local rate of infection of these ticks with B. burgdorferi is ⩾20% and • doxycycline treatment is not contraindicated. IDSA guideline, 2006 Other Deer Tick-Borne Illnesses • HGA: Human granulocytic anaplasmosis –Previously known as “ehrlichiosis” •Due to Anaplasma phagocytophilum –Within 3 weeks of tick bite… •Fever, chills, and headache, with •Thrombocytopenia, leukopenia, elevated LFTs • Babesiosis –Malaria-like illness with intracellular protozoa –Hemolytic anemia, thrombocytopenia, elevated LFTs All 3 associated with deer tick Other Tick-Borne Illnesses • Rocky Mountain Spotted Fever – Organism: Rickettsia rickettsii, transmitted by • The American dog tick and the Rocky Mountain wood tick. – 90% of cases are April to September. – >50% of cases involve children <15 years old – Symptoms: 5-10 days after tick bite: • Flu-like illness • Rash • Later … multisystem involvement Photo courtesy of CDC Allergic Reactions

3. A 64 yo male presents to the ED with diffuse pruritus and erythema along with facial and oral swelling. This occurred 15 minutes after eating peanuts. His blood pressure is 65/35 mm Hg, pulse is 120 bpm.

See photos - next slide

Allergic Reactions 3. A 64 yo male presents to the ED with diffuse pruritus and erythema along with facial and oral swelling. This occurred 15 minutes after eating peanuts. His blood pressure is 65/35 mm Hg, pulse is 120 bpm. The first medication this patient should receive is: A. Epinephrine B. Diphenhydramine C.Methylprednisolone D.Ranitidine Allergy: Reactions • Urticaria (hives) - IgE mediated • Angioedema - may be 1. IgE-mediated or 2. Idiopathic (ACE-induced), not true allergy Result: Swelling of face, neck, and tongue • Anaphylaxis - may occur within seconds to 1 hour – Skin rash, respiratory symptoms, hypotension, GI distress • Anaphylactoid reactions: Non-immunologic (non-IgE) - Direct release of granules from mast cells and basophils - Not true allergy (Example: radiocontrast) Allergic Reactions: Etiologic Agents

Anaphylactic Anaphylactoid IgE-dependent Non-IgE/nonimmunologic • Food (35%) • Opioids • Medication (20%) • ASA and NSAIDs • Insect venom (20%) IgE • Radiocontrast media • Latex Y • Exercise Allergic Reactions: Etiologic Agents

Anaphylactic Anaphylactoid IgE-dependent Non-IgE/nonimmunologic • Food • Opioids • Medication • ASA and NSAIDs • Insect venom • Radiocontrast media • Latex • Exercise Allergic Reaction: Treatment

• Vasoconstrictors: Epinephrine A. Mild-moderate: 0.3-0.5 cc 1:1000 solution, SQ or IM

B. Severe: 1-5 cc of 1:10,000 solution, IV 1 ug/min (1:10,000) 1 mg in 1L NS at 1 cc/min Allergic Reaction: Treatment

• Vasoconstrictors: Epinephrine

If Epi-Pen is used, refer to ED for follow up Reason: chance of biphasic reaction “second wave”

Note: this is exceedingly rare (5/2,323; 0.18%) Grunau B, et al. Ann Emerg Med, June 2014 Allergic Reactions: Treatment • Vasoconstrictors: Epinephrine *Mild-moderate: 0.3-0.5 cc 1:1000 solution, SQ or IM **Severe: 1-5 cc of 1:10,000 solution, IV ***If pt. on B-Blocker, give glucagon 1-5 mg IV • H1 antagonist: 1st or 2nd generation antihistamines • H2 antagonist: H2-blocker du jour, decreases itch • Steroids - Do nothing for acute episode - May prevent recurrence This is NOT an allergic reaction... ACE-Induced Angioedema

• NOT an allergic reaction • Due to accumulation of bradykinin (?) • Up to 5x more common in African-Americans • Women > Men • Can occur months to years after ACE use • Treatment: supportive Scombroid Poisoning: “Pseudo” Fish Allergy

• Mimics allergic reaction – facial flushing, diaphoresis, hives, edema, , peppery taste • Occurs minutes to 1-2 hours after eating contaminated fish • Classically tuna and mackerel (Scombroidae family), can occur in others • Histidine in muscle converted by bacteria to histamine. • Rx: H1 and H2 blockers • Self-limiting: 4-6 hours Ciguatera Poisoning

• Ingestion of reef fish that have accumulated sufficient amounts of the dinoflagellate – Most common: barracuda, amberjack, grouper, snapper, sturgeon, king mackerel • GI or neurologic symptoms (or a mixed) – Onset 1-6 hrs after eating, lasts weeks-months – Cold sensation reversal: perceives cold temperatures as hot sensations (and vice versa) – Occurs in 80% of patients and pathognomonic Environmental Injuries: A. Cold-Related Injuries

• Chilblains (or pernio): is an abnormal vascular response to cold resulting in inflammatory skin condition with pruritus and/or painful erythematous to violaceous acral lesions • Frostnip: superficial freeze injury characterized by lack of extracellular ice crystal formation => pale, painful tissue Resolves with rewarming; no tissue loss • Frostbite: ice crystal formation, (+) tissue loss

Treatment: Rapid rewarming in circulating water, 104-108°F (40-42°C) Environmental Injuries: Heat-Related Illness

• Heat exhaustion: nonspecific symptoms Risk factors – Dizziness, weakness, N/V, HA, diaphoresis 1) Exogenous heat gain ° ° 2) Increased heat production – Temp: normal - 104 F (40 C), normal neuro exam 3) Decreased heat dispersion • Heat stroke: T > 105°+ CNS dysfunction a. Dehydration b. CV disease A. Classic: elderly, develops gradually c. Extreme of age d. Obesity ➔ Delirium/seizures (looks like sepsis) e. Improper clothing → Typically anhidrosis f. Skin disease g. Drugs B. Exertional Environmental Injuries: Heat-Related Illness

• Heat exhaustion: nonspecific symptoms Risk factors: – Dizziness, weakness, N/V, HA, diaphoresis 1) Exogenous heat gain – Temp: normal - 104°F (40°C), normal neuro exam 2) Increased heat production 3) Decreased heat dispersion • Heat stroke: (+) CNS dysfunction a. Dehydration b. CV disease A. Classic: elderly, develops gradually c. Extreme of age d. Obesity ➔ Delirium/seizures (looks like sepsis) e. Improper clothing → Typically anhidrosis f. Skin disease g. Drugs B. Exertional: younger, rapid onset, high temp • Will continue to sweat Environmental Injuries: Heat-Related Illness • Heat exhaustion: nonspecific symptoms • Dizziness, weakness, malaise, N/V, HA, diaphoresis • Temp: normal - 104°F (40°C), normal neuro exam • Heat stroke: (+) CNS dysfunction –Classic –Exertional Treatment for heat stroke: “evaporate cooling” or “immersion cooling” Note: antipyretics don’t work Toxicology

4. A 21 yo college student presents to the ED with friends who report the student swallowed “a whole bottle” of acetaminophen 45 minutes before arrival. They also note the patient has been drinking alcohol. The patient is awake but appears intoxicated. Which of the following would be the best course of action:

A. Administer syrup of ipecac B. Perform a gastric lavage C.Administer activated charcoal D.Administer N-acetylcysteine Toxicology: Gastric Decontamination

• Syrup of ipecac – No, No, No!!!!!!! - AAP says do not keep in home (Pediatrics, Nov 2003) • Gastric emptying – 36-40 Fr tube - Possibly helpful if used within 60 min - Risk for iatrogenic injury (aspiration, esophagus) • Charcoal (best option) - If given <30 min, decreases absorption by 70% - If given 30-60 min, decreases absorption by 30% - Dose: 1-2 g/kg (max 100 g) Acetaminophen Toxicity • Max daily dosing: 4 g per day, toxic dose = 150 mg/kg • 2nd most common cause of liver transplantation in U.S., however, only 4% of those with hepatoxicity develop liver failure • 4 clinical phases - Phase 1 (0-24 hrs): asymptomatic, nausea/vomiting - Phase 2 (18-72 hrs): RUQ abd pain, N/V, rising LFTs - Phase 3 (72-96 hrs): Abd pain, N/V, , encephalopathy, renal failure, death - Phase 4 (4 -14 days): resolution • Acetaminophen levels: drawn at 4 hours after ingestion, treatment based on Rumack-Matthew nomogram • Treatment: N-acetylcysteine (NAC) Toxicology Emergencies Agent Antidote •Acetaminophen •N-Acetylcysteine •Aspirin •Alkaline diuresis •B-blocker •Glucagon •Ca-channel blocker •Glucagon •Digitalis •Fab antibodies •Heparin •Protamine Sulfate •Isoniazid (INH) •Pyridoxine (Vit B6) •Opiates •Naloxone (Narcan) •Organophosphates •Atropine •TCA •NaHCO3 Common Toxidromes #1. Anticholinergic • Presentation – Hot as Hades……..Hyperthermia – Blind as a Bat……..Mydriasis – Dry as a Bone…….Thirst, decreased salivation – Red as a Beet……..Flushing, vasodilation – Mad as a Hatter…..Delirium, agitation, confusion

• Etiology: Antihistamines, Antiparkinson, Antipsychotics, Antiemetics (phenothiazines), Antidepressants (TCA), Antispasmotics Common Toxidromes #2. Serotonin Syndrome • Presentation – Cognitive-behavior: agitation, anxiety, drowsy, delirium, headache, seizures – Autonomic dysfunction: tachycardia, arrhythmias, hyperthermia, HTN, diaphoresis, diarrhea, nausea – Neuromuscular: restlessness, tremor, hyperreflexia, dysarthria, ataxia, myoclonic jerks/twitching • Etiology – Most common: SSRIs, MAOs – Especially if combined with: meperidine, cocaine, dextromethorphan, venlafaxine, amphetamine Watch out for: linezolid (Zyvox) Serotonin Syndrome: Presentation

Boyer E and Shannon M. N Engl J Med 2005;352:1112-1120. Image © New England Journal of Medicine. Differences Between Serotonin and Anticholinergic Syndromes

Skin Muscular Tone Reflexes

Serotonin syndrome Diaphoretic Increased Hyperreflexia

Anticholinergic Dry Normal Normal

Neuroleptic malignant Diaphoretic, “Lead pipe” rigid Bradyreflexia syndrome pallor Toxicology

Treatment for the serotonin syndrome is: Cyproheptadine - Has antiserotonergic properties - Only available orally Thank you! Answers 1. A 2. B 3. A 4. C Supplemental slides Lyme Disease: Other types of testing Just say “No”

•Examples of unvalidated tests include: •Capture assays for antigens in urine •Culture, immunofluorescence staining, or cell sorting of cell wall-deficient or cystic forms of B. burgdorferi •Lymphocyte transformation tests •Quantitative CD57 lymphocyte assays •Reverse Western blots •In-house criteria for interpretation of immunoblots •Measurements of antibodies in joint fluid (synovial fluid) •IgM or IgG tests without a previous ELISA/EIA/IFA

MMWR, 2014;63:333 Johnson BJ, Pilgard MA, Russell TM.J Clin Microbiol 2014;52:721-4. MMWR, CDC Surveillance Summary, 2005;54:125. Marques A, Brown MR, Fleisher TA:. Clin Vaccine Immunol 2009, 16:1249-1250. Other Tick-Borne Illnesses • Tularemia • Bartonella • Q fever • Relapsing fever • STARI-Masters’ disease • Colorado tick fever • Tick paralysis • Borrelia mayonii • African tick bite fever • Ehrlichiosis Allergic Reactions: Treatment H2 blockers help decrease urticaria Methods: 91 ED pts. with acute allergic symptoms, randomized to: Results: Diphenhydramine 50 mg IV Diphenhydramine 50 mg IV + ranitidine 50 mg IV + placebo Urticaria at 2 hrs 8% 26%

Need for additional antihistamines 4% 23%

Lin, RY, et al. Ann Emerg Med 36(5): 462, Nov. 2000. Can You Give an ARB to a Patient with ACE- Induced Angioedema?

In theory….YES Data: Very limited… (on pts. switched to ARB) 1.3/39 pts. (7.7%) developed AE (CHARM trial)1 2.2/26 pts. (8%) developed AE2

1 Lancet 2003 2Arch Intern Med 2004 Allergic Reactions: Back to Medical School • Type I (immediate hypersensitivity): antigen attaches to IgE and IgG4 on mast cells and basophils ==> degranulation release mediators (increased vascular permeability, smooth muscle constriction, etc…)

• Type II: IgG and IgM Ab’s react to Ag on cell surfaces eg, blood transfusion rxn, ITP, hemolytic anemias

• Type III - (immune complex): Ag-Ab complex triggers complement system ==> eg, post-strep GN, serum sickness

• Type IV (delayed hypersensitivity): T cell-mediated eg, PPD, poison ivy © 2020 American Academy of Family Physicians. All rights reserved.

All materials/content herein are protected by copyright and are for the sole, personal use of the user.

No part of the materials/content may be copied, duplicated, distributed or retransmitted

in any form or medium without the prior permission of the applicable copyright owner.

Challenging Issues in Hematology

Teresa Holt, M.D. Associate Program Director Grant Family Medicine Residency

Nanette Lacuesta, MD Associate Program Director Riverside Methodist Hospital Family Medicine Residency OhioHealth,Columbus, Ohio Disclosure Statement

It is the policy of the AAFP that all individuals in a position to control content disclose any relationships with commercial interests upon nomination/invitation of participation. Disclosure documents are reviewed for potential conflicts of interest. If conflicts are identified, they are resolved prior to confirmation of participation. Only participants who have no conflict of interest or who agree to an identified resolution process prior to their participation were involved in this CME activity.

All individuals in a position to control content for this session have indicated they have no relevant financial relationships to disclose. Learning Objectives

• Recognize the evaluation and treatment of DVT and PE in medical and pregnant patients. • Understand classifications and work up for anemia. • Identify the diagnosis and clinical hallmarks of sickle cell disease, multiple myeloma, TTP, leukemia and hemophilia. Venous Thromboembolism (VTE)

• 2/3 DVT • 1/3 pulmonary embolus PE

• Of patients with proximal DVT, 40% have associated PE

• 70% of patients with PE also have DVT

• Mortality rate 6% (DVT) and 12 % (PE)

Am Fam Physician. 2012 Nov 15;86(10):913-919. VTE Risk Factors • Virchow’s Triad – Stasis – Hypercoagulability – Endothelial injury • Other risk factors – Family history of VTE (thrombophilia) – Obesity – Prior episodes of VTE – Malignancy – Stroke – Pregnancy/postpartum Pretest Probability for DVT: Wells Criteria Clinical Feature Score Active Cancer: Treatment within last 6 months 1 Palliative treatment Paralysis, paresis or immobilization of lower extremity 1 Bedridden for >3 days or major within 4 weeks 1 Localized tenderness along deep veins 1 Entire leg swollen 1 Calf swelling >3 cm in symptomatic leg 1 Pitting edema greater in symptomatic leg 1 Collateral superficial veins 1 History of DVT/ PE 1 Alternative diagnosis more likely -2 Pretest Probability for DVT: Wells Criteria

Scoring Low Risk Intermediate High Risk Risk 0 1-2 >/= 3 1. A 38-year-old female patient presents with pain in her R calf of 3 days duration. She had a lap cholecystectomy 8 weeks ago and a cervical conization for -in-situ 6 years ago with no recurrence. No one in her family ever had a DVT. She has 2+ bilateral pitting ankle edema. Your choice of an initial diagnostic study would be: A. D-dimer B. Proteins C and S C. Venous compression ultrasounds D. Venogram Diagnosing DVT

• D-dimer (ELISA) – Cut off <500 mcg/L • Consider age x 10 for >50 years old* • Increase in false positive with age – 95% sensitive, 40-60% specific – 95% negative predictive value – Excellent test to rule out, not so good to rule in!

J Am Coll Cardiol. 2017;70(19):2411 *Schouten HJ, et al. Diagnostic accuracy of conventional or age adjusted D-dimer cut-off values in older patients with suspected venous thromboembolism: systematic review and meta-analysis. BMJ2013;346:f2492 SORT B Diagnosing DVT

• D-dimer <500 and low probability Well’s Score makes DVT unlikely • Moderate or high probability Wells score should get noninvasive testing (D-dimer NOT helpful) • Venous compression ultrasounds have 94% positive predictive value • If negative and clinical suspicion is high, repeat US at 5-7 days • Best sensitivity and specificity in symptomatic pts with proximal thrombosis of lower extremities

Am Fam Physician. 2012 Nov 15;86(10):913-919 SORT C If VTE Is Diagnosed, Who Needs…

•Screening for malignancy?

•Screening for thrombophilia? Screening for Malignancy • Cancer risk = 1.3x expected • Complete H&P (with rectal and pelvic exams) • CBC, LFTs, CXR, stool guaiac • Aggressive cancer work-up not necessary or cost- effective – Cancer usually makes its presence known prior to VTE – Lung, pancreas, colon, kidney, prostate Screening for Thrombophilia

• Initial thrombosis prior to age 50 • Family history of VTE • Recurrent venous thrombosis • Unusual vascular beds (Non-extremity site) • Warfarin-induced skin necrosis • Testing – Protein C, protein S, fibrinogen, antithrombin III, factor V Leiden, lupus anticoagulant, anticardiolipin antibody, prothrombin 20210 gene mutation 2. A 63-year-old man presents with a history of several days of shortness of breath with exertion and pleuritic chest pain. He suffered a DVT 5 years ago. His pO2 = 85% on RA and his HR is 110. He recently returned from vacation in Japan. What is the best test for initial evaluation of this patient’s symptoms? A. D-dimer B. Compression US of the lower extremities C. VQ scanning D. CT angiography Diagnosis of PE

• Symptoms and physical findings are very nonspecific • Likewise, EKG and CXR findings not specific • Arterial blood gases/pulse oximetry also lack sensitivity • Normal SpO2 does not rule out PE Modified Well’s Criteria for PE

Clinical Feature Score Symptoms of DVT 3 Other Dx less likely 3 HR > 100 1.5 Immobilization or surgery in past 4 wks 1.5 Previous DVT or PE 1.5 Hemoptysis 1 Malignancy with tx within 6 mos or palliative 1

Low Risk Intermediate Risk High Risk <2 2-6 >6 Diagnosis of PE

• D-dimer (ELISA) – D-dimer <500 combined with low pretest probability (<2) rules out PE Diagnostic Imaging

• Computed Tomography Angiography – High sensitivity and specificity (99% NPV) – Test of choice if • Low pretest probability and positive D-Dimer • Moderate pretest probability • High pretest probability – Preferred over pulmonary angiogram due to • 2% mortality • 5% morbidity

Am Fam Physician. 2012 Nov 15;86(10):913-919 SORT C Diagnostic Imaging: V/Q Scanning VQ Scan Result Pretest Probability Interpretation NORMAL ANY PE excluded LOW probability LOW PE excluded HIGH probability HIGH PE confirmed

– All other combinations require angiography or other imaging – Imaging of choice in patients with contraindications to CT • Contrast allergy • Renal disease • Pregnancy (VQ test of choice if only full dose CT scan available)

3. A 48-year-old female presents with 3 days of left lower extremity swelling. Venous Dopplers reveal a proximal DVT. She is stable and has no other medical problems. What would be the best initial treatment for this patient? A. Rivaroxaban (Xarelto) only as an outpatient B. Low-molecular-weight heparin and concurrent warfarin as an outpatient C. Hospitalization for unfractionated heparin and concurrent warfarin therapy D. Hospitalization for thrombolytic therapy Treatment of VTE 2016 ACCP Guidelines

• DOAC are the preferred anticoagulation choice for proximal DVT or PE when cancer not present. • Dabigatran (Pradaxa), Rivaroxaban (Xarelto), Apixaban (Eliquis), or Edoxaban (Savaysa) • Increased time in the therapeutic window, less major bleeding, patient convenience • Dose adjust DOACs for renal disease • LMWH with concurrent warfarin is still acceptable, just not preferred.

Kearon C, Akl E, et al. Antithrombotic Therapy for VTE Disease: CHEST Guideline and Expert Panel. CHEST 2016; 149(2):315-352 SORT A Treatment of VTE 2016 ACCP Guidelines

• Reversal of DOACs • Antidotes now available for some • Charcoal • Hemodialysis • Cancer-associated thrombosis - LMWH recommended over warfarin and DOACs

Kearon C, Akl E, et al. Antithrombotic Therapy for VTE Disease: CHEST Guideline and Expert Panel. CHEST 2016; 149(2):315-352 SORT A Acute Isolated Distal DVT 2016 ACCP Guidelines

• If no severe symptoms and low risk for extension, then serial imaging x 2 weeks rather than anticoagulation • Risk factors for extension - Strongly positive D-dimer - >5 cm thrombus - Thrombus close to proximal veins - No reversible provoking factors - Active cancer - History of VTE - Inpatient status Kearon C, Akl E, et al. Antithrombotic Therapy for VTE Disease: CHEST Guideline and Expert Panel. CHEST 2016; 149(2):315-352 SORT A Treatment of VTE 2016 ACCP Guidelines

• Outpatient treatment with DOACs is safe and cost effective in selected patients – Ambulatory and stable- no O2, IV ABX, or IV pain control – Low risk of bleeding – No renal insufficiency – Reliable patient and system to monitor complications • Criteria for inpatient treatment – Massive DVT – Symptomatic PE – High risk for bleeding – Comorbid conditions

Kearon C, Akl E, et al. Antithrombotic Therapy for VTE Disease: CHEST Guideline and Expert Panel. CHEST 2016; 149(2):315-352 SORT A Duration of Treatment of VTE 2016 ACCP Guidelines

• The first three months is high-risk period • Treatment duration 3 months for provoked DVT/PE (surgical or non-surgical) 3 or more months if unprovoked DVT/PE Indefinite if recurrent DVT/PE

Kearon C, Akl E, et al. Antithrombotic Therapy for VTE Disease: CHEST Guideline and Expert Panel. CHEST 2016; 149(2):315-352 SORT A Treatment of VTE 2016 ACCP Guidelines

• Early ambulation is suggested instead of prolonged bedrest • Compression stockings are no longer recommended because there is no evidence of benefit

Kearon C, Akl E, et al. Antithrombotic Therapy for VTE Disease: CHEST Guideline and Expert Panel. CHEST 2016; 149(2):315-352 SORT A Treatment of PE 2016 ACCP Guidelines

• Consider systemic thrombolysis if hypotensive (SBP <90) • Start anticoagulation before diagnosis is confirmed • Early discharge and outpatient treatment is an option

– If patient is stable and doesn’t require O2 – Low risk of bleeding – No renal insufficiency – Reliable patient and system to monitor blood work and complications • Consider IVC filter if anticoagulants contraindicated • Remove after treatment window to decrease recurrent DVT Treatment of PE 2016 ACCP Guidelines

• Subsegmental PE - New Recommendation - No proximal pulmonary artery involvement - No proximal DVT in the legs - Low risk for recurrent VTE - Clinical surveillance recommended over anticoagulation

Kearon C, Akl E, et al. Antithrombotic Therapy for VTE Disease: CHEST Guideline and Expert Panel. CHEST 2016; 149(2):315-352 SORT A 4. A 28-year-old pregnant female, G3P2, is diagnosed with a DVT in her right lower extremity. The treatment of choice for this patient would be:

A. Apixaban (Eliquis) B. Unfractionated heparin and warfarin C. Low-molecular-weight heparin alone D. Vena cava filter VTE in Pregnancy

• VTE risk increases 5-fold during pregnancy • Risk 1/1,600 pregnancies • PE is a leading cause of pregnancy-associated mortality and morbidity • 20-50% have an underlying thrombophilia • Women with a history of thromboembolic events have a 3- to 4-fold increase in risk for recurrence during pregnancy VTE in Pregnancy

• Risk of VTE overall, antepartum = postpartum and occurs with equal frequency throughout all trimesters • PE is more common in the postpartum period, especially the first week. • D-dimer less useful in pregnancy (almost always positive) • Lower dose chest CTPA safe in pregnancy – If not available, do CXR – If CXR is normal, do V/Q scan – If CXR is abnormal, do full dose CT scan Am J Respir Crit Care Med. 2011 Nov 15;184(10):1200-8 Anticoagulation in Pregnancy • Heparin and LMWH do not cross placenta and are safe in pregnancy SORT B • Unfractionated Heparin (UH) – Dosage requirements increase due to increases in heparin binding proteins, plasma volume, renal clearance and degradation by the placenta • Low-Molecular-Weight Heparin (LMWH) – Reduced risk for bleeding, osteoporosis and HIT in nonpregnant patients compared to UH – Longer half-life – longer dosage intervals, but increased risk if epidural/spinal – Epidural anesthesia 12 hours after last dose of LMWH SORT C

Obstet Gynecol 2018;132(1): p e1–e17. Anticoagulation in Pregnancy

• Warfarin – not for use in pregnancy – Crosses the placenta – Associated with harmful fetal effects – Use is generally only in the postpartum period or in selected patients with mechanical heart valves • DOACS not adequately studied in pregnancy. • All anticoagulants are safe for breastfeeding mothers • LMWH and coumadin with good data • DOACs – limited data

Obstet Gynecol 2018;132(1): p e1–e17. SORT B Anticoagulation in Pregnancy

• Convert from LMWH to UH for last month of pregnancy. • Compression stockings until anticoagulation resumed postpartum. • Restart anticoagulation after delivery – 4-6 hours post vaginal delivery – 6-12 hours post C-section • May be bridged to warfarin (or potentially DOACs) postpartum • Avoid paradoxical thrombosis and skin necrosis from early anti- protein C effect of warfarin by bridging with LMWH or UFH until INR 2-3 x 2 days

Obstet Gynecol 2018;132(1): p e1–e17. SORT B. Contraception if History of VTE

•Avoid Estrogen

•Use of progesterone is controversial −NIH: ok with first DVT, but not if recurrent −Depo Provera may have the highest risk but not well studied 5. A 48-year-old female with rheumatoid arthritis presents to your office complaining of two months of fatigue, and more recently dyspnea on exertion. Her Hgb=9.2 with normocytic, normochromic indices. You order iron studies, which show low iron, low TIBC and high ferritin. The best treatment for this patient would be:

A. Oral B12 for fatigue B. Oral ferrous sulfate for anemia C. Home oxygen for dyspnea D. DMARDs for rheumatoid arthritis Kinetic Approach to Anemia

• Two reasons for anemia – Decreased RBC production (reticulocyte < 2) • Lack of substrate • Marrow disorders/suppression • Decreased trophic hormones • Chronic illness/inflammation – Increased RBC loss • Hemolysis – inherited or acquired (reticulocyte >2) • Blood loss – occult or obvious Morphologic Approach to Anemia

• RBC indices – reflect cell size – Mean Corpuscular Volume (MCV) – Mean Corpuscular Hemoglobin (MCH) – Mean Corpuscular Hemoglobin Concentration (MCHC) – Red cell Distribution Width (RDW) • Use the RBC indices to direct your work-up of anemia

http://www.uptodate.com/online/content/topic.do?topicKey=red_cell/2950&selectedTitle=1~150&source=search_result SORT C Morphologic Approach to Anemia

• Macrocytic (MCV >100) – Reticulocytosis (high RDW)

– B12 and folate deficiencies • PPIs can cause B12 deficiency

– Myelodysplasia Courtesy: Wikimedia Commons – Alcohol abuse, liver disease, hypothyroidism Morphologic Approach to Anemia

• Microcytic Anemia (MCV <80) – Iron deficiency – Decreased heme synthesis • lead, sideroblastic anemia – Decreased globin synthesis By Dr Graham Beards (Own work) [CC BY-SA 3.0 (http://creativecommons.org/licenses/by-sa/3.0)], via Wikimedia (thalassemia states, hemoglobinopathies) Commons – Chronic illness/inflammation (uncommon) Morphologic Approach to Anemia

• Normocytic, normochromic anemia – Acute blood loss – Acute hemolysis – Hypersplenism – Anemia of chronic disease (most common) Iron Deficiency Anemia • Confirmed by low iron, high iron-binding capacity and low ferritin (<10) • Identify WHY the patient is iron deficient! – Menstrual blood loss – GI malignancy/blood loss • Response to iron therapy • Retic count 5-7 days • Hct 4-6 weeks • Iron stores 4-6 months 6. A 3-year-old patient is brought to see you after moving to your community. He has Hgb SS disease. The parents ask you whether he needs to continue taking the penicillin he was prescribed by another physician. You should recommend:

A. Stop the penicillin to avoid antibiotic resistance B. Take penicillin V 125 mg daily for the rest of his life C. Take penicillin V 250 mg BID until age 5 at least D. Take penicillin V 250 mg daily until age 12 Sickle Cell Disease

• Most common single gene disorder in African-Americans • Diagnosed by hemoglobin electrophoresis • Due to homozygous abnormal hemoglobin S (SS disease) – Sickle SC disease (less severe disease, more retinal disease) – Sickle-Thal (variable) • Hemoglobin S is poorly soluble when deoxygenated and forms polymers that deforms the cells Sickle Cell Disease

• Clinical Manifestations – Moderate anemia (7.9/22.9) with high retic count and normal or high MCV – Vaso-occlusive crises – muscular, CVA, renal, priapism, retinopathy – Infections – pneumococcus, hemophilus, salmonella – Aplastic crisis – parvovirus B19 marrow suppression – Acute chest syndrome – pneumonia + infarct – Splenic sequestration crisis – acute anemia Sickle Cell Disease • Infection Prophylaxis – Immunizations • Strep pneumonia (PCV13 and 23) • Neisseria meningitides (Menactra and Men-B) • H. influenza, Type B • Hepatitis B • Influenza – Penicillin prophylaxis (for all types of SSD) • Pen V p.o. 125 mg BID from age 3 month to 3 years • Pen V p.o. 250 mg BID from age 3 to 5 years • After age 5 is controversial Sickle Cell Disease Annual Screening with Transcranial Doppler Ultrasound to evaluate risk of stroke age 2-16 years (SORT A)

Hydroxyurea therapy age 9-42 months or adults who have 3 or more crises in 12 months to decrease vasocclusive effects and decrease risk of acute chest syndrome (SORT B)

• Disease modifying therapy • Increases levels of HbF • Initiation requires contraceptive counseling, CBC and reticulocyte q4w

Am Fam Phys 2015 Dec 15; 92(12): 1069-1076 Multiple Myeloma

•Incidence: 1.6% of all cancer, 10% of hematologic cancer in U.S. •Risk factors: age >65, African Americans •Signs/ sx −anemia (73%) −bone pain (58%) −elevated creatinine (48%) −hypercalcemia (28%) −N/V, weakness, recurrent infections, weight loss Multiple Myeloma •Dx: abnormal plasma cells in marrow monoclonal protein in serum/ urine bone lesions

•Work Up: CBC w/ diff, Cr, LDH, beta-2 macroglobulin, immunoglobulin studies, skeletal survey, BM biopsy

•Treatment −Primary care: bisphosphonate tx when first diagnosed, DVT prevention, prophylactic ABX −Referral to : chemo, stem cell transplant AM Fam Physician. 2017 Mar 15:95(6): 373-383A. Hemophilia

• Inherited bleeding disorders – Hemophilia A – Factor VIII deficiency – Hemophilia B – Factor IX deficiency (Christmas Disease) • X-linked recessive – predominantly males affected • Become symptomatic within first two years of life – Only 50% have bleeding with circumcision – 3-5% have bleeding in perinatal period Hemophilia

• Bleeding sites – Muscles – Hematuria – Gastrointestinal – Epistaxis and oral cavity – Joints • Late joint destruction • Joints preserved by early initiation of factor concentrate treatments Thrombotic Thrombocytopenic Purpura “Pentad”

• Microangiopathic hemolytic anemia • Thrombocytopenia usually with purpura • Acute renal insufficiency (HUS) • Neurological symptoms (TTP) • Fever • Actually rare for all five to be present Thrombotic Thrombocytopenic Purpura • Causes – Usually idiopathic – ADAMTS13 deficiency • Protease of VWF – inhibitory auto-antibody • Long polymers of VWF attract platelets – Shiga-toxin from E. coli 0157:H7 – Collagen vascular diseases, cancer and transplants – Medications • Ticlopidine, clopidogrel, quinine, mitomycin, tacrolimus • Treatment – plasma exchange therapy curative Leukemias

• Acute Leukemias – Acute Lymphoblastic – Children • Fever, bleeding, musculoskeletal symptoms, • (75%), lymphadenopathy (60%) – Acute Myelogenous – Adults • Fever, fatigue, weight-loss, bleeding problems • Anemia, thrombocytopenia Leukemias

• Chronic Leukemias – Adults • Most asymptomatic at time of diagnosis • Discovered with leukocytosis on blood count • Work Up: repeat CBC, smear, flow cytometry, onc referral

• Chronic lymphocytic leukemia – 50% • Hepatosplenomegaly and lymphadenopathy • Chronic myelogenous leukemia – 20% • Splenomegaly Board Frequent Fliers •D Dimer only helpful for low pretest probability −If not CTPA for PE and doppler for DVT •DOAC preferred over coumadin (LMWH if cancer) −Dose adjust DOACs with renal disease •No coumadin in pregnancy (DOACs with insufficient data) •Anemia of Chronic DZ: treat underlying DZ •Macrocytic anemia – from PPIs due to B12 deficiency •Sickle Cell: transcranial U/S screening •Multiple Myeloma: start bisphosphonate tx •TTP pentad: fever, anemia, thrombocytopenia, AKI, neuro sx •Acute Leukemia: fever, bleeding •Chronic Leukemia: asymptomatic, elevated WBC- work up: repeat WBC, smear, flow cytometry, onc referral Answers

1. A 2. D 3. A 4. C 5. D 6. C © 2020 American Academy of Family Physicians. All rights reserved.

All materials/content herein are protected by copyright and are for the sole, personal use of the user.

No part of the materials/content may be copied, duplicated, distributed or retransmitted

in any form or medium without the prior permission of the applicable copyright owner.

Common Problems in Neurology

Teresa Holt, M.D. Associate Program Director Grant Family Medicine Residency

Nanette Lacuesta, MD Associate Program Director Riverside Methodist Hospital Family Medicine Residency OhioHealth, Columbus, OH Disclosure Statement

It is the policy of the AAFP that all individuals in a position to control content disclose any relationships with commercial interests upon nomination/invitation of participation. Disclosure documents are reviewed for potential conflicts of interest. If conflicts are identified, they are resolved prior to confirmation of participation. Only participants who have no conflict of interest or who agree to an identified resolution process prior to their participation were involved in this CME activity.

All individuals in a position to control content for this session have indicated they have no relevant financial relationships to disclose. Learning Objectives

• Seizure Disorders – Describe the classification, diagnostic tests used in the evaluation, use of and common side effects of antiepileptic medications, management in the pregnant patient.

• Diagnosis and treatment of vascular headaches and chronic tension headaches and recognize the indications for neuroimaging in the evaluation of headaches.

• List the clinical presentations and management options for multiple sclerosis. 1. A 65-year-old male patient of yours presents to the ED having had a seizure in his bedroom witnessed by his wife. She heard a cry and the fall, saw him stiffen and shake all over, and then become incontinent of urine. He was not arousable until he had been in the ED for several minutes. His seizure would be classified as:

A. Complex Partial B. Generalized Tonic-Clonic C. Grand Mal D. Myoclonic Definition

•A seizure is a transient occurrence of signs or symptoms due to abnormal excessive or synchronous neuronal activity in the brain, and can be either focal (partial) or generalized Classification of Seizures

• Focal seizures (formerly partial) – Local – Ipsilateral propagation – Contralateral propagation – Secondarily generalized – Consciousness may or may not be impaired (formerly complex vs simple) Classification of Seizures

• Generalized seizures – Nonconvulsive (absence) • Typical (3/sec spike and slow waves on EEG) • Atypical (<3/sec spike and slow waves on EEG) – Convulsive • Myoclonic (very short muscle contractions) • Clonic (repeated jerking) • Tonic (stiffening) • Tonic-clonic (stiffening followed by jerking) – Grand Mal term is now outdated and no longer used • Atonic (“drop attacks” – loss of muscle tone) 2. The most likely cause of a new seizure in a patient >65 years old would be:

A. Cerebrovascular disease/stroke B. Idiopathic C. Metabolic derangement D. Brain tumor Etiology of Seizures

• Idiopathic – 62% overall ages • Stroke – 15% overall, 49% >age 60 • Brain tumor – 6% overall, 11% >age 60 • Head trauma • Intracranial infection • Cerebral degeneration • Congenital brain malformations • Inborn errors of metabolism “Provoked” Seizures

• Seizures that occur within a medical setting, that, if removed, presumably the seizures would not occur • Metabolic derangements – Hypo- and hyperglycemia – Hyponatremia – Hypocalcemia (usually neonates) – Renal failure and uremia “Provoked” Seizures

• More rare metabolic causes – Hyperthyroidism – Acute intermittent porphyria • Cerebral anoxia – Arrest, anesthesia, drowning, CO – Syncope with brief hypoventilation (adolescents) • Drug toxicities/withdrawal – Alcohol – Benzodiazepines Epilepsy

•Epilepsy is a chronic condition characterized by at least two unprovoked seizures at least 24 hours apart. 3. A patient with a new onset seizure has a complete workup that is unremarkable for any provoked causes, signs of infection, drug toxicities, or neurological disease. The next step in the workup for this patient would be:

A. Event monitor B. Prolactin level C. Neuroimaging (CT/MRI) D. Lumbar puncture Diagnostic Evaluation

• Must start with a complete H&P – Witnessed description of the event – Substance abuse – Head trauma – Cerebrovascular events – Cardiac history – Sleep disorders – Medication history • Diphenhydramine, tramadol, buproprion Diagnostic Evaluation

• Blood studies – CBC – Electrolytes – Calcium, magnesium, phosphorus – Glucose – BUN/creatinine – ?ESR, LFTs, RPR – Prolactin has limited utility – low sensitivity Diagnostic Evaluation

• Lumbar puncture – only if signs of infection • EEG – essential to diagnosis and classification • Neuroimaging – MRI preferred over CT – Better for identifying structural lesions – Is the lesion the cause of the seizures? 4. A patient presents to you having had a single new unprovoked seizure with negative workup. Your first step in treatment would be:

A. Start no medications at this time B. Start oral valproic acid C. Start oral phenytoin D. Start IV phenytoin Starting Antiepileptic Drugs (AEDs) • Not necessary in individuals with a single new onset seizure or infrequent seizures • Immediate AED therapy compared to delay of treatment pending a 2nd seizure is likely to reduce recurrence risk within the first 2 years (SORT B) but not improve quality of life (SORT C) • Immediate AED unlikely to improve prognosis over a longer term (>3 years) (SORT B) • Treat only if there is a high risk of recurrence st nd Neurology. 2015; 84(16): 1705 • Most will respond to 1 or 2 monotherapy AED JAMA Neurol. 2018 75(3): 279-286 High Risk of Seizure Recurrence

•Status epilepticus •Hx of brain injury •Brain lesion on neuroimaging •Focal neurological abnormalities •Intellectual disability •Abnormal EEG with epileptiform discharges •High-risk seizure types −Focal, absence, myoclonic or atonic seizure Antiepileptic Drugs • Start with AED monotherapy • Selection is individualized based on – Relative efficacy – Other medications (enzyme induction) – Comorbid conditions – Potential adverse effects, tolerability, serious toxicity – Patient preferences and cost • Do not consider combination therapy until patient has failed two monotherapy trials • Monitor patients closely for effectiveness, side effects, and compliance

Epilepsia. 2006 July; 47 (7): 1094-120 SORT A Neurol Clin . 2010 Nov; 28 (4): 843-52 Classification of AEDs • Broad Spectrum • All seizure types • Narrow Spectrum • Focal or secondarily generalized seizures • First Generation • Less expensive • More side effects and drug interactions • Require monitoring of drug levels • Second Generation • More expensive • Fewer side effects and interactions • Monitoring often unavailable Toxicities of AEDs

• Common toxicities with AEDs – Suicidality – twice the risk with AEDs compared to placebos – Neurotoxicities – ataxia, dizziness, somnolence, fatigue, headache – Rash – wide spectrum from simple maculopapular rashes to Stevens-Johnson Syndrome – Liver enzyme – induction or inhibition 5. A 46-year-old male patient of your practice presents with fever, arthralgias, and erythroderma. He has no respiratory symptoms. His medications include lisinopril, valproic acid, and atenolol. His fever is 102.2° and his skin reveals a very erythematous maculopapular rash and some tenderness. There are no oral lesions. You order bloodwork that reveals a WBC = 16,000 with 17% eosinophils, and AST and ALT about 2x normal. The most likely diagnosis is:

A. Toxic shock syndrome B. DRESS syndrome C. Stevens-Johnson syndrome D. Neurocutaneous phakomatosis syndrome DRESS Syndrome • Drug Reaction with Eosinophilia and Systemic Symptoms • Maculopapular rash, fever, arthralgias and lymphadenopathy • Persists weeks to months with 10-25% mortality • Common offenders – Carbamazepine, phenytoin, lamotrigine, valproate – Allopurinol and sulfonamides

Am J Med. 2011 Jul; 124(7): 588-97 Status Epilepticus

A single unremitting seizure lasting longer than 5 minutes or recurrent seizures without interictal return to baseline • Convulsive: rhythmic jerking of extremities (Generalized Convulsive SE) • Non-convulsive: seizure activity on EEG without clinical findings of GCSE • Refractory: do not respond to standard treatment regimens for SE Neurocritical Care. Aug 2012; 17 (1): 3-33. Non-Convulsive Status Epilepticus

• “Wandering confused” • Acutely ill patient with severely impaired mental status +/- subtle motor movements (rhythmic eye twitches or tonic eye deviation) “subtle status” Frequently follows uncontrolled GCSE Often in ICU setting Neurocritical Care. Aug 2012; 17 (1): 3-33. Mortality Rates

Status Epilepticus Type Mortality Rate at Hospital Discharge Convulsive 9-21% Non-Convulsive 18-52% Refractory 23-61%

Neurocritical Care. Aug 2012; 17 (1): 3-33. Status Epilepticus Initial Therapy • Evaluate causes while you treat • Drug of choice is lorazepam 0.1 mg/kg IV up to 4 mg/dose • Drug of choice via other routes: Midazolam IM, Diazepam rectally • Urgent Control Therapy with AED in all SE patients unless immediate cause is known and corrected (e.g., hypoglycemia) with phenytoin/fosphenytoin, phenobarbital, valproate sodium Neurocritical Care. Aug 2012; 17 (1): 3-33. SORT A Seizures and Driving • 0.01-0.1% of MVAs are attributable to seizures • Fatal MVAs 8x greater from alcohol than seizures • Seizure-free interval is best indicator of driving safety – The longer the seizure-free interval, the less likely an MVA – The longer the patient is restricted from driving, the lower the compliance • 1994 consensus statement of the American Academy of Neurology, American Epilepsy Society, and the Epilepsy Foundation of America recommends a 3-month seizure-free interval before driving resumed. Consensus statements, sample statutory provisions, and model regulations regarding driver licensing and epilepsy. American Academy of Neurology, American Epilepsy Society, and Epilepsy Foundation of America. Epilepsia 1994; 35:696 Seizures and Driving

• States authorities make the ultimate decision about driving, not physicians. • State laws vary regarding seizure-free intervals. • Only a few states require physicians to report patients with seizures. • KNOW YOUR OWN STATE’S LAWS!

www.epilepsy.com/driving-laws/2008721 Epilepsy and Contraception

• Hormonal contraception failure on AEDs that stimulate cytochrome P450 system: – Carbamazepine – Phenytoin – Phenobarbital/primidone – Topiramate – Oxcarbazepine Epilepsy and Contraception

• AEDs also interfere with progesterone-only pills, contraceptive rings, and progesterone implants. • AEDs do NOT interfere with Depo Provera or IUDs (hormonal or copper) • Combined OCs can decrease lamotrigine blood levels by as much as 50% Epilepsy and Pregnancy

• Consider withdrawing AEDs 6 months prior to conception if patient has been seizure-free >2 years • Do not change AEDs in pregnancy if patient well-controlled • Supplement folic acid 4 mg per day if patient on carbamazepine or valproic acid Epilepsy and Pregnancy

• Monitor both total and free drug levels of AEDs at 6 weeks, 10 weeks, then once a trimester, then first or second postpartum week. • Vitamin K supplementation 10-20 mg/day in last month of pregnancy if on phenobarbital, carbamazepine, phenytoin, topiramate, oxcarbazepine. • May breastfeed on AEDs except lamotrigine 6. A 46-year-old male smoker comes to you for a complaint of unilateral, cyclical, nocturnal headaches that last 30 minutes each. He has taken OTC meds but they have not been helpful. The best prophylactic medication for his headaches would be: A. Oxygen B. Sumatriptan C. Propranolol D. Verapamil Primary Headaches

• Tension-type • Trigeminal Autonomic Cephalgias – Primary, unilateral headache syndromes in the trigeminal nerve distribution – Associated with ipsilateral autonomic symptoms – Cluster headaches are the most common • Migraines • Others: cough, exercise, sexual activity, others Chronic Tension Headaches •Bilateral, band like •Tx: OTC analgesics −if >2x/week, increased risk of chronic daily HA −TCAs for prophylaxis (SOR B) −Butalbital and opioids should be avoided- increase risk of chronic daily HA

•Chronic Daily Headaches: 15 or more HA per month x 3 months −Can be due to chronic tension headaches or chronic migraines −Treatment: stop all abortive treatments, begin prophylactic treatments: −Non-pharmacologic: biofeedback/ relaxation, cognitive behavioral therapy (SOR B) −Pharmacologic: SOR A: topiramate, valproate SOR B: amytritriptyline, gabapentin, tizanidine no effect: SSRIs, propranolol, botox Am Fam Physician 2014:89(8): 642-648. Cluster Headaches • Prevalence <1% • Males > females • Orbital or temporal unilateral pain • Associated ptosis, miosis, rhinorrhea, lacrimation or conjunctival injection • Last 15 minutes to 3 hours • Acute treatment – oxygen and sumatriptan SQ • Preventive treatment – Verapamil – drug of choice (240-320 mg per day or more) – Steroids, lithium, topiramate Migraine Headache • Affects 18-26% women, 6-9% men in US • Pathophysiology – unclear – Cellular and blood-brain permeability changes→ inflammation of meninges and vasodilation – Genetic factors play a major role. • Recurrent attacks in four phases – Prodrome: euphoria, depression, irritability, yawning – Aura: Visual – flickering lights, spots or lines Sensory – pins and needles, numbness Dysphasic speech – Headache – Postdrome – sudden head movement causes head pain ICHD-3 Criteria: Migraine without Aura A. At least five attacks fulfilling criteria B–D B. Lasting 4-72 hours C. Has at least two characteristics 1. Unilateral location 2. Pulsating quality 3. Moderate or severe pain intensity 4. *Aggravation by or causing avoidance of routine physical activity (e.g., walking or climbing stairs) D. At least one of the following 1. Nausea and/or vomiting 2. *Photophobia and phonophobia E. Not better accounted for by another ICHD-3 diagnosis International Classification of Headache Disorders ICHD-3 Criteria: Migraine WITH Aura

A. At least two attacks fulfilling criteria B and C B. One or more fully reversible aura symptoms C. At least three of the following: 1. At least one aura symptom spreads gradually over ≥5 minutes 2. Two or more aura symptoms occur in succession 3. Each individual aura symptom lasts 5-60 minutes 4. At least one aura symptom is unilateral 5. At least one aura symptom is positive 6. The aura is accompanied, or followed within 60 minutes, by headache D. Not better accounted for by another ICHD-3 diagnosis. Diagnosis of Migraine Headache

• Clinical diagnosis – primarily history • Negative neurological examination • Neuroimaging unnecessary, except with – Unexplained abnormal neuro findings – Headaches that do not fit the strict definition – Choose Wisely Campaign (Radiology) • No benefit for EEG – Choose Wisely Campaign (Neurology) “Red Flags” for Neuroimaging • Rapidly increasing headache frequency • Lack of coordination, abnl neuro exam, papilledema • Headaches that awaken from sleep • Worse headache of life • Patients with cancer, HIV, Lyme disease • New onset over age 50 • Rapid onset with strenuous exercise • CT or MRI w/o contrast – test of choice (need contrast if immunocompromised or concern for temporal arteritis, dissection, or aneurysm) Am Fam Physician 2013 May 15;87(10): 682-687 Acute Treatment of Migraine: First Line • Acetaminophen, NSAIDS • Triptans – serotonin 1b/1d agonists – All triptans have been found to be effective and well-tolerated – Avoid triptans (due to increased BP risk) in • Familial hemiplegic migraine • Basilar migraine • Ischemic stroke • Ischemic heart disease, Prinzmetal’s angina, uncontrolled hypertension • Pregnancy (relative contraindication) • Acetaminophen/aspirin/caffeine • Sumatriptan/naproxen Am Fam Physician. 2018 Feb 15; 97 (4): 243-251 Acute Treatment of Migraine: Second Line

• Intranasal Dihydroergotamine (DHE) • Side effects: leg cramps, HTN, tinging in extremities • Contraindicated in pregnant • Antiemetics: dopamine antagonists • Prochlorperazine, Metoclopramide, promethazine • Risk extrapyramidal adverse effects • Opioids: tramadol, codeine, meperidine, butorphanol • Moderate effectiveness but high abuse potential • Routine use not recommended

Am Fam Physician. 2018 Feb 15; 97 (4): 243-251 Refractory Migraine • Parenteral dihydroergotamine (DHE45) − Avoid for 24 hours after triptan • Magnesium sulfate IV: alternate treatment migraine with aura • Dexamethasone IV: consider to decrease recurrence- prednisone taper

• Valproate IV: contradictory evidence • Diphenhydramine: not recommended • Opioids: use sparingly and infrequently Am Fam Physician. 2018 Feb 15; 97 (4): 243-251 Preventive Treatment of Migraine

• Indications – Headaches significantly interfere with usual activities – Overuse of acute therapies – Adverse events with acute therapies – Patient preference – Certain migraine variants that mimic CVAs • Hemiplegic migraines • Basilar migraines • Prolonged aura Preventive Treatment of Migraine

• First line agents (if no contraindications): SORT A • Valproic acid • Frovatriptan • Propranolol, timolol, metoprolol • Topiramate • If not effective after 2-3 months, adjust dose. • If max dose or adverse effects, change med • If no single first line agent effective/tolerable, consider combination

• Second line agents: amitriptyline, atenolol, nadolol, naratriptan, zolmitriptan, venlafaxine, supplements (butterbur, riboflavin, feverfew, magnesium

Am Fam Physician 2019 Jan 1;99(1)17-24 Estrogen-Containing Contraceptives and Migraines • Increased risk of CVA if • Migraines with aura at any age • Migraines over age 35

(Lack of good quality data but was still on 2018 ITE) Multiple Sclerosis • Auto-immune inflammatory demyelinating disease • Affects adults 20-40 years old, 2:1 female to male ratio • Most common neurologic disability in young adults • Prevalence 1 per 800 persons; incidence 2-10 per 100,000 • Must diagnose discrete lesions in various parts of the CNS at least 3 months apart • Pathogenesis unclear – Immune system stimuli – Viral infections – Geographic and environmental factors – Genetic factors Am Fam Physician. 2013 May 15; 87 (10): 712-714 Multiple Sclerosis

• Clinical Manifestations – Insidious or sudden onset – Optic neuritis – pain with eye movement and visual loss – Paresthesias, weakness, loss of coordination – Bladder urgency/retention – , sexual dysfunction – Fatigue and depression – Lhermitte’s sign – neck flexion results in an electric shock sensation down limbs and spine – Uhthoff’s phenomenon – worsening of symptoms with increased body temperature – No single specific diagnostic test Multiple Sclerosis

• Acute exacerbations treated with corticosteroids – Methylprednisolone 500-1000 mg daily x 3-7 days – Short prednisone taper Multiple Sclerosis • No treatment has been shown to affect long-term outcome • Disease-modifying agents should be started EARLY • Decreased relapse rate, decreased brain lesions on MRI • SE Immunomodulators Interferons – flu-like syndrome, leukopenia, depression/suicidality Glatiramer – injection site reactions, flushing, chest tightness/palpitations • SE Immunosuppressives Dimethyl fumarate – progressive multifocal leukoencephalopathy Fingolimod – zoster infections Teriflunomide – hepatoxicity Mitoxantrone – cardiac toxicity

Am Fam Physician. 2013 May 15; 87 (10): 712-714 Board Frequent Fliers

•SZ cause – most common idiopathic overall, if >60 CVA •SZ meds – lots of side effects, not needed after first SZ with neg eval, in preg- stop 6 mo prior to conception if no SZ >2yrs •Chronic tension HA – TCAs for prevention •Chronic daily HA – stop abortive, start topiramate or valproate, consider biofeedback, CBT •Migraine prevention – TCAs no longer first line •HA neuroimaging – for red flags only- usually CT or MRI w/o contrast •MS- start immunomodulators early, meds have lots of side effects Answers

1. B 2. A 3. C 4. A 5. B 6. D Antiepileptic Drugs

First Generation Second Generation More complicated Fewer serious adverse effects pharmacokinetics Wider therapeutic Ranges Narrower therapeutic ranges Lower Risk teratogenesis More expensive Examples: Exampes: Carbamazepine Levetiracitam Ethosuxamide Lamotrigine Phenobarbital Oxcarbazepine Phenytoin Gabapentin Primidone Pregabalin Valproic Acid Topiramate Antiepileptic Drugs: First-Line Monotherapy

Seizure type < 16 years old 16-59 years old > 60 years Comments old Focal Oxcarbazepine Carbamazepine Gabapentin SORT A Levetiracetam Lamotrigine Phenytoin Zonisamide Generalized: Ethosuxamide No SORT A No SORT A Choose a Broad Absence Valproic Acid spectrum 2nd generation Generalized: No SORT A No SORT A No SORT A Recommendations Others are SORT C/D

Am Fam Phys 2017 July 15; 96 (2): 87-96 © 2020 American Academy of Family Physicians. All rights reserved.

All materials/content herein are protected by copyright and are for the sole, personal use of the user.

No part of the materials/content may be copied, duplicated, distributed or retransmitted

in any form or medium without the prior permission of the applicable copyright owner.

Peripheral Vascular Disease

Robert Dachs, MD, FAAFP Clinical Associate Professor Ellis Hospital Family Medicine Residency Program Albany Medical College, Albany, New York Disclosure Statement

It is the policy of the AAFP that all individuals in a position to control content disclose any relationships with commercial interests upon nomination/invitation of participation. Disclosure documents are reviewed for potential conflicts of interest. If conflicts are identified, they are resolved prior to confirmation of participation. Only participants who have no conflict of interest or who agree to an identified resolution process prior to their participation were involved in this CME activity.

All individuals in a position to control content for this session have indicated they have no relevant financial relationships to disclose. Learning Objectives

1. Recognize the of abdominal aortic aneurysms and aortic dissection. 2. Define claudication. 3. Describe the physical findings in chronic arterial insufficiency. 4. List some of the means for objective documentation of occlusive disease. 5. Recognize some of the supportive measures for patients with claudication. 6. Describe the benefits of walking programs for patients with claudication. 7. List the medications available to improve walking distance in patients with peripheral vascular disease. Aortic Diseases

• Aortic dissection (thoracic) • Abdominal aortic aneurysm (AAA) Abdominal Aortic Aneurysm (AAA)

• Aneurysm is >50% in vessel diameter • Normal aorta diameter is 1.8-2.0 cm • 2-5% prevalence in elderly populations • Average age of Dx: 65 • 13th most common cause of death in U.S. • 10th most common cause of death in men Abdominal Aortic Aneurysm (AAA)

• Location: 95% are infrarenal #1 risk factor • Pathogenesis: Atherosclerosis Cigarette Smoke (Thoracic and suprarenal aneurysms do occur- …think Marfan’s, Ehlers-Danlos, syphilis) • Male to female ratio: 3-8:1 Abdominal Aortic Aneurysm (AAA): Mortality

• Rupture: 60% of patients die before arrival • Only 50% that do arrive alive, survive

• Overall mortality rate (with rupture) = 80% 1. Because of the prevalence and its lethality, the USPSTF recommends that ultrasound screening be performed in which patients?

A. One-time screening for men ages 65-75, who have ever smoked B. One-time screening for men ages 65-75, regardless of smoking history C. One-time screening for both men and women ages 65-75, who have ever smoked D. One-time screening for both men and women ages 65-75, regardless of smoking history Should You Ultrasound “Screen” Patients for AAA? USPSTF Recommendations: 2014

• (+) Screen 1 time for abdominal aortic aneurysm (AAA) with ultrasonography in men aged 65-75 years who have ever smoked (grade B) • Selectively offer screening for AAA in men aged 65-75 years who have never smoked (grade C). • There is too little evidence to recommend for or against screening for AAA in women who have ever smoked (grade I). • (-) Do not screen for AAA in women who have never smoked (grade D) Should You US “Screen” Patients for AAA? Why not women???

Chichester Trial Methods 9342 women, ages 65-80 randomly

Assigned to Screening vs. control (no screening)

Results • At 5 years, no difference in AAA mortality • At 10 years, no difference in AAA rupture rate

Scott, RA; et al. Br J Surg; 1995 Medicare Part B Covers One-time AAA Screening for:

• Men aged 65-75 years who have ever smoked • (+) Family Hx AAA (1st degree relative) 2. You identified an abdominal aortic aneurysm (AAA) in your patient.

At what size (in centimeters) should you refer your patient for surgical intervention?

A. 3-3.5 cm B. 4-4.5 cm C. 5-5.5 cm D. 6-6.5 cm AAA: Risk of Rupture

Used with permission © New England Journal of Medicine. Powell JT and Greenhalgh RM. NEJM 348: 1895, May 8, 2003 AAA: Risk of Rupture

• Rate of increase = Is exponential!!!! • Smaller aneurysms expand slower (Conversely, larger aneurysms expand faster)

Aneurysm size Mean yearly increase 3.0-3.9 cm 0.20 cm 4.0-4.0 cm 0.34 cm 5.0-5.9 cm 0.64 cm

Vardulaki, KA et al. Br J Surg 1998; 85: 1674 You Have Identified a Small AAA. How Often Should You Get a F/U Ultrasound?

Surveillance Diameter, cm recommendation Society of Vascular Surgery 3.0-3.9 36 months 2018 4.0-4.9 12 months 5.0-5.4 6 months

Chaikof EL, et al. J Vasc Surg 2018; 67:2-77. Aortic Dissection: Who’s at Risk? • Ages: 40-80, average age: 63 • Males: 2:1 • Hypertension: 76% - Prior cardiac surgery 16% - *Marfan’s 5% (more common in younger patients) - Cocaine use: 2% - Peri/post-partum Evangelista A, et al. Circulation 2018: 1846-1860 Marfan’s Syndrome: Clinical Features • Above-average height • Disproportionally long, slender limbs • Scoliosis • Thoracic lordosis • Pectus carinatum or Pectus excavatum • Abnormal joint flexibility -----Thumb sign (Steinberg’s sign) • High-arched palate • Flat feet • Lens dislocation

• Spontaneous pneumothorax Images courtesy of wikipedia A Pet Peeve

• AAA do not “dissect” • Aortic dissections are NOT aneurysms

The pathophysiology is different!!!! AAA Aortic dissection Atherosclerosis Hypertension Aortic Dissection: Location

Stanford Classification • Ascending Aorta (60-65%) Type A • Descending Aorta (30-35%) Type B (After origin of subclavian artery) IRAD Study: Circulation 2018, 7,300 patients Type A Type B Any Pain 94% 98% Anterior CP 79% 63% Back pain 43% 64% 22% 42% Tearing/ripping 49% 52% Migrating 15% 19% Syncope 19% 4% Hypertensive 36% 70% Hypotensive/shock 25% 4% Pulse deficits 30% 20% Aortic Dissection: Diagnosis

• Echocardiography 1. Transthoracic (TTE) 2. Transesophageal (TEE) • CT-Angiography • MRI • Aortography

Image reprinted with permission from Joel E Fishman, MD, PhD, University of Miami School of Medicine, published by Medscape Reference (http://emedicine.medscape.com/), 2014, All are acceptable, depends available at: on what you have available! http://emedicine.medscape.com/article/416776-overview Aortic Dissection: Management

2 Goals • Lower blood pressure to BP sys 90-110 IV nitroprusside • Lower velocity of LV ejection IV esmolol

Pearl: Start with your B-Blocker Arterial Occlusive Disease

•Chronic due to: Atherosclerosis => Claudication

•Acute due to: Thromboembolic => 5 “P’s” Claudication: Definition

-Reproducible ischemic muscle pain - That occurs with exercise, - Relieved with rest

It’s stable angina … of the legs!!! Claudication: Presentation

• Symptoms are distal to the location of occlusion Calf symptoms ------→ femoral- popliteal disease Calf and thigh ------→ profunda femoral artery Thigh, hip, buttock pain, with impotence -→ aortoiliac disease (Leriche Syndrome) Chronic Arterial Occlusive Disease Differential Diagnosis • Spinal stenosis – “Pseudoclaudication” • Spinal cord tumors • Lumbar radiculopathy • DJD • DVT 3. A 68 yo male presents with complaints of an aching pain in both thighs when he walks about one block. The pain subsides within about 1-2 minutes after he stops ambulating. The best initial test in this case is: A.Perform an ankle-brachial index (ABI) B.Perform bilateral leg ultrasound C.Perform pulse volume recordings (PVR) D.Perform lower extremity magnetic resonance arteriogram (MRA) Used with permission © New England Journal of Medicine White C. N Engl J Med 2007;356:1241-1250 The New England Journal of Medicine Fig 2, Peripheral Vascular Disease Hennion D, Am Fam Phys, Sept 1, 2013 p.306-310 Chronic Arterial Occlusive Disease Screening: Ankle-Brachial Pressure Index ABI Interpretation 0.9 - 1.40 Normal 0.7 - 0.89 Mild 0.4 - 0.69 Moderate <0.4 Severe • Use higher of 2 brachial pressures if different • Use higher of 2 ankle pressures (DP or PT) if different • CPT # 93922 Screening: Ankle-Brachial Pressure Index What if the value is markedly elevated?? Example: ankle BPsys = 210 = 1.6 brachial BPsys = 130 • An ABI >1.4 indicates noncompressible arteries (calcified vessels) • 1.4% of adults >40 yo have ABI >1.4 - Accounts for approx 20% of all adults with PVD. Bottom Line: “Normal” ABI = 0.9-1.4 4. When obtaining an ABI, AHA/ACC recommends obtaining the brachial (arm) systolic BP in both arms and using the higher of the 2 readings. However, if a difference in systolic BP >20 mm between the arms, the most likely diagnosis is?

A.Acute aortic dissection B.Chronic aortic aneurysm C.Subclavian artery stenosis D.A mediastinal mass/tumor Subclavian Artery Stenosis •Most often due to atherosclerosis •Usually asymptomatic – requires no intervention. •Only intervene if these develop… −Subclavian steal syndrome −Upper extremity claudication −Coronary steal syndrome Peripheral Artery Disease: Management / \ Risk factor modification Intervention • Smoking cessation • Exercise • Hypertension • Cilostazol (Pletal) • Diabetes mellitus • Lipid lowering agents • Antiplatelet therapy • Ramipril - Aspirin: 75-325mg qd or - Clopidogrel: 75mg qd Note: not dual antiplatelet therapy (DAPT) PAD: Interventions 1. Supervised exercise training 32 trials: => improve pain-free walking* • Improved pain-free walking by 90 yards • Does not improve ABI 2. Lipid-lowering agents –7 small trials: => improve pain-free walking** • Improved pain-free walking by 96 yards

*Lane R, et al. Cochrane Library, Issue 12, 2017 **Aung PP, et al. Cochrane Library, Issue 4, 2007 PAD: Interventions 3. Drug therapy for claudication •Cilostazol (Pletal) ‾ Inhibits phosphodiesterase type 3 ‾ Mechanism of action is unclear ‾ 15 double-blind, randomized, placebo-controlled trials => • Improved pain-free walking* • Approx by 41 yards**

*Bedenis, et al. Cochrane Library, Issue 10, 2014 ** Beebe HG, et al. Arch Intern Med 1999; 159: 2041-2050 PAD: Interventions 4. Drug therapy for claudication – A new player ‾ Ramipril 10 mg qd ‾ Methods: double-blind randomized trial ‾ Results: At 6 months, 75-second increase in pain-free walking time, and 255-second increase in maximum walking time

Ahimastos, AA, et al. JAMA, Feb 6, 2013 PAD: Management

Risk factor Mild/moderate Critical leg modification symptoms ischemia • Exercise | • Drug therapy | Symptoms/ \Symptoms | improve worsen | Localize the lesion | | Imaging only when •Pulse volume recording | considering Localize lesion revascularization •Magnetic resonance angiography (MRA)| •Conventional angiography Chronic Arterial Occlusive Disease: As Disease Progresses • Pain at rest in foot or toes (sometimes noted as paresthesias/numbness) • Worse with legs elevated, relieved with legs dependent • Develops leg edema Chronic Arterial Occlusive Disease: As Disease Progresses • Hair loss, smooth shiny skin • Thickened nails • Pallor with leg elevation and • Rubor with legs dependent • Bruits, decreased pulses • Cyanosis, ulceration, gangrene Singer AJ, et al. NEJM 2017; 377: 1559-1567 So, should you screen all at-risk, asymptomatic patients with an ABI?

2013, 2018: USPSTF notes Insufficient Evidence for routine screening for PAD in asymptomatic adults Rating: I Level Recommendation 5. A 72 yo female presents to the ED with sudden severe R leg pain, located from the knee to toes. Her past medical history is significant for HTN and DM. Exam: Vitals: 160/90, pulse = 120 and irregular, afebrile. Lungs are clear. Heart: rapid irregularly, irregular pulse. The right leg is cool to touch, pale in color, and you are unable to obtain posterior tibial or dorsalis pedis pulse. At this point, you should: A. Start heparin and immediately consult a vascular surgeon B. Immediately obtain an ultrasound of the lower extremity C. Immediately obtain an echocardiogram D. Immediately obtain an abdominal aortic ultrasound Acute Arterial Occlusion • Thromboembolic • Heart is most common source: 80-90% • Presentation: The 5 “P’s” ‾ “P”ain ‾ “P”allor ‾ “P”aresthesia ‾ “P”ulselessness ‾ “P”aralysis Acute Arterial Occlusion

• Cardioarterial emboli: Where do they lodge? ‾ Lower extremities: 65-70% (usually at bifurcations) ‾ Cerebral arteries: 20-25% ‾ Upper extremities: 5-10% ‾ Visceral arteries: 5-10%

Don’t miss: Acute SMA occlusion “Pain out of proportion to physical findings” 6. A 76 yo male with a Hx of HTN, hyperlipidemia, and smoking presents with a painful toe. He denies trauma. No Hx of atrial fibrillation. The toes are dusky blue in color. He has 2+ posterior tibial and 1+ dorsalis pedis pulses. The most likely diagnosis is: A. Acute gout B. Raynaud’s C.Cellulitis D.Blue toe syndrome © AAFP Acute Arterial Occlusion

• Arterioarterial emboli: The Blue Toe Syndrome ‾ Cholesterol or atherothrombotic emboli ‾ Occludes small vessels ‾ Don’t be fooled • Pulses remain present • Often confused with bruising ‾ Can involve multiple organs – especially kidneys ‾ Can confirm diagnosis with • Skin or muscle biopsy • Cholesterol crystals on funduscopic exam Raynaud’s disease : Raynaud’s phenomenon recurrent vasospasm of the fingers/toes/tip of nose in response to stress or cold exposure • Primary: idiopathic (no underlying disease) • Secondary: associated with another illness • Rheumatic • Progressive systemic sclerosis/scleroderma • Mixed/Undifferentiated connective tissue disease, SLE • Dermatomyositis, Sjogren’s • Hematologic • Cryoglobulins 3-5% of population • Paraneoplastic syndromes Most do not seek treatment Thank you! Answers

1. A 2. C 3. A 4. C 5. A 6. D Supplemental Slides Raynaud’s phenomenon: Diagnosis

• Dx: biphasic white (pallor) and blue (cyanosis) change in skin color • Primary vs. Secondary: - primary seen in younger patients (ages 15-30) - 37% initially dx as primary are later discovery to be secondary - secondary has abnormal capillaroscopy © 2020 American Academy of Family Physicians. All rights reserved.

All materials/content herein are protected by copyright and are for the sole, personal use of the user.

No part of the materials/content may be copied, duplicated, distributed or retransmitted

in any form or medium without the prior permission of the applicable copyright owner.

The Major Arthritides

Benjamin Gilmer, MD, MS MAHEC Family Medicine University of North Carolina SOM Disclosure Statement

It is the policy of the AAFP that all individuals in a position to control content disclose any relationships with commercial interests upon nomination/invitation of participation. Disclosure documents are reviewed for potential conflicts of interest. If conflicts are identified, they are resolved prior to confirmation of participation. Only participants who have no conflict of interest or who agree to an identified resolution process prior to their participation were involved in this CME activity.

All individuals in a position to control content for this session have indicated they have no relevant financial relationships to disclose. Learning Objectives

1. Cite a systematic approach to the diagnosis of arthritic disorders. 2. Discuss the Dx & management of RA, OA, & gout. 3. Identify indications and use of disease- modifying antirheumatic drugs. 1. A 34 yo F with Hx of lupus presents with acute pain, rubor, calor and swelling in her left knee. She denies recent trauma and is currently taking hydroxychloroquine and prednisone. Her other SLE sx have been well-controlled. The most likely cause of this patient’s knee pain is: A. Infectious arthritis B. Patellofemoral syndrome C. Rheumatoid arthritis D. Sjögren syndrome E. Systemic lupus erythematous flare An Approach to Articular Disease

Look for 1 of 3 patterns • Monoarticular • Symmetric Polyarticular • Asymmetric Polyarticular Acute Monoarthritis • Inflammation (swelling, tenderness, warmth) in one joint • Occasionally polyarticular diseases can present with monoarticular onset: (RA, JRA, reactive and enteropathic arthritis, sarcoid, viral, psoriatic arthritis) • The most critical diagnosis to consider is – acute infection Monoarticular Differential

Common Less Common • Osteoarthritis •Rheumatologic • Crystals (Gout) ‾ Ankylosing spondylitis • Trauma ‾ JRA • Infectious ‾ Psoriatic arthritis - Septic ‾ Reactive arthritis - Viral ‾ Sarcoidosis • Neoplastic ‾ SLE • Overuse •Hemoglobinopathies • Vascular (Necrosis) Symmetric Polyarthritis

• Rheumatoid Arthritis • Polymyalgia Rheumatica • Systemic Lupus (older pt with shoulder/ hip symptoms) Erythematosus • Lyme disease (Skin findings) • CPPD (pseudogout) • Psoriatic Osteoarthritis • Sarcoid (CXR) • Scleroderma • Spondyloarthropathy (IBD) Key Points in the Approach to Arthritides 1. Distinguish arthritis from non-articular, soft tissue syndromes • “Active” ROM restriction implies soft tissue • “Passive” ROM (intra-articular) implies joint involvement 2. Distinguish single-joint from multiple-joint involvement 3. Mechanical vs. inflammatory mechanism Diagnostic Clues •Active ROM restricted Periarticular •Back pain/eye inflammation Ankylosing spondy •Coagulopathy Hemarthrosis •Diuretics/renal stones/tophi Gout •Hilar adenopathy Sarcoidosis •Immunosuppression/steroids Septic arthritis •Insidious onset of pain OA •Previous attacks w/ resolution Crystals •Skin Lesions Psoriatic arthritis 2. A 59 yo F presents with symptoms of fatigue, mild weight loss, and morning stiffness in both hands lasting more than an hour. She notes worsening tenderness in her MCPs and DIP’s bilaterally. Which of the following is the most specific to the diagnosis of this inflammatory process?

A. High ESR B. Positive ANA C. Rheumatoid joint erosions D. Rheumatoid factor Rheumatoid Arthritis

Rheumatoid joint erosions most specific to RA diagnosis

Typically associated with RA - High ESR - Positive ANA - Positive RF 3. When using the Joint Score Classification, which of the following does not contribute toward a diagnosis of RA? A. 4-10 small joints affected B. + Rheumatoid factor C. + Sedimentation rate D. Duration of symptoms less than 6 weeks E. + anticyclic citrullinated peptide antibodies Criteria for RA Every patient with >6 points is unequivocally positive for RA ‾ Patients with sx in >1 joint with clinical synovitis not explained by another diagnosis. 4 areas considered in diagnosis ‾ Number of Joints: 2-10 large joints = 1; 1-3 small joints = 2; 4-10 small joints = 3; > 10 small joints = 5 ‾ Serology: + RF and ACPA ‾ Acute Phase reactants: + CRP/ESR ‾ Duration of symptoms: > 6 weeks 4. After diagnosing RA, when should disease modulating anti-rheumatic drugs (DMARDs) be initiated? A. Promptly upon making the diagnosis B. Following 3 months of NSAID therapy C. Following a 5-day steroid burst. D. Only after the patient has failed conservative therapy DMARDs (Nonbiologics)

• Methotrexate: Inhibits dihydrofolate reductase: First line! - Need 1000 mg of folate daily • Leflunomide (Arava): Inhibits pyrimidine synthesis • Hydroxychloroquine (Plaquenil): Antimalarial, blocks toll-like receptors • Sulfasalazine (Azulfidine): Folate depletion • Minocycline (Minocin): Antimicrobial. MOA unknown. DMARDs (Anti-TNF-α Biologics)

• Adalimumab (Humira) • Certolizumab (Cimzia) • Etanercept (Enbrel) • Golimumab (Simponi) • Infliximab (Remicade) Key Recommendations for Practice (SORT)

• Combination therapy with two or more DMARDs is more effective than monotherapy. However, more than one biologic agent should not be used at one time. (A) • No regimen of monotherapy is clearly superior to any other. (A) • A guided exercise program can improve quality of life and muscle strength. (B) • Patients with inflammatory joint disease should be referred to rheumatology if symptoms last more than six weeks. (C) Key Recommendations for Practice (SORT)

• Cardiovascular disease is the main cause of mortality in persons with RA; therefore, risk factors for coronary artery disease should be addressed in these patients. (C) • Corticosteroids are effective but have high toxicity. Use lowest dose possible for shortest time. (A) Extraarticular Manifestations of RA • Accelerated CAD • Neuropathy • Pericarditis • Pleural effusion • Scleritis • Interstitial lung disease • Keratoconjunctivitis • Pulmonary nodules • Ulcerative keratitis • Rheumatoid nodules • Amyloidosis • Vasculitis • Splenomegaly Major Spondyloarthropathies

• Ankylosing spondylitis • Reactive arthritis (Reiter’s syndrome) • Psoriatic arthritis • Inflammatory bowel disease – associated spondyloarthropathy • Undifferentiated spondyloarthropathy Ankylosing Spondylitis

• The most common spondyloarthropathy 0.1-0.2% • Related to the prevalence of HLA-B27 • Most often affects white males between 15-40 • Back pain insidious, dull, radiates to gluteal, worse in AM, improves with activity, but nocturnal sx • Enthesitis common – Achilles and plantar fascia • Anterior uveitis is common • First-line tx: NSAIDs 5. A 35 yo M presents with a 3-month history of an asymmetric polyarthritis noted in 6 separate joints. PE reveals pitting of the fingernails along with a scaly, silver rash noted on both elbows. Which tx has been shown to delay further progression of this patient’s joint erosion? A. Aspirin B. Etanercept C. Methotrexate D. Naproxen sodium E. Sulfasalazine Psoriatic Arthritis

© American Academy of Dermatology © Richard Usatine, MD Psoriatic Arthritis

• Asymmetric, oligoarticular-associated dactylitis • Predominant DIP involvement – nail changes • Polyarthritis “RA-like” – lacks RF or nodules • Arthritis mutilans – destructive erosive hands/feet • Axial involvement – spondylitis • HIV-associated – more severe Psoriatic Arthritis

• Prevalence of arthritis in psoriasis: 5-20% ‾ Psoriasis usually precedes PSA – 75% ‾ In 15-20%, arthritis precedes psoriasis ‾ Nail changes common ‾ Usually insidious, but 1/3 have acute onset ‾ Dactylitis ‾ Uveitis • Psoriatic plaques ‾ Scalp, extensor surfaces, anal cleft, umbilicus Diagnosis of Psoriatic Arthritis

Point system: Need to have established inflammatory articular disease with >3 points from • Current psoriasis: 2 points • Hx of psoriasis: 1 point • FHx of psoriasis: 1 point • Dactylitis: 1 point • RF negative: 1 point • Nail dystrophy: 1 point • Juxta-articular new bone formation: 1 point • No specific lab findings Treating Psoriatic Arthritis

• Physical therapy: Start early! • Treatment ‾ DMARDs for slowing down joint erosions ‾ Methotrexate ‾ Sulfasalazine: Cochrane, 2009

Cochrane, 2009 Reactive Arthritis (Reiter’s)

• Triggered by an extra-articular infection • Reiter’s triad: (1) nongonococcal urethritis, (2) conjunctivitis and (3) arthritis • Sx usually occur 1-4 weeks after infection • Typically oligoarticular (lower extremities) • Onset is acute (days), 2-4 joints become painful and swollen in asymmetric distribution Key Clinical Recommendations • Clinical criteria supported by laboratory tests, synovial fluid analysis, and radiographs help to establish the presence of spondyloarthropathies. (C) • Initial management begins with NSAIDs. (C) • Sulfasalazine (Azulfidine) may be an effective second-line agent that provides short-term relief. (B) • DMARDs such as etanercept (Enbrel) and infliximab (Remicade) are effective in treating inflammatory symptoms. (B) • Despite the possibility of a bacterial etiology in reactive arthritis, antibiotic therapy has been ineffective. (B) • Second-line treatment of psoriatic arthritis (after NSAIDs) includes systemic corticosteroids, methotrexate, sulfasalazine, cyclosporine, and tumor necrosis factor-α inhibitors. (B) 6. Right-handed 80 yo M, previous painter, presents with chronic right, dull hand pain. States his DIPs and PIPs most tender, but not his MCPs. On exam, there is mild swelling in DIP/PIP distribution but no rubor or calor. Which of the following is NOT consistent with this diagnosis?

A. Osteophytes at the joint margins B. Changes in joint alignment C. Heberden’s nodes D. MCP inflammation Osteoarthritis A chronic joint disorder in which there is progressive softening and disintegration of articular cartilage accompanied by new growth of cartilage and bone at the joint margins (osteophytes) and capsular fibrosis. ‾ Primary or idiopathic ‾ Secondary • Posttraumatic, childhood anatomic abnormality, metabolic, neuropathic, endocrinopathy, Paget’s, gout, septic arthritis Osteoarthritis Symptoms

• Absence of systemic findings • Minimal articular inflammation • Distribution: DIP and PIP but not wrist and MCP • Insidious onset, “aching or burning” • Transient stiffness • More common in hands and large weight-bearing joints Osteoarthritis X-Ray Changes

• Joint space narrowing • Subchondral sclerosis • Osteophytes • Osteoarthritis X-Ray Changes OA Medical Management

• NSAIDs • APAP – no longer has great data for OA of large joints • Walking aids • Weight loss • Physiotherapy – aquatics • Altered activity • Injections: Steroids vs. Hyaluronic Acid ‾ Weaker data to support the use of hyaluronic acid Osteoarthritis Key Practice Guidelines AAFP 2014 • Persons with symptomatic knee osteoarthritis should participate in self-management programs, strengthening, low- impact aerobic exercise, and neuromuscular education. • Acupuncture, glucosamine, and chondroitin are not recommended therapies for knee osteoarthritis. • Recommended pharmacologic therapies include oral or topical nonsteroidal anti-inflammatory drugs or tramadol. • Hyaluronic acid injections are not recommended, and evidence to support corticosteroid injections is inconclusive. 7. A 53 yo M with hx of renal stones presents with an exquisitely painful left great toe and heel that started last night. Patient denies fever, rash and other systemic symptoms. Which diagnostic feature is most specific to this inflammatory disorder? A. A serum uric acid >6.8 mg/dL B. Negatively birefringent crystals in tissues C. Radiographic evidence of joint erosion D. Positively birefringent crystals in the tissues Gout: Common Clinical Symptoms

• Acute gout often begins at night. • Rapid onset of rubor, calor, swollen, intensely tender. • Rare fever, rash or other systemic signs. • Involvement of 2 or more joint is uncommon in early flares. • MTP of the great toe (podagra) is the first joint affected in half of all cases. • 20% of patients have renal stones. Gout: A Chronic Disease of 4 Stages

1. Asymptomatic hyperuricemia 2. Acute flares of crystallization 3. Intervals between flares 4. Advanced gout & complications Gout and Serum Uric Acid

• Hyperuricemia alone is insufficient to diagnose gout!

• Gout risk increases when serum levels persistently exceed 6.8 mg/dL – at which point extracellular fluids become saturated and hyperuricemia occurs. Microscopy and the Boards

• Gout: MSU crystals appear as needle-shaped intracellular and extracellular crystals. When examined with a polarizing filter and red compensator filter, they are negatively birefringent. • Pseudogout: CPP crystals appear shorter than MSU crystals and are often rhomboidal and positively birefringent. Foods That Increase Gout Risk

•Foods high in protein ‾ Beef, pork, lamb (RR 1.41) ‾ Seafood (RR 1.51)

•High-fructose corn syrup ‾ In women, one fructose-sweetened soft drink daily raises risk for gout by 74% (RR 1.74) ‾ 2 drinks per day 139% (RR 2.39) Antihypertensives and Gout

•After adjusting for multiple confounders •CCBs RR 0.9 •Losartan RR 0.8 •Thiazides RR 2.4 •B-Blockers RR 1.5 •ACEIs RR 1.3 •Nonlosartan ARBs RR 1.3 ACUTE MANAGEMENT OF GOUT Acute INITIATE PROPHYLAXIS - NSAIDS (with PPI) (A) - Low-dose colchicine (0.6mg qd or BID) (A) Management - Low dose corticosteroid (if colchicine or NSAIDS not tolerated) (C) of Gout NO symptoms EVALUATE YES symptoms SYMPTOMS while on ULT

DETERMINE DURATION CONTINUE RX

At least 6 months (A) CONTINUE anti- inflammatory prophylaxis TOPHI? NO TOPHI?

6 months after 3 months after achieving target achieving target UA if tophi UA level if no detected on PE tophi (B) (C) Gout Indications for UA lowering agent (UALA) • Tophus or tophi (A) • Frequent attacks (>2 attacks/yr) (A) • CKD stage 2 or worse (C) • Gout with urolithiasis (C) • UA overproduction and urinary overexcretion (>1000 mg daily) (C) American College of Rheumatology 2012 Recommendations for Acute Flare • Monotherapy with NSAIDs, steroids, or colchicine for mild-to-moderate attack. • Colchicine should be the first-line therapy for “attack prophylaxis” • Prophylaxis should continue for the greater of: 6 months, 3 months after achieving target UA levels without tophi, or 6 months after achieving target levels with resolution of tophi. Bibliography

1. Gill JM, Quisel AM, Occa PV, Walters DT. Diagnosis of systemic lupus erythematosus. AFP. 2003; 68:2179-86. 2. Siva C, Velasquez C, Mody A, Brasington R. Diagnosing acute monoarthritis in adults. AFP. 2003;68:83-90. 3. Richie AM, Francis ML. Diagnostic approach to polyarticular joint pain. AFP. 2003; 68:1151-60. 4. Lane SK, Gravel JW. Clinical utility of common serum rheumatologic tests. AFP. 2002;65:1073-80. 5. Rindfleisch JA, Muller D. Diagnosis & Management of Rheumatoid Arthritis. AFP. 2005;72:1037-47. 6. Westlake SL et al. The Effect of Methotrexate on Cardiovascular Disease in Patients with Rheumatoid Arthritis. Rheumatology. 2010;49:295-307. 7. Avina-Zubieta J. Rheumatology, Jan, 2013. 8. British Soc of Rheumatology; The 2012 guideline for the treatment of psoriatic arthritis…. Rheumatology. 2013;52:1754-7. Bibliography

9. E Ringdahl, and S Pandit. Treatment of Knee Osteoarthritis. Am Fam Physician 2011 Jun 1; 83(11):1287-1292 10. A Wasserman. Diagnosis and Management of Rheumatoid Arthritis. Am Fam Physician. 2011 Dec 1;84(11):1245-1252. 11. ACR Guidelines for Pharmacologic and Nonpharmacologic Treatment of Gout. Available at: http://onlinelibrary.wiley.com/doi/10.1002/acr.21772/pdf 12. ACR Recommendations on Therapies for Osteoarthritis of the Hand, Hip, and Knee. Available at: http://ql3dq7xx6q.search.serialssolutions.com/?sid=Entrez:PubMed&id=pmid:2256358 9 13. Implementing AHRQ Effective Health Care Reviews: Helping Clinicians Make Better Treatment Choices. Analgesics for Osteoarthritis. Am Fam Physician. 2013 Mar 1;87(5):354-356. Bibliography

14.Choi HK et al. Purine-rich foods, dairy and protein intake, and the risk of gout in men. N Engl J Med 2004 Mar 11; 350:1093-103 15.Choi HK et al. Fructose-rich beverages and risk of gout in women. JAMA 2010 Nov 10. 16.Choi HK et al. Antihypertensive drugs and risk of incident gout among patients with hypertension: Population based case-control study. BMJ 2012 Jan 12; 344:d8190 Answers

1. A 2. C 3. D 4. A 5. B 6. E 7. B Supplementary Slides AFP Review Articles

•Overview •12/15/2014Diagnosis, Treatment, and Prevention of Gout •01/01/2012Osteoarthritis: Diagnosis and Treatment •12/01/2011Diagnosis and Management of RA •09/01/2008Shoulder Osteoarthritis: Dg and Management AFP Review Articles

•11/01/2015Exercise for Osteoarthritis of the Knee •06/01/2015Colchicine for Acute Gout •01/01/2015Methotrexate Therapy for RA •07/15/2014Corticosteroid Injections for OA of the Knee •06/01/2014Tx of Knee OA: A Clinical Practice Guideline •04/01/2013ACR Recommendations on Therapies for OA •03/01/2013Analgesics for Osteoarthritis •09/01/2011Exercise for the Tx of Knee Osteoarthritis Non-Rheumatic Diseases with Positive RF

Hepatitis C < 70% Mixed cryoglobulinemia 90% Sarcoidosis 5-30% Pulmonary fibrosis 20% Infections Varies Aging 5% ANA

• Reported as titers: > 1:320 more likely to be true dz • Titers of < 1:40 unlikely to have a rheumatologic dz • ANA pattern is more specific for dz • Best for SLE, drug-induced lupus, Sjögren, scleroderma & MCTD Disease-Specific ANAs Disease ANA Biomarker SLE Anti-Smith RA RF Scleroderma Anti-centromere MCTD Anti-U1RNP Polymyositis Anti-Jo-1 Sjögren Anti-SSA & anti-SSB Wegener c-ANCA & p-ANCA Chromatin Antibodies • Anti-dsDNA: Rule in SLE • Anti-histone: Rule out drug-induced lupus • Anti-Smith: Rule in SLE • Anti-Ro: Sjögren • Anti-centromere: Scleroderma • c-ANCA: Wegener’s Pseudogout

• Monoarthritis; clinically indistinguishable from gout. • Often precipitated by illness or surgery. • Pseudogout is most common in the knee (50%) and wrist. • Reported in any joint (including MTP). • CPPD disease may be Asx (deposition of CPP in cartilage). UALA: Allopurinol • MOA: Reduces the production of UA (inhibits xanthine oxidase). • Starting dose: 100 mg/d (50 mg/d if CrCl 30-60). • Increase dose in 100 mg increments until the UA is <6.0 or max dose of 800 mg. • Typical dose is 300 mg/d. • Maintenance UA q 3 mo for 6 mo then once yearly. Allopurinol Hypersensitivity

• 1% of patients can develop severe allopurinol hypersensitivity syndrome, which carries a mortality of 20-30%. • Severe allopurinol hypersensitivity syndrome is more likely to occur in patients with renal insufficiency, those who are taking a thiazide diuretic, and those started on allopurinol at a dosage of 300 mg/day. Lyme Arthritis • Erythema migrans 7-10 days after Borrelia burgdorferi tick bite • Early dissemination – Migratory arthralgias, fever, systemic complaints • Late dissemination/chronic disease – Migratory oligoarthritis – Carditis – Neurological © 2020 American Academy of Family Physicians. All rights reserved.

All materials/content herein are protected by copyright and are for the sole, personal use of the user.

No part of the materials/content may be copied, duplicated, distributed or retransmitted

in any form or medium without the prior permission of the applicable copyright owner.

Acute CVA and TIA

Robert Dachs, MD, FAAFP Clinical Associate Professor Ellis Hospital Family Medicine Residency Program Albany Medical College, Albany, New York Disclosure Statement

It is the policy of the AAFP that all individuals in a position to control content disclose any relationships with commercial interests upon nomination/invitation of participation. Disclosure documents are reviewed for potential conflicts of interest. If conflicts are identified, they are resolved prior to confirmation of participation. Only participants who have no conflict of interest or who agree to an identified resolution process prior to their participation were involved in this CME activity.

All individuals in a position to control content for this session have indicated they have no relevant financial relationships to disclose. Learning Objectives 1. Identify patients with underlying risk factors for stroke. 2. Employ cost-effective measures to decrease stroke in patients at risk. 3. State the 2009 AHA/ASA definition of TIA and describe the recommended evaluation. 4. Propose appropriate treatment options to improve outcomes in patients who suffer a stroke. In 30 Minutes … The Plan

• CVA risk factors and 1° prevention • Acute CVA care • 2° prevention • TIA – everything has changed Stroke Pathogenesis 1. Acute stroke events are most often the result of which of the following pathological process?

A. Acute thrombosis B. Acute embolic event C. Acute intracerebral hemorrhage D. Acute subarachnoid hemorrhage Stroke Type/Subtypes Large-vessel Thrombotic (50%) (20-25%) Ischemic (80%-85%) Small-vessel Embolic (30%) (20-25%)

Intracerebral (10%) Hemorrhagic (15%-20%) Subarachnoid (6%)

What is a “cryptogenic stroke”? Stroke Type/Subtypes Large-vessel (20-25%) Thrombotic (50%) Ischemic (80-85%) Small-vessel (20-25%) Embolic (30%)

But the underlying cause is undetermined

What is a “cryptogenic stroke”? Stroke Risk Factors

• Modifiable vs. Non-modifiable – Age – Family Hx – Ethnicity Modifiable Risk Factors and CVA

70

60

50 Accounts 40 for 90% 30 of all 20 strokes 10

0 HTN Afib DM Physical Smoking inactivity Add in: hyperlipidemia, excessive alcohol use INTERSTROKE study, Lancet 2016 Stroke Type/Subtypes and

Risk Factor: HTN Large-vessel (20-25%) Thrombotic (50%) Ischemic (80-85%) Small-vessel (20-25%) Embolic (30%)

Intracerebral (10%) Hemorrhagic (15-20%) Subarachnoid (6%) If you can control BP, can you reduce CVAs? Stroke Type/Subtypes and Risk Factor: HTN Large-vessel Thrombotic (50%) (20-25%) Ischemic (80-85%) Small-vessel Embolic (30%) (20-25%)

Intracerebral (10%) Hemorrhagic (15-20%) Subarachnoid (6%) If you can control BP, can you reduce CVAs? YES! * Supporting Evidence: Control HTN (the Placebo-Controlled Trials) Population Treatment NNT/yrs MRC, 1985 17,354 pts a) Bendrofluazide or 90/5.5yrs Age: 36-64 b) Propranolol vs. 192/5.5yrs Diastolic: 90-109 placebo 43/4.5yrs 4,736 pts Chlorthalidone vs placebo SHEP, 1991 Age: 60+ BP:160-219/90+ 34/4.0yrs STOP, 1991 1,627 pts a) B-Blocker, or Age:70-84 b) HCTZ + amiloride, or BP: 180-230/90+ c) ACE-I vs. placebo MRC-2, 1992 4,396 pts a) HCTZ + amiloride, or 53/5.8yrs Age: 65-74 b) Atenolol vs 103/5.8yrs BP: 160-209/<115 placebo Stroke Type/Subtypes and Risk Factor: DM Large-vessel (20-25%) Thrombotic (50%) Ischemic (80-85%) Small-vessel Embolic (30%) (20-25%)

Intracerebral (10%) Hemorrhagic (15-20%) Subarachnoid (6%) Stroke Type/Subtypes and Risk Factor: Afib Large-vessel (20-25%) Thrombotic (50%) Ischemic (80-85%) Small-vessel Embolic (30%) (20-25%)

Intracerebral (10%) *if anticoagulated Hemorrhagic (15-20%) Subarachnoid (6%) Atrial Fibrillation and Stroke

8 7 6

5 4 3 CVA rate 2 (% per yr) 1 0 < 65 yrs 65-75yrs > 75 yrs • Anticoagulation decreases absolute annual risk from 4.5% -> 1.4%(NNT=30) • The older the patient with atrial fibrillation, the higher the risk of cardioembolic stroke • Strokes due to Afib have higher mortality and morbidity. Afib and CVA risk stratification: CHA2DS2-VASc

Score CHA2DS2-VASc 1 CHF Risk Category 1 HTN 0: Low-risk (ASA) 2 Age >75 yrs 1: Women: (ASA or DOAC > warfarin) 1 DM 1: Men: (ASA or DOAC > warfarin)* 2 Prior Stroke or TIA 2+: High-risk (DOAC > warfarin)* 1 Vascular disease 1 Age 65-74 yrs 1 Female sex

*2019 AHA/ACC guideline changes Note: DOAC noninferior to warfarin, improved safety profile Atrial Fibrillation: Anticoagulation Potential Harms vs. Potential Benefits

• Decreases CVA – approx. 3% per yr • Best calculator: www.afib.ca • Rate of ICH 0.1-0.6% –Increased with advanced age, HTN • Major bleeding rates: 1.2% per yr Which of the following new oral anticoagulants is approved for the management of non-valvular Afib?

A.Dabigatran (Pradaxa) – BID B.Rivaroxaban (Xarelto) – qd C.Apixaban (Eliquis) – BID D.All of the above Which of the following new oral anticoagulants is approved for the management of non-valvular Afib?

A.Dabigatran (Pradaxa) – BID B.Rivaroxaban (Xarelto) – qd C.Apixaban (Eliquis) – BID D.All of the above If you were asked… Which of the following new oral anticoagulants is approved for the management of DVT/PE?

A.Dabigatran (Pradaxa) B.Rivaroxaban (Xarelto) C.Apixaban (Eliquis) D.All of the above If you were asked… Which of the following new oral anticoagulants is approved for the management of DVT/PE?

A.Dabigatran (Pradaxa) B.Rivaroxaban (Xarelto) C.Apixaban (Eliquis) D.All of the above Modifiable Risk Factors and CVA

70

60

50

Accounts 40 for 90% 30 of all 20 strokes 10

0 HTN DM Smoking Afib Physical inactivity Add in: hyperlipidemia, excessive alcohol use INTERSTROKE study, Lancet 2016 AHA/ASA Guideline for Primary Prevention of Ischemic Stroke (2014)

• Obesity: recommend weight reduction Class I, Level of evidence B • Physical Activity: recommend it Class I, Level of evidence B (moderate to vigorous aerobic activity 3-4 x/week) • Hypertension: annual screen, treat if >140/90 Class I, Level of evidence A

• Diabetes: control HTN, Class I, Level of evidence A -start statin if > 1 risk factor Class I, Level of evidence A • Smoking: Stop Class I, Level of evidence A AHA/ASA Guideline for Primary Prevention of Ischemic Stroke (2014)

• Alcohol: for those who choose Class IIb, Level of evidence B - ≤2 drinks/day for men and ≤1 drink/day for women “might be reasonable”

• A fib: anticoagulate if CHA2DS2-Vasc > 2 Class I, Level of evidence A

• Lipids: see 2018 lipid guideline in Cardiology lecture* reproduced in Supplemental slides at end of lecture

What about aspirin for primary prevention of CVA/CVD? USPSTF 2016 Recommendations Aspirin for 1° Prevention for CV/Stroke

The Big 3 randomized trials…

•ARRIVE trial (Lancet, Sept 22, 2018) −12,546 pts (55yrs+), Ave CV risk score (17% over 10yrs) – followed 5 yrs −No difference in Composite CV endpt, or mortality − 0.5% increase bleeding with ASA •ASPREE trial (NEJM, Oct 18, 2018) −19,114 pts (70yrs+), no hx of CV disease – followed for 5 yrs −No difference in death, dementia or disability, 1% increase in bleeding

•ASCEND trial (NEJM, Oct 18, 2018) −15,480 pts with DM but no CV disease, Ave age 63, – followed 7.4 yrs −1.1% decrease in first vascular event, 0.9% increase in bleeding Published Online, March 2019 Aspirin for 1° prevention for CV/Stroke USPSTF 2016 Vs. AHA/ACC 2019

IIb

III Stroke Risk Factors • Traditional vs. novel

• Modifiable vs. Non-modifiable – Age – Family Hx – Ethnicity The Last “Traditional” Risk Factor: What About Family History?

• Documented parental stroke by 65 yrs of age is associated with a 3-fold increase in stroke in offspring.

Based on 8-year follow-up of 3,443 stroke-free Framingham offspring

Seshadri S, et al. Circulation. 2010 Risk Factors: Intracerebral Hemorrhage

• Hypertension • High-risk groups • Amyloid angiopathy – Older age – Ethnicity • AVMs (African American, Asian) • Brain tumors • Bleeding disorders • Drugs • Vasculitis – Anticoagulants, antiplatelets – Cocaine • CNS infection – Amphetamines • Septic embolism – SSRIs Subarachnoid Hemorrhage • 80% due to saccular aneurysms % of SAH per 100 cases • Who is at risk? 80 • Hypertension 70 60 • Smoking 50 • Vasculitis, SLE 40 30 • Genetic 20

10 • Peak age 50 0 < 10 yrs 10-20 yrs 20-40 yrs 40-65 yrs Sudden “thunderclap” headache, “worst HA of my life” Neck pain/nuchal rigidity, vomiting, onset with exertion In 30 Minutes … The Plan • CVA risk factors and prevention • Acute CVA care • Secondary prevention • TIA – everything has changed 2. An 82 yo male developed sudden dysarthria and RUE weakness. Onset at 8am. He arrives at the ED at 11 am. IV is inserted, labs sent. Which of the following imaging studies should be ordered STAT?

A. Non-contrast head CT B. Contrast head CT C. CT- A (angiogram) of the head D. Non-contrast MRI of head Stroke Type/Subtypes Large-vessel (20-25%) Thrombotic (50%) Ischemic (80-85%) Small-vessel Embolic (30%) (20-25%)

Intracerebral (10%) Hemorrhagic (15-20%) Subarachnoid (6%) Stroke Type/Subtypes

Ischemic (80-85%)

<------>

Hemorrhagic (15-20%) 3. An 82 yo male developed sudden dysarthria and RUE weakness. Onset at 8 a.m. He arrives at the ED at 11 a.m. BP= 200/100 At noon, the labs and head CT are reported as “normal.” Your management will include…? A.Initiate IV thrombolytic (tPA), initiate IV nicardipine (for BP control), initiate aspirin 325 mg po. B.Initiate IV tPA, initate IV nicardipine, do NOT initiate ASA 325 mg C.Do NOT start IV tPA, initiate IV nicardipine, initiate ASA 325 mg D.Do NOT start IV tPA, do NOT IV nicardipine, initiate ASA 325 mg 3. An 82 yo male developed sudden dysarthria and RUE weakness. Onset at 8 a,m. He arrives at the ED at 11 a.m. BP= 200/100 At noon, the labs and head CT are reported as “normal.” Your management will include…? A.Initiate IV thrombolytic (tPA), Initiate IV nicardipine (for BP control), initiate Aspirin 325 mg po. B.Initiate IV tPA, initate IV nicardipine, do NOT initiate ASA 325 mg C.Do NOT start IV tPA, initiate IV nicardipine, initiate ASA 325 mg D.Do NOT start IV tPA, do NOT IV nicardipine, initiate ASA 325 mg The NINDS Trial: tPA within 3 hrs of Onset of Symptoms Positive Results • 39% of pts with minimal/no disability at 3 months (with tPA) vs. • 26% of pts with minimal/no disability at 3 months (with placebo) Number needed to treat (to experience benefit): 1 in 8

Absolute risk reduction (ARR) = 13% (NNT = 8)

Number needed to treat (NNT) = 1/ARR Example: 1/.12 = 8.3 The NINDS Trial Positive Results • 39% of pts with minimal/no disability at 3 months (with tPA) vs. • 26% of pts with minimal/no disability at 3 months (with placebo) Number needed to treat (to experience benefit): 1 in 8 Negative Results •6.4 % of pts develop intracranial hemorrhage (with tPA) vs. •0.6% of pts develop intracranial hemorrhage (with placebo) Number needed to harm: 1 in 16 I Heard That tPA Can Now Be Given Up to 4.5 Hrs After Onset of Stroke? ECASS 3, NEJM, 9/25/08: tPA 3 - 4.5 hours Results: (+) tPA (n=418) Placebo (n=403) mRankin score (0-1) 52.4% 45.2% Absolute difference = 7%, NNT = 14 Symptomatic ICH 2.4% 0.2% Absolute difference = 2.2%, NNH = 45 But only in patients < age 80 yrs!!! tPA (Thrombolytic for Stroke) Recommended for ischemic stroke when: 1) CT reveals no evidence of hemorrhage 2) No hx of intracerebral hemorrhage 3) No seizure at onset of stroke 4) BP <185/110

Age <80 yrs Age >80 yrs • Onset of symptoms <4.5 hrs •Onset of symptoms <3 hrs • INR <1.7 •No anticoagulants 3. An 82 yo male developed sudden dysarthria and RUE weakness. Onset at 8 a.m. He arrives at the ED at 11 a.m. BP= 200/100 At noon, the labs and head CT are reported as “normal.” Your management will include…? A.Initiate IV thrombolytic (tPA), Initiate IV nicardipine (for BP control), initiate Aspirin 325 mg po. B.Initiate IV tPA, initate IV nicardipine, do NOT initiate ASA 325 mg C.Do NOT start IV tPA, initiate IV nicardipine, initiate ASA 325 mg D.Do NOT start IV tPA, do NOT IV nicardipine, initiate ASA 325 mg Blood Pressure Control: CAUTION in Acute (ischemic) CVA!!!

• Elevated BP is body’s desire to maintain cerebral perfusion

• AHA guidelines: Treat BP systolic >220 (2003, 2007) Treat BP diastolic >120 • Recommended meds: 1. Labetalol: 10 mg q 10-20 min Goal: 15% decrease in BP 2. Nicardipine: 5 mg/hr, titrate q 5 min AHA Stroke Guideline, 2007, 2013, 2018 Blood Pressure Control: A. For Acute (Hemorrhagic) CVA!!!

• If systolic BP 150-220 – lower BP sys to <140 Class I, Level of Evidence A, AHA ------guideline, 5/2015 Blood Pressure Control: B. For Subarachnoid Hemorrhage!!! • Decrease systolic BP < 160 Class IIa, Level of Evidence C, AHA guideline, 2012 Acute Ischemic Stroke • What about aspirin? –Just say, “YES!”

AHA/ASA Ischemic Stroke Guidelines, 2007, 2013, 2018 Acute Ischemic Stroke • What about prophylactic antiseizure meds? –Just say, “NO!” • What about heparin? –Just say, “NO!” • What about warfarin? –Just say, “NO!” • What about clopidogrel (Plavix)? –Just say, “NO!” (***maybe in TIA/minor stroke) AHA/ASA Ischemic Stroke Guidelines, 2007, 2013, 2018 Stroke Units • CVA accounts for 4% of all hospital admissions • Cochrane Review (2013): 28 trials reviewed ===> decreases odds of death or dependency at 1 year!!! • Why? It’s not the high-tech stuff!!!!

1. Aspiration prevention, use of oxygen, and use of acetaminophen (for fever) were more commonly used in stroke units than general wards. 2. Less use of urinary catheters were noted in stroke units. 3. Stroke units experienced less stroke progression or recurrence, chest infections, other infections, falls, and pressure sores. 4. A review of death certificates suggests that stroke units do not prevent neurologic deaths, but deaths from stroke complications such as infections. The post-CVA patient now returns to your care (office/rehab). Note the following recommendations for secondary prevention of ischemic stroke or TIA:

• Hypertension: start or resume BP meds “beyond the first several days” Class I, Level of evidence A - Goal: <140/90 (Class IIA), ? <130 systolic for lacunar CVA (Class IIb) • Lipids: (+) statins - for LDL-C >100mg/dl Class I, Level of evidence B - for LDL-C <100mg/dl Class I, Level of evidence C

• Glucose/DM: Screen for DM Class IIa, Level of evidence C AHA/ASA guideline 2014 The post-CVA patient now returns to your care (office/rehab). Note the following recommendations for secondary prevention of ischemic stroke or TIA:

• ASA: 50-325mg qd monotherapy Class I, Level of evidence A OR • ASA 25mg + dipyridamole 200mg Class I, Level of evidence B OR • Clopidogrel 75mg qd monotherapy Class IIA, Level of evidence B

“Selection of an antiplatelet agent should be individualized” Class I, Level of evidence C

AHA/ASA guideline 2014 4. TIA (Transient Ischemic Attack) is Defined as:

A. Sudden focal neurologic deficit caused by focal brain ischemia of vascular origin that completely resolves in 24 hours. B. A brief episode of neurologic dysfunction caused by focal brain or retinal ischemia, with clinical symptoms typically lasting <1 hour, and without evidence of acute infarction. C. A brief episode of neurologic dysfunction caused by focal brain or retinal ischemia, with clinical symptoms typically lasting <1 hour, and with hyperacute changes on MRI. D. A transient episode of neurologic dysfunction caused by focal brain, spinal cord, or retinal ischemia, without acute infarction. TIA: The Definition Has Changed!!!! • Classic definition: sudden focal neurologic deficit caused by focal brain ischemia of vascular origin that completely resolves in 24 hours • 2002 TIA Working Group “A brief episode of neurologic dysfunction caused by focal brain or retinal ischemia, with clinical symptoms typically lasting less than 1 hour, and without evidence of acute infarction” TIA: The Definition Has Changed!!!! • Classic definition: sudden focal neurologic deficit caused by focal brain ischemia of vascular origin that completely resolves in 24 hours • 2002 TIA Working Group “A brief episode of neurologic dysfunction caused by focal brain or retinal ischemia, with clinical symptoms typically lasting less than 1 hour, and without evidence of acute infarction” TIA: Definition AHA/ASA Statement: June 2009 “A transient episode of neurologic dysfunction caused by focal brain, spinal cord, or retinal ischemia, without acute infarction”

Note 1: No time limitation Note 2: A tissue-based definition (no evidence of acute infarction) Why No Time Limits?

Duration of symptoms: hrs (+) MRI - DWI (ie. +CVA) 0-1 33.6% 1-2 29.5% 2-3 39.5% 3-6 30.0% 6-12 51.1% 12-18 50.0% 18-24 49.5% Frequency of DWI abnormalities in patients with TIA of different durations: Pooled data from 10 MRI studies enrolling 818 patients. Shah SH, et al. Stroke. 2007;38:463. MRI - DWI image

–Bright white signal –Associated with cytotoxic edema (i.e., cell death) –Occurs within minute of stroke

Source: VirtualMedStudent.com Why is this (the MRI) important? (in the patient that has returned to baseline) Patients with minor CVA (+ DWI - MRI) have a worse prognosis than those with true TIA (- DWI - MRI) Transient Neurologic Changes and has returned to baseline MRI / \ TIA CVA (-)MRI (+)MRI Acute Neurovascular Syndrome

MRI / \ TIA CVA (-)MRI (+)MRI TIA/Stroke Mimics (The Differential Diagnosis of TIA/CVA) • Structural brain lesion (tumor, hemorrhage, AVM, aneurysm) • Infection (focal abscess, septic emboli) • Seizure/Todd’s paralysis • Complicated migraine • Hypoglycemia • Syncope from any cause (especially arrhythmia) • Labyrinthine disorders • Temporal arteritis • Multiple sclerosis (flare) So My Patient Has a Neg (-) MRI Was It a TIA? Not Associated With TIA: TIA: Anterior Circulation Nonfocal Symptoms • Hemiparesis • Loss of consciousness** • Unilateral sensory loss • Dizziness • Visual field deficit • Generalized weakness • Gaze preference • Mental confusion • Aphasia • Vision: wavy lines, flashing lights • Left-sided spatial neglect (retina) • Limb shaking or “tingling” “negative” or “lost” • Incontinence TIA Management: Risk Assessment

Who is going on to acute CVA? Who is such high-risk they need hospitalization?

The AHA/ASA recommends the ABCD2 score to calculate a patient’s short-term risk of developing a CVA ABCD2 Score • Age: greater than or equal to 60 (1 pt) • Blood pressure: SBP > 140 or DBP > 90 (1 pt) • Clinical Features – Focal weakness (2 pt) or – Speech impairment without focal weakness (1 pt) • Duration of symptoms – > 60 minutes (2 pt) or Risk of CVA at 2 days – < 59 minutes (1 pt) 0-3 points = 1% risk • Diabetes (1 pt) 4-5 points = 4.1% risk Johnston SC, et al. 6-7 points = 8.1% risk Lancet. 2007;369:283-292. What Do You Do With the ABCD2 Score? In 2009, the AHA/ASA Recommended: “It is reasonable to hospitalize patients with TIA if they present within 72 hours of the event and any of the following criteria are present: • ABCD2 score of >3 • ABCD2 score of 0-2 and uncertainty that diagnostic workup can be completed within 2 days as an outpatient • ABCD2 score of 0-2 and other evidence that indicates the patient’s event was caused by focal ischemia” All Class IIa recommendations, Level of Evidence C Medical Management of a) Minor CVA and b) TIA

A. Minor CVA In patients presenting with minor stroke, treatment for 21 days with dual antiplatelet therapy (aspirin and clopidogrel) begun within 24 hours can be beneficial for early secondary stroke prevention for a period of up to 90 days from symptom onset. *New IIA recommendation (AHA/ASA, 2018) B. TIA Dual antiplatelet therapy (ASA + clopidogerel) are recommended for 10-21 days in patients with high-risk TIA (ABCD2 > 4) BMJ clinical practice guideline, published 12/18/18 In patients presenting with minor stroke, treatment for 21 days with dual antiplatelet therapy (aspirin and clopidogrel) begun within 24 hours can be beneficial for early secondary stroke prevention for a period of up to 90 days from symptom onset. *New IIA recommendation (AHA/ASA, 2018)

*CHANCE Study: Patients with minor ischemic stroke or TIA in China Methods: Double-blind, randomized (within 24 hrs) to: Results: ASA (75-300mg) ASA 75mg + placebo + Clopidogrel 300mg Day1, then 75mg qd CVA at 90 days 11.7% 8.6%

Note: dual therapy for 21 days, then clopidogrel 75mg qd. In patients presenting with minor stroke, treatment for 21 days with dual antiplatelet therapy (aspirin and clopidogrel) begun within 24 hours can be beneficial for early secondary stroke prevention for a period of up to 90 days from symptom onset. *New IIA recommendation (AHA/ASA, 2018) *POINT Study: Patients with minor ischemic stroke or TIA Methods: Double-blind, randomized (within 12 hrs) to: Results: ASA (50-325mg) ASA (50-325mg) + placebo + Clopidogrel 600mg Day1, then 75mg qd CVA at 90 days 6.3% 4.6% NNT=59

Note: dual therapy for 90 days NEJM, 2018 Summary: 2° Prevention for TIA and CVA

TIA (ABCD2 > 4) or minor CVA CVA (NIHSS > 3) (NIHSS score 3 or less)

ASA + clopidogrel x 21 days ASA qd (monotherapy)

Then…ASA qd (monotherapy)

------if fails (i.e., recurrence)------

Consider increasing dose or adding second agent BUT THERE IS NO EVIDENCE TO SUPPORT THIS What happens when patient with CVA/TIA on ASA has another event?

“for patients who have a noncardioembolic AIS while taking aspirin, increasing the dose of aspirin or switching to an alternative antiplatelet agent for additional benefit in secondary stroke prevention is not well established”.

AHA/ASA guideline update 2018 The TIA Diagnostic Evaluation Step 1: MRI – done … next➔ Step 2: Duplex US of carotids • “Duplex US is recommended to detect carotid stenosis in patients who develop focal neurological symptoms corresponding to the territory supplied by the L or R internal carotid artery”

• “…MRA or CTA is indicated to detect carotid stenosis when US either cannot be obtained or yields equivocal or …nondiagnostic results” Class I recommendation, Level of Evidence C AHA/ASA 2011 Guideline on Carotid and Vertebral Disease Released 1/31/11 Symptomatic Carotid Artery Disease: When Do You Recommend Endarterectomy? • Indicated in symptomatic patients with 70-99% stenosis • Consider in symptomatic patients with 50-70% stenosis – 3 trials (NASCET, ECST, VA) benefits of CEA best in: •Men > women •Age > 75 •Recent minor stroke (vs TIA) •Presence of hemispheric symptoms (not amaurosis fugax) •Early surgery (within 2 weeks of TIA) • Note: These studies done prior to era of widespread aggressive medical therapy AHA, American Stroke Association, 2006 5. Screening for Carotid artery disease… in 2007 and 2014, the USPSTF stated: A. All adults >65 years of age should have ultrasound screening for carotid artery disease. B. Adults >65 years of age with diabetes should have ultrasound screening for carotid artery disease. C. Adults >65 years of age with any of the common risk factors for atherosclerosis (DM, HTN, smoking, family history, or hypercholesterolemia) should have ultrasound screening for carotid artery disease. D. Adults should not be screened for carotid artery disease with ultrasound or other tests. One last plea...

DO NOT SCREEN for Carotid Artery Disease in ASYMPTOMATIC PATIENTS

USPSTF guideline 2007 and 2014 Carotid Artery Screening… (Asymptomatic Population): D recommendation • The bottom line: Prevalence of disease in age >65 = 1% • 4,348 persons would need screening to prevent 1 CVA in 5 years. • 8,696 persons would need screening to prevent 1 disabling CVA in 5 years. Carotid Artery Screening … (Asymptomatic Population): D recommendation

• Put another way: (the harms of screening) Prevalence of disease in age >65 = 1% • If you screen 100,000 adults… • 940 true positives • 7920 false positives* - Confirmatory angiography => 1.2% CVA rate - MRA = 90% specificity ==> 792 needless * Thank you! Answers 1. A 2. A 3. D 4. D Supplementary Slides 2019 ACC/AHA guideline on primary prevention of CV disease Published March 7, 2019

1) Patients ages 20-75 years and LDL-C ≥190 mg/dl, -- use high-intensity statin without risk assessment.

2) T2DM and age 40-75 years, use moderate-intensity statin -- and risk estimate to consider high-intensity statins. * Risk-enhancers in diabetics include: ≥10 years for T2DM and 20 years for type 1 DM, ≥30 mcg albumin/mg creatinine, eGFR <60 ml/min/1.73 m2, retinopathy, neuropathy, ABI <0.9. In those with multiple ASCVD risk factors, consider high-intensity statin -- with aim of lowering LDL-C by 50% or more.

3) Age >75 years, clinical assessment and risk discussion. 2019 ACC/AHA guideline on primary prevention of CV disease

4) Age 40-75 years and LDL-C ≥70 mg/dl and <190 mg/dl without diabetes, -- use the risk estimator A. Risk 5% to <7.5% (borderline risk). Risk discussion: if risk-enhancing factors are present, discuss moderate-intensity statin and consider coronary CACs in select cases. B. Risk ≥7.5-20% (intermediate risk). Risk discussion: use moderate-intensity statins and increase to high-intensity with risk enhancers C. Risk ≥20% (high risk). Risk discussion to initiate high-intensity statin to reduce LDL-C by ≥50%.

Option of CACs to risk stratify if there is uncertainty about risk. A. If CAC = 0, can avoid statins and repeat CAC in the future (5-10 years), *the exceptions being high-risk conditions such as diabetes, family history of premature CHD, and smoking. B. If CACs 1-100, it is reasonable to initiate moderate-intensity statin for persons ≥55 years. C. If CAC >100 or 75th percentile or higher, use statin at any age. Physical Activity: Just Do It!

• Methods: Women’s Health Study - 39,315 women, reported physical activity at baseline, followed 11.9 yrs

• Results: compared to sedentary (nonwalkers) - Walk > 2 hrs/week ==> lowered CVA risk 30% -… and speed (ie, vigorous) did not matter!!

Sattelmair, JR, Kurth T, Buring JE, et al. Physical activity and risk of stroke in women. Stroke. 2010 [Epub ahead of print]. What Is the Data Supporting tPA for Stroke? • 12 randomized trials of thrombolytics vs placebo for acute ischemic CVA

• 2 were positive • 10 were negative or neutral

The NNT.com The NINDS Trial (1995) Location # of Pts Time of CVA Drug and Dose United States 624 3 hours 1) tPA 0.9 mg/kg vs.2) Placebo RESULTS A. Mortality: No difference!!! B. Part 1: At 24-hour neurologic assessment (291 pts) No difference!!! C. Part 2: At 3-month neurologic assessment (333 pts) (+) Significant difference*** Heparin (UFH) and Low-Molecular-Weight Heparin (LMWH) for acute CVA? Just say “NO.” • AHA/ASA 2003, 2007 recommend “against” - IST (Lancet, 1997): 19,435 pts, UFH => no benefit - LMH - initial trials promising, subsequent disappoint - TOAST trial (JAMA,1998) LMH ==> no benefit • Cochrane Review: 24 trials (23,748 pts) ==> - 9 fewer ischemic strokes (per 1,000) with heparin - 9 more intracerebral hemorrhages (per 1,000) What about Aspirin and ischemic stroke? Just say “YES”

•Cochrane Review, Issue 3, 2008 -12 trials (43,041 pts) - 13 pts; alive and independent (per 1,000) with ASA - 10 more pts: made complete recovery (per 1,000) - 2 more pts: intracerebral hemorrhage (per 1,000) Endovascular Treatment: What are the outcomes?B. Stent retrievers (vs. standard therapy= IV tPA)

# of pts Hrs to treat mRS 0-2 @ 90 days NNT A. MR CLEAN 500 6hrs 19.1% vs. 32.6% 8 B. ESCAPE trial 316 12hrs 29.3% vs. 53.0% 4 C. EXTEND-IA trial 70 6hrs 40.0% vs. 71.4% 3 D. SWIFT PRIME 196 6hrs 35.5% vs. 60.2% 4 E. REVASCAT 206 8hrs 28.2% vs. 43.7% 7 Endovascular Treatment: What are the outcomes? Stent retrievers + IV tPA (vs. standard therapy= IV tPA)

# of pts Hrs to treat mRS 0-2 @ 90 days NNT Notes A. MR CLEAN 500 6hrs 19.1% vs. 32.6% 8 *90% +tPA B. ESCAPE trial 316 12hrs 29.3% vs. 53.0 4 *15% > 6hrs C. EXTEND-IA trial 70 6hrs 40.0% vs. 71.4% 3 *CT perfusion D. SWIFT PRIME 196 6hrs 35.5% vs. 60.2% 4 E. REVASCAT 206 8hrs 28.2% vs. 43.7% 7

Notes: 1) No mortality benefit 6. Very few pts > age 80 2) All manufacture-supported studies (average age = 67.4) 3) 6 hour limit – too few beyond this 4) CTA required to confirm large-vessel clot 5) Last 4 trials halted early: ?exaggerate results Endovascular Treatment: What are the outcomes? C. Stent retrievers + IV tPA (vs. standard therapy= IV tPA)

# of pts Hrs to treat mRS 0-2 @ 90 days NNT Notes A. MR CLEAN 500 6hrs ``````19.1% vs. 32.6% ``````` 8 *90% +tPA B. ESCAPE trial 316 12hrs 29.3% vs. 53.0% 4 *15% > 6hrs C. EXTEND-IA trial 70 6hrs ``````40.0% vs. 71.4% `````` 3 *CT perfusion D. SWIFT PRIME 196 6hrs `````` 35.5% vs. 60.2% ``````4 E. REVASCAT 206 8hrs `````` 28.2% vs. 43.7% `````` 7

Some have suggests to re-organize system Small % are IV tPA candidates =➔ to specialized centers Even less pts are candidates for this… Ex: Mr. CLEAN – 500pts/300 years/16 centers Result: 10pts/site = approx 1/month

What would be the unintended consequences (and costs) redirecting/shipping all? AHA/ASA 2018 AIS guideline The benefit of prophylactic-dose subcutaneous heparin (unfractionated heparin [UFH] or LMWH) in immobile patients with AIS is not well established. ASA/AHA 2018; IIb A New recommendation.

The most recent and comprehensive meta-analysis of pharmacological interventions for venous thromboembolism prophylaxis in AIS included 1 very large trial (n=14 578) and 4 small trials of UFH, 8 small trials of LMWHs or heparinoids, and 1 trial of a heparinoid.240 Prophylactic anticoagulants were not associated with any significant effect on mortality or functional status at final follow-up. There were statistically significant reductions in symptomatic pulmonary embolisms (OR, 0.69; 95% CI, 0.49–0.98) and in DVTs, most of which were asymptomatic (OR, 0.21; 95% CI, 0.15–0.29). There were statistically significant increases in symptomatic intracranial hemorrhage (OR, 1.68; 95% CI, 1.11–2.55) and symptomatic extracranial hemorrhages (OR, 1.65; 95% CI, 1.0–2.75). There may be a subgroup of patients in whom the benefits of reducing the risk of venous thromboembolism are high enough to offset the increased risks of intracranial and extracranial bleeding; however, no prediction tool to identify such a subgroup has been derived.197,198,240 AHA/ASA 2018 Acute Stroke recommendations © 2020 American Academy of Family Physicians. All rights reserved.

All materials/content herein are protected by copyright and are for the sole, personal use of the user.

No part of the materials/content may be copied, duplicated, distributed or retransmitted

in any form or medium without the prior permission of the applicable copyright owner.

Urologic Problems

Benjamin Gilmer, MD, MS MAHEC Family Medicine University of North Carolina SOM Disclosure Statement

It is the policy of the AAFP that all individuals in a position to control content disclose any relationships with commercial interests upon nomination/invitation of participation. Disclosure documents are reviewed for potential conflicts of interest. If conflicts are identified, they are resolved prior to confirmation of participation. Only participants who have no conflict of interest or who agree to an identified resolution process prior to their participation were involved in this CME activity.

All individuals in a position to control content for this session have indicated they have no relevant financial relationships to disclose. Learning Objectives

After attending this lecture, the learner will be able to 1. Identify the signs, symptoms, diagnosis and treatment/management of acute prostatitis, erectile dysfunction, hypogonadism and BPH 2. Discuss the (updated) USPSTF recommendation for screening for prostate cancer 3. Describe the typical work-up & diagnosis of urolithiasis

NOTE: • Urinary Incontinence →Geriatrics session; UTI → STIs • Supplemental Slides: Hematuria, Chronic prostatitis, testicular issues 1. A 52 yo M presents with evolving symptoms of frequent urination at night. He notes increase in dribbling following urination and less force of stream. Which of the following is the most appropriate first line agent of treatment? a) Ciprofloxacin b) Terazosin c) Dutasteride d) Saw Palmetto Benign Prostatic Hyperplasia

PREVALENCE Dynamic AGE PERCENT muscle tension 31-40 years 8% 51-60 years 45% 60-80 years >80% BPH

Increased • Lifetime risk of surgery = 29% prostate volume Benign Prostatic Hyperplasia

SYMPTOMS •  force of stream • Incomplete emptying • Hesitancy • Urgency • Terminal dribbling • Nocturia • Frequency

COMPLICATIONS • Acute urinary retention • Hydronephrosis • Recurrent UTIs • Renal failure BPH Meds

Alpha-1 antagonists 5-Alpha reductase inhibitors

• 5 approved • 3 approved • Similar efficacy • Similar efficacy • Different side effect profiles • Similar side effect profiles • tamsulosin, terazosin, • 6-12 months Rx for full effect doxazosin with SE • Reduce size •Dynamic component • Level A Rec; Cochrane, 2008 • Level A Rec; Cochrane, 2008 NNT for hematuria = 2 NNT to prevent a TURP = 6 BPH Meds cont.

• Combination treatment MAY help especially when >30 cc volume Level B Rec, Cochrane

• Saw palmetto is controversial & is no better than placebo Other BPH Treatments

• Tadalafil (Cialis) • Transurethral microwave • Reason for improvement is thermotherapy (TUMT) unknown • Is effective when there is • Level B • No urinary retention • Do not use with alpha-blockers • No previous prostate procedure • Prostate volumes between 30- •Transurethral resection of the 100 mL prostate (TURP) • Not as effective as TURP (Cochrane, 2007) Key Recommendations for Practice

•Alpha blockers are effective first-line treatments for patients with bothersome, moderate-severe symptoms (SOR A) Alpha •The addition of 5-alpha reductase inhibitor is Blockers effective in men with bothersome, moderate to severe BPH symptoms and a documented + 5 alpha enlarged prostate when alpha blocker reductase monotherapy is not effective. (SOR A) inhibitor 2. A 70 yo M presents with dysuria, urinary frequency, urinary urgency, incomplete voiding, and suprapubic pain for several days. He denies fever, chills, nausea, emesis, and malaise. His physical exam was significant for a tender, enlarged, boggy prostate. You diagnose him with acute bacterial prostatitis (ABP). All of the following risk factors increases the risk of a poor prognosis with ABP in his age group EXCEPT? a) History of BPH b) BMI >25 c) Temperature greater than 100.4°F d) Urinary retention e) Transurethral catherization Acute Bacterial Prostatitis

• Incidence peaks: 20-40y & >70 y • Symptoms include: • Voiding symptoms: irritative (e.g., dysuria, urinary frequency, urinary • Dx with history & physical urgency) or obstructive (e.g., hesitancy, incomplete voiding, • PE: Tender, warm, straining to urinate, weak stream) swollen/boggy, firm & irregular • May have suprapubic, rectal, or perineal pain • Painful ejaculation, • Diagnostics: UA , C&S hematospermia, and painful defecation possible • Systemic symptoms (fever, chills, nausea, emesis, and malaise) → assess for sepsis Differential Diagnosis of Acute Bacterial Prostatitis

Benign prostatic hypertrophy Obstructive voiding symptoms, enlarged nontender prostate, negative urine culture Chronic bacterial prostatitis Recurring prostatitis symptoms for at least 3 months, positive urine culture with each episode Chronic syndrome Pain attributed to the prostate with no demonstrable evidence of infection Cystitis Irritative voiding symptoms, normal prostate exam http://www.aafp.org/afp/2016/0115/p114.html Differential Diagnosis of Acute Bacterial Prostatitis

Diverticulitis LLQ pain, acute change in bowel habits, hx of diverticuli, TTP in LLQ Epididymitis Irritative voiding symptoms, tenderness to palpation on affected epididymis Orchitis Swelling, pain and/or TTP in 1 or both testicles

Proctitis Tenesmus, rectal bleeding, feeling of rectal fullness, passage of mucus through the rectum Prostate Cancer Presence of constitutional symptoms, nodules on prostate exam http://www.aafp.org/afp/2016/0115/p114.html Acute Bacterial Prostatitis Rx

• Base empiric antibiotic on the suspected mode of infection & presumed infecting organism • Gram-negative enteric bacteria; Cover GC/CT if sexually active • Antibiotic choice (Level C) • Ceftriaxone/Doxycycline, Quinolones*, Bactrim • Ciprofloxacin 500 mg bid x 2-4 weeks • *Increasing gram negative enteric bacteria with quinolone resistance • Broad spectrum IV antibiotics if risk for antibiotic resistance • Rx Duration: mild infections is typically 10 to 14 days; four weeks for severe infections Key Recommendations for Practice

Not Recommended − Prostatic massage (SOR C) − PSA (SOR C) − Ignore fever lasting over 36 hours (SOR C)

http://www.aafp.org/afp/2016/0115/p114.html 3. A 62 yo M presents with inability achieve a satisfactory erection & increased fatigue for 6 months. Past medical history reveals obstructive sleep apnea, and BPH. Physical examination reveals decreased muscle mass and thinning hair. What is the most appropriate next diagnostic step? a) Confirm empiric use and efficacy of phosphodiesterase-5 inhibitors b) Obtain an LH level c) Obtain a prolactin level d) Obtain an early morning testosterone level e) Order a scrotal ultrasound 4. Which labs are most accurate in predicting primary hypogonadism? A. AM labs: Low testosterone, high FSH, high LH B. AM labs: Low testosterone, low FSH, low LH C. PM labs: Low testosterone, high FSH, high LH D. PM labs: Low testosterone, low FSH, low LH Symptoms of Male Hypogonadism

• Decreased libido • Erectile dysfunction • Decreased energy • Loss of muscle mass • Decreased sexual hair • Decreased bone density • Infertility Total testosterone Normal level Testosterone (AM draw) Diagnosing Low Male Testosterone

Hypogonadism Repeat testosterone Low T, Low Prolactin, MRI, (consider free), FSH, Secondary FSH, LH T4, TSH LH hypogonadism

Low T, High FSH, LH

Primary Hypogonadism Lab Testing for Hypogonadism

Testosterone Semen analysis Gonadotropins Diagnosis (LH/FSH)

Low Low sperm ct Elevated Primary hypogonadism Low Low sperm ct Low/low nl Secondary hypogonadism Causes of Male Hypogonadism

Primary Secondary Androgen Resistance

• Chromosomal • Pituitary tumors • 5-alpha reductase abnormalities • Infiltrative disease deficiency (Klinefelter syndrome • Inflammatory XXY) diseases Testosterone • Infections • Idiopathic X • Trauma hypogonadotropic • Cryptorchidism hypogonadism Dihydrotestosterone • Drugs • Kallmann Syndrome (DHT) • Chemotherapy • HIV/AIDS • Androgen receptor • Obesity abnormalities Goals of Treating Hypogonadism

•Restore sexual function, libido, well-being, and behavior •Produce and maintain virilization •Optimize bone density and prevent osteoporosis •In elderly men, possibly normalize growth hormone levels •Potentially affect the risk of cardiovascular disease •In cases of hypogonadotropic hypogonadism, restore fertility

Bhasin S, J Clin Endocrinol Metab. 2006;91:1995–2010 Treating Male Hypogonadism

ANDROGEN IMPROVING FERTILITY REPLACEMENT • Testosterone enanthate or • Stimulating spermatogenesis cypionate hCG, and hMG • Transdermal delivery GnRH Androgen 5 mg patch daily AndroGel 1% 5 mg daily Side Effects of Testosterone Rx

•Worsening of the prostatic •Swelling of ankles, feet, or body, hypertrophy with or without heart failure •Increased risk of prostate cancer •Gynecomastia •Lower sperm count with large •Sleep apnea doses •Blood clots Contraindications to Testosterone Therapy

  Risk adverse outcomes  Risk of adverse outcomes

• Prostate cancer • Undiagnosed prostate nodule • • Unexplained PSA elevation • BPH with severe urinary retention • Erythrocytosis • NYHA Class III or IV heart failure 5. A 46 yo M presents complaining of worsening erectile dysfunction. He denies history of depression, anxiety, CAD or peripheral vascular disease. His BMI is 32. He exercises 3 times weekly. A recent AM testosterone level was 310 ng/dL (normal range: 280 to 1,100 ng/dL). Which of the following is the most appropriate therapy?

a) Testosterone IM b) Testosterone Gel c) Trazodone d) A phosphodiesterase inhibitor e) Yohimbine Erectile Dysfunction = Small Vessel Dz

• ED is a robust predictor of all-cause mortality & CV events in men Hazard ratio for mortality = 2.04 Hazard ratio for CV event = 1.62

• “Dose-response” increase with ED severity • Bohm, Circulation, March 15, 2010 ED Rx begins with PDE5-Inhibitors

• Oral phosphodiesterase-5 (PDE5) inhibitors are 1st line. NNT = 2.1: Level A Rec, Cochrane, 2007 • Most effective for men with: DM, spinal cord dysfunction, & antidepressants side effects Level A Rec, Cochrane, 2007 • Helpful in nerve-sparing prostatectomy Level B Rec, Bandolier, 2005 • Efficacy & side effects similar among the 4. Drop-out rates  for sildenafil Level A Rec, Bandolier, 2005 Adverse Effects of PDE-5 Inhibitors

• Vision disturbances • Priapism • Angina • Sudden, permanent sensorineural hearing loss (May 2010) Other ED Treatments

• Testosterone • Vacuum devices Helpful if low T (<12 nmol/L) Level B Rec NNT = 2.1: Level A Rec

• Yohimbine • Alprostadil NNT = 6.4: Level A Rec NNT = 3.5; Not a 1st-line agent 2/2 side effects: Level A Rec Other ED Treatments cont.

• Trazodone: NOT USEFUL • Managing obesity (level B) • Level A Rec • BMI >30  ED risk • Losing weight improves • Fibrates & statins may ED contribute to ED • Level B Rec Key Recommendations for Practice

•First line therapy for ED is oral phosphodiesterase type 5 inhibitors (SOR A) •PDE type 5 inhibitors are most effective in the treatment of ED associated with DM, spinal cord injury and antidepressant side effects. (SOR A) •Psychosocial therapy and testosterone supplementation in men with hypogonadism are additional ED therapies (SOR B)

http://www.aafp.org/afp/2010/0201/p305.html Prostate Cancer

Source: National Cancer Institute USPSTF (2017)

• INFORM • INDIVIDUAL DECISION- MAKING https://screeningforprostatecancer.org/ AAFP (2018)

PSA-Based Prostate Cancer Screening in Men Aged 55-69 (C)

•The AAFP does not recommend routine prostate-specific antigen (PSA)-based screening for prostate cancer. For men ages 55-69 who are considering periodic prostate cancer screening, clinicians should discuss the risks and benefits and engage in shared decision-making that enables an informed choice.

https://www.aafp.org/patient-care/clinical-recommendations/all/prostate-cancer.html USPSTF (2017) & AAFP (2018)

➢ DON’T GET PSA.

https://screeningforprostatecancer.org/ https://www.aafp.org/patient-care/clinical-recommendations/all/prostate-cancer.html 6. A 49 yo F presents for evaluation of recurrent episodes of hematuria and flank pain. Spiral CT scan of the abdomen and pelvis reveals urolithiasis. She eventually passes the stone into a urine strainer. Analysis of the stone reveals calcium oxalate. Dietary recommendations would include:

a) Decrease in calcium b) Normal calcium diet c) Decrease in fiber intake d) Decrease in natural forms of citrate Key Recommendations for Practice

• Patients with kidney stones should increase fluid intake to at least 2L per 24 hours. (SOR B)

http://www.aafp.org/afp/2011/1201/p1234.html Kidney Stones – Risk Factors

RISK FACTORS SYMPTOMS

• Male Gender • Abdominal Pain • *Age (to 65) • Renal colic • Low urine volume • Sudden; not relieved • Situational; Geography • Radiating pain to back or groin • Heredity • Hematuria • Diet (high Na) • Medications Meds Associated w/ Stone Formation

• Triamterene and the sulfonamides (poor solubility) • Calcium and vitamin D supplements (hypercalciuria) • Carbonic anhydrase inhibitors (increases urinary pH, thereby increasing calcium phosphate precipitation) • Indinavir, a protease inhibitor, (urinary crystallization) • NOT HCTZ (Increase Calcium reabsorption) Renal Colic Suspected Imaging Options (based on hx/pe)

Pregnant, Gallbladder dz or Gyn Hx of radio-opaque Everyone else cause suspected renal calculi *A strategy beginning with ultrasonography (including point- of-care ultrasonography *Could consider performed by an emergency bedside US if physician) can help many not obese Ultrasound X-ray patients avoid CT and its associated radiation. (Level of Evidence = 1b–) IV Non- If Xray normal, but Pyelogram contrast (only if no CT you still have helical CT concerns for stone. available) http://www.aafp.org/afp/2015/0115/p132a.html Imaging modality Sensitivity (%) Specificity (%)

Non-contrast CT 95 to 100 94 to 96 Advantages Limitations

Most sensitive & specific radiologic Less accessible and test (i.e., facilitates fast, definitive relatively expensive diagnosis) No direct measure of renal Indirect signs of the degree of function. obstruction Provides information on non- genitourinary conditions <4 mm Management Options

~98% pass on their own in 1-2 wks

Analgesia 5-10 mm

Alpha-blockers (tamsulosin or >10 mm nifedipine) ~53% pass on their own unlikely to have any benefit Indication: for intervention: Strain urine • evidence of persistent obstruction UROLOGY F/U KUB q 1-2 wks • failure of stone progression Consult • persisting colic (SOR C) Urology if not passed in 2-4 wks ESWL or ureteroscopy RTC signs of sepsis Management Prevention (3 Principles) (of Recurrences)

• Recognize emergencies • Need to analyze calculi (hydronephrosis) • Adequate analgesia • Labs • Impact of size and location on Hx & Rx 7. A 55 yo M is noted to have incidental urolithiasis on plain films of the abdomen. He denies abdominal pain, nausea, vomiting and bowel changes. Abdominal examination is unremarkable. The most appropriate next step is?

a) Dissolution therapy with ursodiol b) Endoscopic retrograde cholangiopancreatography c) Watchful waiting d) Extracorporeal shockwave lithotripsy Hemataspermia

Post Procedural Sporadic • Prostate Biopsy (80%) • Prostatitis • Radiation (25%) • STI’s • Cancer (Men over 40) • Frequent daily ejaculation (over weeks) • Schistosomiasis Hematospermia

Evaluation Treatment •Persistent, unexplained sx • Reassurance for most patients should be evaluated by a •Treat identifiable structural transrectal ultrasonography. abnormalities • MRI is second line. • Emerging therapy: Finasteride • Exclude infection - get a UA. in patients with refractory sx •Rule-out prostatitis •Routine PSA is not recommended Thank you! Answers

1. B 2. B 3. D 4. A 5. D 6. B 7. C Bibliography

1. USPSTF. Screening for Prostate Cancer. 2012. http://www.uspreventiveservicestaskforce.org/Page/Document/UpdateSummaryFin al/prostate-cancer-screening (Topic is in process of being updated) 2. Coker TJ, Dierfeldt D. Acute Bacterial Prostatitis: Diagnosis and Management Am Fam Physician. 2016 Jan 15;93(2):114-120. http://www.aafp.org/afp/2016/0115/p114.html 3. Holt JD, Garrett WA, McCurry TK, Teichman JM. Am Fam Physician. Common Questions About Chronic Prostatitis. 2016 Feb 15;93(4):290-6. http://www.aafp.org/afp/2016/0215/p290.html 4. Pearson R, Williams PM. Common questions about the diagnosis and management of benign prostatic hyperplasia. Am Fam Physician. 2014 Dec 1;90(11):769-74. http://www.aafp.org/afp/2014/1201/p769.html 5. Sharp VJ, Barnes KT, Erickson BA. Assessment of asymptomatic microscopic hematuria in adults. Am Fam Physician. 2013 Dec 1;88(11):747-54. http://www.aafp.org/afp/2013/1201/p747.html Bibliography

1. Holt JD, Garrett WA, McCurry TK, Teichman JM. Common Questions About Chronic Prostatitis. Am Fam Physician. 2016 Feb 15;93(4):290-6. http://www.aafp.org/afp/2016/0215/p290.html 2. Ebell M. Ultrasonography Is Best Initial Imaging for Suspected Kidney Stones. Am Fam Physician. 2015 Jan 15;91(2):132a-133. http://www.aafp.org/afp/2015/0115/p132a.html 3. Frassetto L, Kohlstadt I. Treatment and prevention of kidney stones: an update. Am Fam Physician. 2011 Dec 1;84(11):1234-42. http://www.aafp.org/afp/2011/1201/p1234.html 4. Mulhem E, Fulbright N, Duncan N. Prostate Cancer Screening. Am Fam Physician. 2015 Oct 15;92(8):683-8. http://www.aafp.org/afp/2015/1015/p683.html 5. Crawford P, Crop JA. Evaluation of scrotal masses. Am Fam Physician. 2014 May 1;89(9):723-7. http://www.aafp.org/afp/2014/0501/p723.html 6. Heidelbaugh JJ. Management of erectile dysfunction. Am Fam Physician. 2010 Feb 1;81(3):305-12. http://www.aafp.org/afp/2010/0201/p305.html Bibliography

1. Stevermer JJ, Easley SK. Treatment of Prostatitis. AFP. 2000;61:3015-22. 2. NJAFP. CME Report: Diagnosis & Management of Overactive Bladder in Family Medicine. 2007; 2:1-14. 3. Stendardo S. Caring for Patients Who have BPH. AAFP CME Bulletin. 2007;6:1- 6. 4. Ciftci, AO. Clinical Predictors for Diff. Diagnosis of Acute Scrotum, European J. of Ped. Surgery. Oct 2004. 5. Luzzi GA. Acute Epididymitis. BJU International. May 2001. 6. Fleisher G, Ludwig S, Henretig F. Textbook of Pediatric Emergency Medicine. 2006. 7. Sudakoff GS, et al., “Multidetector CT Urography as the Primary Imaging Modality for Detecting Urinary Tract Neoplasms in Patients with Asymptomatic Hematuria,” J Urol. 2008 March, 179(3). Bibliography

9. Jang T, Schaeffer A. Chronic Prostatitis. (Clinical Evidence Concise). AFP. Aug 1, 2005. 10. Amer. Cancer Soc. Guideline for the Early detection of prostate cancer: Update 2010. Cancer J Clin. 2010. 11. Wilbur J. Prostate Cancer Screening. Am Fam Physician. 2008;78:1377-84. 12. Edwards J. Diagnosis & Management of Benign Prostatic Hyperplasia. Am Fam Physician. 2008;77:1403-10. 13. JFP. June 2012; vol 61:S1-S10. 14. Beers MH, et al., Merck Manual of Diagnosis and Therapy (18th print and online editions), “Chapter 226: Approach to the Genitourinary Patient: Isolated Hematuria.” 15. Cohen RA and Brown RS, “Microscopic Hematuria,” New England Journal of Medicine. 348:23, 5 June 2003. 16. Sharp VJ et al. Testicular Torsion. Am Fam Phys. 2013;88:835-40. 17.Ferre RM et al. Tamsulosin for ureteral stones in the emergency department: A randomized, controlled trial. Ann Emerg Med 2009 Sep; 54:432. Supplementary Slides The Take-Home Points for Hematuria

• Positive dipsticks for blood should get microscopic confirmation • R/O myo- or hemoglobinuria and decide glomerular vs. non-glomerular. • Top 3 suspects are infection, stones, and malignancy. • Look for illness patterns: • Unilateral flank pain, afebrile, N/V (stones) • Obstructive Sxs, fever, prostate tenderness (prostatitis) • CVAT, fever, dysuria (pyelo) • If no easy answer, ask: Glomerular or not? • Glomerular - protein or renal dz? If so, refer to nephrology. • Not - 1. CT-U; 2. Cytology; 3. Cystoscopy Age and Hematuria

Age (yr) Common Uncommon

Glomerulopathy (IgA, Alport syndrome, thin BM Factitious disease, APSGN) Fever Hypercalciuria with stones HUS 0-15 Congenital obstructive anomalies UTIs Hemophilia Sickle cell disease HSP Viral infection Schistosomiasis Calculi AVMs or fistulae Menstrual contamination DIC Exercise Goodpasture’s syndrome UTIs Loin pain-hematuria syndrome 15-50 PKD Renal infarction Sickle cell disease Renal vein thrombosis Intercourse Schistosomiasis Papillary necrosis Medullary sponge kidney AVMs or fistulae BPH Cyclic hematuria in women Cancer (renal, ureteral, bladder, prostate) Endometriosis >50 Overanticoagulation TTP PKD Renal vein thrombosis Prostatitis Toxins (cantharidin, djenkol bean) LP-HS Most recent AFP review article on BPH: http://www.aafp.org/afp/2014/1201/p769.html Chronic Bacterial Prostatitis

Updated AFP article: http://www.aafp.org/afp/2016/0215/p290.html

• Source of recurrent UTIs

• Similar Sx as Acute with ASx intervals

• WBCs + on pre- & post-massage UAs

• C&S neg on pre- & + on post-massage Chronic Bacterial Prostatitis

• Most (&5-80%) due to E. coli • Treatment is 6-12 weeks • Alpha blockers may help in RX • If man does not improve, consider a prostatic calculus Chronic Bacterial Prostatitis Rx

• TMP-SMX as 1st line: Level C Rec

• Quinolone for Rx failures: Level C Rec

• Rarely: TUP of infected tissue for very Sx complete failures on Abx Acute Infectious Epididymitis

• Treat men with presumptive Dx of acute infectious epididymitis for chlamydia and gonorrhea if < 35 y.o., and Rx for enteric UTI pathogen if > 35 y.o. (SOR A) • Epididymotis/orchitis should be suspected in patients with testicular pain and a C-reactive protein > 24 mg/dl (SOR C) Kidney Stone Labs All Patients

• CBC • Urate • UA • Urine C & S • BMP • Stone Analysis • Ca • Vitamin D • PO4 Kidney Stone Labs Circumstantial • Hypercalcemia: PTH • Abnormal Albumin: Ionized Calcium • Hyperoxaluria: Oxalate level • Sarcoidosis: ACE level & calcitriol More Extensive Testing

• Children • Solitary kidney • CRI/CKD • Residual stone burden • Infected stones • Gout • Intestinal disease Prevention

• All patients : 2-3 L water q day, 8-12 oz QHS (urine volume = 2 L/day) • “B rec”

• NaCL (2g) ‘C’ Rec unless • Animal protein (8 oz) dietary excesses • Oxalate • Calcium in diet ‘B’ rec (to 1200 mg/day) Causes of Scrotal Pain and Swelling Updated Scrotal Mass article: http://www.aafp.org/afp/2014/0501/p723.html

• Pain • Swelling Testicular torsion Hydrocele Torsion of appendix testis Varicocele Epididymitis Spermatocele Trauma Tumor Orchitis and others http://www.aafp.org/afp/2014/0501/p723.html Epidemiology

• Accounts for 30% of all acute scrotal swelling • Bimodal ages – neonatal (in utero) and pubertal ages 65% occur in ages 12-18yo • Incidence 1 in 4000 in males < 25 yo • Increased incidence in puberty due to inc weight of testes Predisposing Anatomy

•Bell-clapper deformity • Testicle lacks normal attachment at vaginalis • Increased mobility • Transverse lie of testes • Typically bilateral • Prevalence 1/125 Torsion: Clinical Presentation

• Abrupt onset of pain – usually testicular, can be lower abdominal, inguinal • Often < 12 hrs duration • May follow exercise or minor trauma • May awaken from sleep Cremasteric contraction with nocturnal stimulation in REM • Up to 8% report testicular pain in past • May have N&V Torsion: Examination

• Edematous, tender, swollen • Elevated from shortened spermatic cord Horizontal lie common (PPV 80%) Reactive hydrocele may be present • Cremasteric reflex absent in nearly all (unreliable in < 30 mo old) (PPV 95%) • Prehn’s sign (elevation relieves pain in epididymitis and not torsion) is NOT reliable Intermittent Torsion

• Intermittent pain/swelling with rapid resolution (seconds to minutes) • Long intervals between symptoms • PE: testes with horizontal lie, mobile testes, bulkiness of spermatic cord (resolving edema) • Often evaluation is normal – if suspicious need GU follow-up Diagnosis – “Time is Testicle”

• Ideally -- prompt clinical diagnosis • Imaging: 2ndary to clinical exam Color doppler – decreased intratesticular flow • False + in large hydrocele, hematoma • Sens 69-100% and Spec 77-100% • Lower sensitivity in low-flow pre-pubertal testes • Nuclear technetium-99 radioisotope scan Show testicular perfusion 30-min procedure time Sens and spec 97-100% Management

• Detorsion within 6hr = 100% viability Within 12-24 hrs = 20 – 50 % viability After 24 hrs = 0 - 10% viability • Surgical detorsion and orchiopexy if viable Contralateral exploration and fixation if bell-clapper deformity • Orchiectomy if non-viable testicle • Never delay surgery on assumption of nonviability, as prolonged symptoms can represent periods of intermittent torsion Manual Detorsion

• If presents before swelling • Appropriate sedation • In 2/3rds of cases testis torses medially, 1/3rd laterally • Success if pain relief, testes lower in scrotum • Still need surgical fixation Torsion of Appendix Testis

• Peak age 3-13 yo (prepubertal) • Sudden onset, pain less severe • Classically, pain more often in abd or groin • Non-tender testicle Tender mass at superior or inferior pole • May be gangrenous, “blue-dot” (21% of cases) • Normal cremasteric reflex, may have hydrocele • Inc or normal flow by doppler U/S Torsion of Appendix Testis

• Management supportive Analgesics, scrotal support to relieve swelling • Surgery for persistent pain No need for contralateral expl. Epididymitis

• Inflammation of epididymis • Subacute onset pain, swelling localized to epididymis, duration of days With time, swelling and pain less localized • Testis has normal vertical lie • Systemic signs of infection Inc WBC and CRP, fever + in 95% • Cremasteric reflex preserved • Urinary complaints: discharge/dysuria PPV 80% Epididymitis

• Sexually active males Chlamydia > N. gonorrhea > E. coli • Less commonly pseudomonas (elderly) and tuberculosis (renal TB) • Young boys, adolescents often post-infectious (adenovirus) or anatomic Reflux of sterile urine through vas into epididymis 50-75% of prepubertal boys have anatomic cause by imaging Epididymitis Diagnosis

• Leukocytosis on UA in ~40% of patients • PCR Chlamydia + in 50%, GC + in 20% of sexually active • 95% febrile at presentation • Doppler and nuclear imaging show increased flow • If Hx consistent with STD, CDC recommends: Cx of urethral discharge, PCR for C and G Urine culture and UA Syphilis and HIV testing Epididymitis Treatment

• Sexually active treat with ceftriaxone/ doxycycline or ofloxacin • Pre-pubertal boys Treat for co-existing UTI if present Symptomatic Tx with NSAIDs, rest Referral all to GU for studies to rule out VUR, post urethral valves, duplications • Negative culture has 100% NPV for anomaly © 2020 American Academy of Family Physicians. All rights reserved.

All materials/content herein are protected by copyright and are for the sole, personal use of the user.

No part of the materials/content may be copied, duplicated, distributed or retransmitted

in any form or medium without the prior permission of the applicable copyright owner.

Maternity Care I

David G. Weismiller, MD, ScM, FAAFP Department of Family and Community Medicine University of Nevada, Las Vegas School of Medicine Disclosure Statement

It is the policy of the AAFP that all individuals in a position to control content disclose any relationships with commercial interests upon nomination/invitation of participation. Disclosure documents are reviewed for potential conflicts of interest. If conflicts are identified, they are resolved prior to confirmation of participation. Only participants who have no conflict of interest or who agree to an identified resolution process prior to their participation were involved in this CME activity.

All individuals in a position to control content for this session have indicated they have no relevant financial relationships to disclose. Learning Objectives

1. Cite the content and sequence of routine prenatal care. 2. Recall the laboratory and imaging tests commonly used in prenatal care. 3. Define the diagnosis of diabetes in pregnancy.

1. Nausea and vomiting in early pregnancy is very common. Although often called “morning sickness,” it can any time of the day. Which one of the following is considered first-line pharmacotherapy?

A. Diphenhydramine, 25–50 mg orally every 4–6 hours B. Ondansetron, 4 mg orally every 8 hours

C. Vitamin B6, 10–25 mg orally in combination with Doxylamine, 12.5 mg orally every 6 to 8 hours D. Methylprednisolone, 16 mg orally every 8 hours Nausea and Vomiting of Pregnancy

• Etiology – Unknown • Prevalence – Nausea 50-80% – Vomiting and retching 50% • Recurrence 15-81% • Typically starts within four to eight weeks of gestation and is expected to subside between 13 and 14 weeks – Perception of the severity of her symptoms, her desire for treatment, and the potential effect of treatment on her fetus all influence clinical decision making • Early treatment of nausea and vomiting of pregnancy may be beneficial to prevent progression to hyperemesis gravidarum Nonpharmacologic Therapy • Dietary measures – Eating frequent small meals and avoid smells and food that cause nausea – Bland food, high in carbs, low in fat (salty foods can be tolerated in the morning, sour and tart liquids are tolerated better than water) • Emotional support – Although not strongly associated with psychologic illness, some may become depressed or have other affective changes Management

• Convert prenatal vitamin to folic acid 1 supplement only

• Ginger capsules 250 mg QID 1 Persistent symptoms

• Consider P6 acupressure with wrist bands 1 Pharmacologic Options

• Vitamin B6 (pyridoxine) 10–25 mg po (alone or in combination with Doxylamine 12.5 mg po), TID to QID. Adjust schedule/dose according to severity of patient’s symptoms OR – Doxylamine is available as the active ingredient in some OTC sleep aids; one half of a scored 25-mg tablet can be used to provide a 12.5-mg dose of doxylamine.

• Vitamin B6 10 mg/Doxylamine 10 mg combination product, two tablets po at bedtime initially, up to four tablets per day (one tablet in the morning, one tablet in midafternoon, and two tablets hs) OR

• Vitamin B6 20 mg/Doxylamine 20 mg combination product, one tablet po at bedtime initially, up to two tablets per day (one tablet in the morning and one tablet hs.) 2. Which one of the following tests is recommended in routine prenatal care?

A. Serologic testing for herpes simplex virus infection B. Pap cytology in each pregnancy C. Cystic fibrosis carrier screening before conception D. Screening for asymptomatic bacteriuria by urine culture Initial Prenatal Visit Lab Tests

• Blood group/Rh • Urine culture (11-16 weeks EGA) • Antibody screen − Asymptomatic bacteriuria • H/H (Screening for anemia) • Sickle cell screen • Rubella antibody titer − African American or Caribbean − 2001 – CDC. Interval to 1 month descent for pregnancy after rubella vaccine • HIV screening • Syphilis screen • Varicella screening − CDC and USPSTF – three times if • +/− CF screening high risk • GDM screening • Hepatitis B virus surface antigen − Intake for Type 2 DM (if increased • Cervical cytology (if needed) risk) ??? • STI − Routine 24-28 weeks − GC/chlamydia Anemia in Pregnancy ACOG Practice Bulletin – July 2008 • Iron supplementation decreases the prevalence of anemia at delivery (Level A) • Iron deficiency anemia during pregnancy has been associated with an increased risk of (Level B): − LBW − Preterm delivery − Perinatal mortality • All pregnant women should be screened for anemia (Level C) − (+) iron deficiency anemia (Hgb < 11 mg/dL (first trimester), < 10.5 (second), < 11 (third): treat with supplemental iron, in addition to prenatal vitamin. Obstetrics and Gynecology. 2008;112(1) 201-207. (Affirmed 2015) USPSTF - 2015

• The USPSTF concludes that the current evidence is insufficient to assess the balance of benefits and harms of screening for iron deficiency anemia in pregnant women to prevent adverse maternal health and birth outcomes. • The USPSTF concludes that the current evidence is insufficient to assess the balance of benefits and harms of routine iron supplementation for pregnant women to prevent adverse maternal health and birth outcomes. USPSTF Recommendations for STI Screening in Pregnancy

• Hepatitis B, Syphilis, HIV − All women HIV Testing

• CDC 2006, 2008; ACOG 2008; USPSTF 2005, 2019 − All pregnant women offered HIV testing at the first prenatal visit and in the third trimester • Routine opt-out HIV testing • Decreased maternal-fetal transmission with detection and treatment • Rapid testing in labor − if undocumented HIV status − Positive test − initiate antiretroviral prophylaxis (with consent) without waiting for confirmatory test.

Centers for Disease Control and Prevention. US Public Health Service Task Force’s Recommendations for Use of Antiretroviral Drugs in Pregnant HIV-1-Infected Women for Maternal Health and Interventions to Reduce Perinatal HIV-1 Transmission in the United States, July 2008. CDC. Revised recommendations for HIV testing of adults, adolescents, and pregnant women in health care settings. MMWR. 2006;55(RR-14):1-17. USPSTF. Screening for HIV: Recommendation statement. Ann Intern Med. 2005; 143:32-37. ACOG Committee Opinion Number 418, September 2008. USPSTF Recommendations for STI Screening in Pregnancy • Hepatitis B, Syphilis, HIV − All women • Chlamydia§ and Gonorrhea − < 25 years of age − Engaging in high-risk sexual behaviors • Multiple sex partners • Incarcerated • Illicit drug use • Does not recommend routine screening for chlamydia in pregnant women not at increased risk; it notes that individual circumstances may support screening. • No recommendation made about screening for gonorrhea in pregnant women not at increased risk (Category I)

§CDC recommends universal testing for chlamydia in pregnancy Asymptomatic Bacteriuria

• Defined − > 100,000 cfu/mL of a single bacterial species • E. coli is most commonly isolated. • Lactobaccilli and Staphylococcus species (other than S. saprophyticus) may be presumed to be contaminants. • Present in 2%-7% of pregnant women − Higher preterm delivery rate than women without bacteriuria • Culture at first prenatal visit − Unusual for women who do not have Asx bacteriuria at initial visit to develop later in pregnancy − Exception? Sickle cell trait • q trimester urine screen 3. Which of the following antibiotics should be used in the empiric treatment of asymptomatic bacteriuria?

A. Ampicillin B. Clindamycin C. Cephalexin D. Ciprofloxacin Treatment • Seven-day course − e.g., cephalexin, 250 mg po QID • Single-dose regimens not widely studied in pregnancy • Ampicillin should not be used − high rates of resistance. • Urine culture after therapy to ensure cure − Culture will also identify patients with persistent or recurrent bacteriuria. • Consider antibiotic suppressive dose (once daily antibiotic dosing) until delivery Antibiotics in Pregnancy

• Considered Safe • Typically Avoid • Amoxicillin • Tetracyclines • Ampicillin • Nitrofurantoin* • Clindamycin • Sulfonamides* • Erythromycin • Penicillin • Cephalosporins * Prescribing sulfonamides or nitrofurantoin in the first trimester is still considered appropriate when no other suitable alternative antibiotics are available. During the second and third trimesters, sulfonamides and nitrofurantoin may be used as first-line agents for infections.

ACOG Committee Opinion #494: Sulfonamides, Nitrofurantoin, and Risk of Birth Defects. Obstet Gynecol. 2011;117(6):1484-1485. Treatment of Asymptomatic Bacteriuria

Antibiotic treatment Outcome compared to placebo RR NNT (95% CI) Clearing asymptomatic 0.25 (0.14-0.48) 1.6 bacteriuria Incidence of pyelonephritis 0.23 (0.13-0.41) 6.7 Incidence of low 0.66 (0.49-0.89) − birthweight babies Incidence of preterm − 20 delivery Antibiotics for asymptomatic bacteriuria in pregnancy. Cochrane Database of Systematic Reviews. 2001;(2):CD000490. Herpes

• All patients and their partners should be asked about a history of genital and orolabial HSV infection. • Rates of vertical transmission at delivery − Primary HSV infection 50% − Non-primary – first episode 33% − Recurrent 0-3% • Genital herpes acquired during pregnancy does not seem to increase rates of neonatal illness or congenital HSV infection as long as HSV seroconversion has completed by time labor begins. Suppressive Therapy > 36 Weeks ACOG 2007 • Women with active recurrent genital herpes should be offered suppressive viral therapy at or beyond 36 weeks of gestation (Level B).

Suppressive treatment after 36 weeks Decreased Risk of recurrent genital herpes at delivery 75% Rate of C/S for recurrent genital herpes 40% Viral detection at delivery using culture or PCR 90%

Indication Acyclovir Valacyclovir Daily suppression 400 mg po TID daily from 36 500 mg po BID daily from 36 weeks until delivery weeks until delivery Evidence of Immunity to Varicella

• Screening for Varicella in pregnancy (CDC 2006) − Recommend routine screening for varicella through history − If negative/unsure history, obtain varicella titer. − Recommend offering vaccination postpartum, if varicella is nonimmune. Cystic Fibrosis Recommendations

• Information about CF screening made available to all couples − Reasonable to offer carrier screening to all couples regardless of race or ethnicity as alternate to selective screening • CF carrier screening should be offered before conception or early in pregnancy − both partners are of Caucasian, European, or Ashkenazi Jewish ethnicity. − May elect to use either sequential or concurrent carrier screening

ACOG Committee Opinion No. 325. Update on Carrier Screening for Cystic Fibrosis. Obstet Gynecol. 2005;106:1465-1468. No Routine Screening

• Hepatitis C − CDC – screen women at increased risk • HPV − Per USPSTF guidelines • < 30 – reflex (ASCUS) q 3 years • > 30 – routine with Pap q 5 years • Parvovirus B-19 Parvovirus B-19 • Embryotoxic, not teratogenic − Infection 1st trimester – miscarriage − Peaks early 2nd trimester – fetal anemia, hydrops fetalis, and stillbirth • Fetus at greatest risk 3-6 weeks after maternal infection − Screening: Maternal IgG and IgM • Fetal monitoring: Weekly U/S and middle cerebral artery Doppler − If hydrops and anemia are present, RBC transfusion may be needed Intimate Partner Violence USPSTF – October 2018

• Screen all women of childbearing age for intimate partner violence regardless of whether symptoms are present. Grade: B recommendation −For women who screen positive, physicians should provide or refer them to intervention services. −Note: Men and women with a h/o IPV are at risk for depression (SOR B) Intimate Partner Violence • Risk assessment during pregnancy should universally include identification of women who are victims of domestic violence • May involve threatened or actual −Physical −Sexual −Verbal −Psychologic Screen at 8, 24, and Screening Questions 32 weeks EGA

Within the last year, have you been hit, slapped, kicked, or otherwise physically hurt by someone?

Since you have been pregnant, have you been hit, slapped, kicked, or otherwise physically hurt by someone?

Within the last year, has anyone forced you to have sexual activities?

If the screen is (+), follow appropriate medical/legal mandates for reporting requirements for state/branch of service. 4. Per ACOG 2013, at 24 weeks gestation you perform a 1-hour glucose tolerance test, which is elevated at 166 mg/dL. You would next:

A. Repeat the 1-hour glucose tolerance in one week. B. Have the patient see the dietician for diabetic diet information. C. Order a 3-hour glucose tolerance test. D. Start low-dose metformin. Gestational Diabetes

• Condition is increasing as obesity and older age at pregnancy become more common. • Increased risk: − Gestational hypertension − Preeclampsia − Cesarean delivery − 7-fold increased risk of developing diabetes later in life Detection of GDM

Organization Recommendation Comments ACOG (2018) • Use a 2-step method at 24-28 weeks (Level Use a blood glucose level of either 135 B). or 140 mg/dL with factors such as • Screen for undiagnosed type 2 diabetes at community prevalence rates of GD the first prenatal visit in those with risk factors determining the cutoff. USPSTF (2014) • Screen asymptomatic women after 24 weeks Goal was not to look at the performance (Grade B). or whether one method was better than another for screening. • Current evidence is insufficient to assess the Found treating can significantly reduce balance of benefits and harms of screening the risk of preeclampsia, macrosomia, for GD in asymptomatic pregnant women shoulder dystocia. before 24 weeks of gestation (Grade I). ADA (2014) • Screen for undiagnosed type 2 diabetes at Updated Guidelines: Use either: the first prenatal visit in those with risk factors 1. 1-step method (75g OGTT) • Screen at 24-28 weeks if not previously 2. 2-step method known to have diabetes. RISK FACTORS (Screen early)…ACOG, NIDDK,ADA • Patient is overweight with BMI of 25 (23 in Asian Americans), and ONE of the following: −Physical inactivity −Known impaired glucose metabolism −Previous pregnancy history of • GDM • Macrosomia (>4000 g) • Stillbirth −Hypertension (140/90 or being treated) −HDL cholesterol < 35 mg/dL −Fasting TG > 250 −PCOS, acanthosis nigricans, nonalcoholic steatohepatitis, morbid obesity and OTHER conditions associated with insulin resistance −HgbA1C>5.7%, impaired glucose tolerance or impaired fasting glucose −ASCVD

−High risk ethnicity ACOG Practice Bulletin No. 190: Gestational Diabetes Mellitus. Obstet Gynecol. 2018;131(2):e49-e64. Two-Step Approach in USA ACOG 2018, ADA 2014 • 24-28 weeks (routine) • Initial screening: 50 g oral glucose load (glucose challenge test) • > 135 or > 140 mg/dL*  3-hour OGTT • Note: > 190, > 90% abnormal 3-hour • 3-hour OGTT ±  2 or more abnormal values = (+) GDM • Overnight fast, 100 g glucose polymer • Abnormal plasma blood glucose: > fasting 95 mg/dL, 1h 180, 2h 155, 3h 140 *Either threshold acceptable, ACOG 2018 (Level C). ± Can also be used as a 1-step method for high-risk women or in areas in which the prevalence of insulin resistance is 5% or higher (eg, southwestern and southeastern US). Screening Later in Pregnancy

• Open neural tube defects − Maternal serum alpha fetoprotein • Aneuploidy − Quad screen • Screening sonogram • Anemia • Group B Streptococcus Summary MSAFP and Quad Screen • Maternal blood sample (Calculated risk) − High MSAFP: Open neural tube defect − Low • Trisomy 21 • Low MSAFP, high hCG, low estriol, high DIA • Trisomy 18 • Low levels of all four analytes • Most accurate when performed between 16 and 18 weeks EGA • Most common reason for false positive is incorrect assignment of EGA Detection Rates Test* Markers Term risk Sensitivity Specificity Positive cutoff (%) (%) Predictive Value (%) First Nuchal translucency, free 1 in 325 83 95 4 Trimester β-hCG, PAPP-A§, Screening maternal age Quadruple Unconjugated estriol; α- 1 in 385 77 95 2 Screening fetoprotein, free β-hCG, (Second Inhibin A, maternal age trimester) Integrated Nuchal translucency, free 1 in 200 87 98 10 Screening β-hCG, PAPP-A, maternal (First and age, Unconjugated estriol; Second α-fetoprotein, free β-hCG, trimesters) Inhibin A, maternal age * All screening tests have a positive rate of ~5% (most of which are false positives) and a detection rate of 69-87%. §PAPP-A: pregnancy-associated plasma protein A Comparison of Traditional and Cell-Free Screening Tests Traditional Aneuploidy Screening Cell-free DNA Screening Recommended for primary screening Primary or secondary screening modality • ACOG and SMFM recommend as first-line • As primary screening, may be more screening for low-risk pregnancies appropriate for high-risk pregnancies • Should not be performed concurrently with, or • May be offered as a secondary screening test after, cell free DNA screening if traditional screening is abnormal Detection rate 80% for trisomy 21 and trisomy 18 Detection rate 99% for trisomy 21, 96% for with EITHER first-or second-trimester screening. trisomy 18, and approximately 90% for trisomy Detection rate about 94% for Trisomy 21 and 13 and monosomy X 93% for trisomy 18 with integrated screening. Accurate GA assessment is essential for Gestational age is not used to calculate results calculating the screening result, and screening Screening may be performed at any point must be performed within specific GA. beyond 9–10 wk of gestation US minor markers (soft signs) are often used to Ultrasound minor markers (soft signs) are not modify the aneuploidy risk. used to modify the aneuploidy risk 5. According to the CDC, all women of childbearing age should consume how much folic acid on a daily basis?

A. 4 mg B. 0.4 mg C. 1 mg D. 0.8 mg Folic Acid Supplementation

• All women of childbearing age: 0.4 mg/day (1992 CDC) − Periconceptional use: Estimated to reduce risk of NTD by 70%-80% • History NTD: 4 mg/day − 1-3 months before, continue through first 3 months of pregnancy. • Note: IOM recommends 600 mcg/day for adult women aged 19 and older and 800 mcg daily for adolescents aged 14-18 years.

Wald et al. Quantifying the effect of folic acid. Lancet. 2001;358:2069-73. Folic Acid Supplementation With Epilepsy • 4 mg/day − Valproate − Carbamazepine • 800 μg/day − Other AEDs • 1-3 months prior to conception and continue through first trimester. ✔Low serum folate levels in women with epilepsy are independently associated with an increased risk of major fetal malformations. ✔Not conclusively determined if folic acid supplementation prevents NTDs in women receiving antiepileptic drugs (AEDs). 6. According to ACOG, which of the following statements best characterizes the use of sonography in pregnancy?

A. It has the ability to diagnose minor fetal anomalies. B. It is best carried out at 12 weeks estimated gestational age. C. It is an accurate method of determining placental location. D. Routine use is recommended. Screening Ultrasound Defined

• Sonographic assessment of the pregnant abdomen between 16 and 20 weeks EGA (second trimester), in the absence of specific indications for a second- trimester ultrasound. Screening Ultrasound The basic sonographic examination should provide the following information: • Fetal number • Fetal presentation • Documentation of fetal life • Placental location • Fetal biometry • Assessment of amniotic fluid volume • Assessment of gestational age • Survey of fetal anatomy for gross malformations • Evaluation for maternal pelvic masses Ultrasound in Pregnancy Summary – ACOG February 2009 • US examination is an accurate method of determining GA, fetal number, viability, and placental location (Level A). • Ability to diagnose major fetal anomalies established (Level A) • Specific indications are the best basis for use in pregnancy. ACOG − Optimal timing in the absence of specific indications is at 18- ACR 20 weeks. AIUM ACOG Practice Bulletin No. 101. Ultrasopnography in Pregnancy. Obstet Gynecol. 2009;113:451-61. 7. An 18 yo G1 P0 at 40 5/7 weeks presents stating that she thinks her water broke 18 hours ago. An exam confirms ROM. The patient is afebrile with a non-tender uterus; FHR tracing reassuring. She is NOT contracting. A GBS culture obtained 4 weeks ago was (−). Patient with NKDA. In addition to induction of labor, which one of the following is the most appropriate management for this patient?

A. No antibiotic prophylaxis for the fetus B. Ampicillin, 2 g IV initially, then 1 g IV q 4 h until delivery C. Clindamycin, 900 mg IV q 8 h until delivery D. Vancomycin, 1 g IV q 12 h until delivery Group B Streptococcus

• Universal screening culture recommended 36-0/7 and 37-6/7 weeks (NEW July 2019 ACOG and AAP) − Provides a five-week window for valid culture results that includes births that occur up to a gestational age of at least 41-0/7 weeks − Swab lower (2 cm) vaginal (introitus) followed by rectum (insert swab into anus 1 cm) using same swab • Two groups DO NOT get screened − Previous infant with early onset GBS Sepsis − GBS bacteriuria (any culture, any number of cfu/mL) during pregnancy • Order susceptibility testing on isolates (if PCN allergic) • Women who present in labor with unknown GBS status but who had known GBS colonization during a previous pregnancy are now candidates for GBS intrapartum prophylaxis. (NEW July 2019 ACOG and AAP) − Rationale for change - women who were GBS-colonized during a previous pregnancy have a 50% chance of GBS carriage in the current pregnancy • Women with negative GBS cultures within 5 weeks of delivery do not require antibiotics even if risk factors develop ✔Delivery at < 37 weeks of gestation ✔Amniotic membrane rupture > 18 hours ✔Intrapartum temperature > 100.4°F (> 38.0°C) Prevention of Group B Streptococcal Early-Onset Disease in Newborns. ACOG Committee Opinion Summary No.782. Obstet Gynecol. 2019;134(1):206-210 Prevention of Group B Streptococcal Early-Onset Disease in Newborns: ACOG Committee Opinion Summary, Number 782. Obstetrics & Gynecology. 134(1):206-210, July 2019. DOI: 10.1097/AOG.0000000000003335

• Two hours of antibiotics exposure has been shown to reduce GBS vaginal colony counts and decrease the frequency of a clinical neonatal sepsis diagnosis • Shorter duration of recommended intrapartum antibiotics is less effective than four or more hours of prophylaxis

[ Figure 3]. Determination of Antibiotic Regimen for Group B Streptococcus Prophylaxis in Labor. Abbreviations: GBS, group B streptococcus; IV, intravenous. *Individuals with a history of any of the following: a non-urticarial maculopapular (morbilliform) rash without systemic symptoms; family history of penicillin allergy but no personal history; nonspecific symptoms such as nausea, diarrhea, yeast vaginitis; patient reports history but has no recollection of symptoms or treatment. +Individuals with a history of any of the following after administration of a penicillin: urticarial rash (hives), intense pruritis, anaphylaxis, angioedema, laryngeal edema, respiratory distress, hypotension, immediate flushing or rare delayed reactions, such as eosinophilia and systemic symptoms/drug-induced hypersensitivity syndrome, Stevens-Johnson syndrome, or toxic epidermal necrolysis. Individuals with recurrent reactions, reactions to multiple beta-lactam antibiotics, or those with positive skin testing also are considered high risk. Modified from Verani JR, McGee L, Schrag SJ. Prevention of perinatal group B streptococcal disease: revised guidelines from CDC, 2010. Division of Bacterial Diseases, National Center for Immunization and Respiratory Diseases, Centers for Disease Control and Prevention (CDC). MMWR Recomm Rep 2010;59(RR-10):1-36. (This Committee Opinion, including Table 1, Box 2, and Figures 1-3, updates and replaces the obstetric components of the CDC 2010 guidelines, "Prevention of Perinatal Group B Streptococcal Disease: Revised Guidelines From CDC, 2010.")

3 8. A 26 yo female presents with lower abdominal pain and vaginal bleeding. Her last menstrual period was 7 weeks ago. A urine pregnancy test is positive, and a quantitative β-hCG level is 2500 mIU/mL. Transvaginal ultrasonography shows no evidence of an intrauterine gestational sac. Baseline laboratory tests, including a CBC, liver function tests, and renal function tests, are all normal. She is treated with a single dose of IM methotrexate at 50 mg/m2 of body surface. Four days later the patient presents for reevaluation, and her quantitative β-hCG level is found to be 3800 mIU/mL. 7 days later the quantitative β-hCG level is found to be 4400 mIU/mL. Which of the following is the most appropriate next step?

A. A repeat dose of methotrexate, 50 mg/m2 of body surface B. Repeat transvaginal ultrasonography to evaluate for a viable intrauterine pregnancy. C. Laparoscopy with salpingostomy D. Expectant management Ectopic Pregnancy

• Ruptured ectopic pregnancy accounts for ~10% of all maternal deaths • Most strongly associated risk factors with ectopic pregnancy: ✔Previous tubal surgery ✔Sterilization ✔Prior ectopic pregnancy • Early diagnosis critical: ✔Linear salpingostomy is a tube-saving procedure. ✔Pregnancy outcomes with methotrexate are comparable to those with surgery Ectopic Risk Factors Risk Factor Odds Ratio Previous tubal surgery 21.0 Sterilization 9.3 Prior ectopic pregnancy 8.3 Current use of IUD 5.0 History of PID 3.4 Infertility > 2 years 2.7 Advanced maternal age • > 40 years 2.9 • 35-39 years 1.4 Smoking • > 1ppd 3.9 • 10-19 cigarettes 3.1 • < ½ ppd 1.7 • Prior smoker 1.5 Diagnosis

• Present with abdominal pain and vaginal bleeding; typically ~7 weeks after LNMP • Ultrasound: Diagnostic test of choice − Suspect ectopic: No intrauterine gestational sac and: ✔Transabdominal US: β-hCG > 3500 mIU/mL ✔Transvaginal US: β-hCG > 1800 mIU/mL • β-hCG − Normal IUP – increases by 53% q 48 hours (“doubles”) − Cannot alone differentiate between an ectopic and IUP • Chorionic villi (diagnostic curettage) − If not detected, ectopic pregnancy should be suspected. − Only considered when β-hCG levels are falling or when levels are elevated and US does not show intrauterine pregnancy Change in β-hCG Level Over 48 Hours Evaluation of Possible Ectopic Pregnancy

Change in β-hCG Level Percentage of cases in which the change occurs

Intrauterine Pregnancy • Appropriate increase (>50%) 99 • Inappropriate increase (<50%) 1 Ectopic Pregnancy • Inappropriate increase or decrease 71 • Rate of increase similar to viable IUP (>50%) 21 • Rate of increase similar to SAB (<50%) 8 Spontaneous Abortion • Appropriate decrease (>35%) 90 • Inappropriate decrease (<35%) 10 Adapted from Apgar and from Speroff. Villi Absent on D&C

Decreasing hCG Rising or stable hCG Rising or stable Mass > 4 cm hCG Mass < 4 cm

Follow hCG titers Laparoscopy Medical or laparotomy Treatment Serum Progesterone Levels

• Can detect pregnancy failure and identify patients at risk for ectopic pregnancy • Not diagnostic of ectopic pregnancy − Sensitivity is 15%. − Therefore, 85% of patients with ectopic pregnancy will have normal serum progesterone levels. • Algorithms for diagnosing ectopic pregnancy that include progesterone levels miss more ectopic pregnancies and require more surgeries than do algorithms without progesterone. Treatment

• Hemodynamically unstable patient − Laparotomy • Early diagnosis and patient stable − Laparoscopic salpingostomy • Most cost-effective treatment − Medical management with Methotrexate (MTX) in a single or multi- dose regimen • Multi-dose more effective than surgery, but more expensive • Single-dose has a higher failure rate than laparoscopic salpingostomy, especially in patients with higher β-hCG levels. Absolute Contraindications to Single-Dose MTX

• Breastfeeding • Overt or lab evidence of immunodeficiency • Alcoholism, alcoholic liver disease, or other chronic liver disease • Preexisting blood dyscrasias, such as bone marrow hypoplasia, leucopenia, thrombocytopenia, or significant anemia • Known hypersensitivity for methotrexate • Acute pulmonary disease • Peptic ulcer disease • Hepatic, renal, or hematologic dysfunction, and several metabolic diseases Methotrexate

• Agent most commonly used for medical treatment of ectopic • Folic acid antagonist that inhibits DNA synthesis and cell replication • Mechanism of action oSelectively kill cytotrophoblasts (rapidly dividing cells at fallopian tube implantation site) oBody spontaneously resorbs Monitoring MTX Therapy for Ectopic Pregnancy Obtain a CBC with differential and liver and renal function tests before starting any regimen

Regimen Surveillance Single dose* ⮚Measure β-hCG level on days 4 and 7 MTX, 50mg/m2 IM • If difference >15% decrease (between day 4 and 7), repeat weekly until undetectable(typically 5-7 weeks) • If difference < 15% decrease (between day 4 and 7), REPEAT MTX dose and begin new Day 1 ⮚If fetal cardiac activity present on Day 7, repeat MTX dose and begin new Day 1 ⮚Refer for surgical treatment if β-hCG level is NOT decreasing or fetal cardiac activity persists after 3 MTX doses Adapted from Seeber BE, et al. Obstet Gynecol. 2006 and Little SH, et al. J Fam Prac. 2012;61(11):684 Single-Dose MTX

• Easier to administer and monitor • Best prognostic indicator of successful treatment is initial β-hCG level. − Lower initial level the higher the success rate. • Contraindications: − Ectopic pregnancy > 4.0 cm and fetal cardiac activity Surgery

• Salpingectomy or Salpingostomy • Salpingostomy may result in inadequate evacuation of the products of conception oThus, recurrence of symptoms oCheck β-hCG level weekly to ensure it reaches zero Ectopic Expectant Management • Criteria oLow and decreasing (β-hCG < 1000) oAdnexal mass not detected (via US) oNo embryonic heartbeat oMinimal pain or bleeding oReliable follow-up Opting for a trial of expectant management?

• Extensive counseling on risk of tubal rupture, need for close surveillance • β-hCG q 48 hours to confirm decrease; then, weekly until they reach zero oNo specific rate of decrease = normal. Expectant management continues if patient is asymptomatic and decrease continues (even if gradual) or temporarily plateaus SORT: Key Recommendations for Practice

Clinical Recommendation . Evidence Rating Serial measurements of β-hCG levels should be performed in patients with possible ectopic pregnancy. Most viable intrauterine pregnancies (99%) have β-hCG values that increase by about 50% in 48 hour. C

When deciding between surgical and medical treatment of ectopic pregnancy, the choice should be based on the patient’s preference after discussing the risks, benefits, and monitoring requirements of C both approaches. The patient’s absolute β-hCG level should be considered when deciding whether an ectopic pregnancy C can be treated with methotrexate, because the success rate is lower with higher β-hCG levels. Treatment failure may be assumed if the patient’s β-hCG level does not decrease by at least 15% from day 4 to day 7 after methotrexate injection, or if it plateaus or increases after the first week of treatment. C In such cases, additional methotrexate administration or surgical intervention is required. Expectant management should be considered for patients who have low and decreasing β-hCG levels, no evidence of an ectopic mass visualized by transvaginal ultrasonography, and minimal symptoms. B

Barash et al., Diagnosis and Management of Ectopic Pregnancy. Am Fam Phys. 2014;90(1):34-40 Diagnosis of Ectopic Summary

• Failure of β-hCG to double in 48-72 hours • Ultrasound (transvaginal) − IUP rules out ectopic − No gestational sac + β-hCG > 1800 highly suggestive − Gestational sac/embryo outside of uterus confirms ectopic. − Pitfalls: Pseudogestational sac, ruptured corpus luteum • Laparoscopy: Gold standard First-Trimester Bleeding • Differential diagnosis • Ectopic pregnancy • Spontaneous pregnancy loss ✔ Chromosomal abnormalities are causative ~50% of the time. ✔ No bed rest or drug therapy including progestogens will correct the common etiologies (Cochrane 2004). • Idiopathic bleeding in a viable pregnancy • Cervical abnormalities • Infection of the vagina or cervix • Subchorionic hemorrhage • Vaginal trauma • Molar pregnancy Criteria for Diagnosing Early Pregnancy Failure Society of Radiologists in Ultrasound

• Definitive pregnancy failure can be diagnosed in a woman with an IUP of uncertain viability when TVUS reveals any of the following: o Embryonic crown-rump length > 7 mm and no heartbeat o Mean gestational sac diameter > 25 mm and no embryo present o No embryo with heartbeat > 2 weeks after TVUS showed a gestational sac without a yolk sac o No embryo with heartbeat > 11 days after TVUS showed a gestational sac with a yolk sac

N Engl J Med. 2013 October 10;369:1443. (http://dx.doi.org/10.1056/NEJMra1302417) 9. For which one of the following conditions is there good evidence (Level A) for the routine administration of anti-D immune globulin (RhoGAM) to an unsensitized, D-negative patient?

A. After a first-trimester pregnancy loss B. Abdominal trauma C. Second- or third-trimester antenatal bleeding D. Attempting an external cephalic version Anti-D Immune Globulin Unsensitized, D-negative patient, if she has had one of the following conditions or procedures: • Level A Evidence ✔At approximately 28 weeks gestation, unless the father of the baby is also known to be Rh D negative ✔Within 72 hours after the delivery of an Rh D-positive infant ✔After a first-trimester pregnancy loss ✔After invasive procedures such as chorionic villus sampling, amniocentesis, or fetal blood sampling • Level C Evidence ✔Threatened abortion ✔Second- or third-trimester antenatal bleeding ✔External cephalic versionAbdominal trauma

Prevention of Rh D Alloimmunization. ACOG Practice Bulletin #4, Obstet Gynecol. 1999;93(5). Reaffirmed 2013 THANK YOU Answers

1. C 2. D 3. C 4. C 5. B 6. C 7. A 8. C 9. A 10. A Supplementary Slides 10. A 22 yo G1 female at 9 weeks gestation sees you for follow-up after an episode of vaginal bleeding. Her β-hCG levels are rising, and ultrasonography confirms a viable 8-week IUP with no apparent source of bleeding. Appropriate information and advice for this patient would include which of the following? A. She is at increased risk for miscarriage. B. She should remain at bed rest for 1 week beyond the end of the bleeding episode. C. She should take prenatal vitamins to reduce the risk of miscarriage. D. She should take a progestogen to reduce the risk of miscarriage. Pregnancy Loss

• Spontaneous pregnancy loss (loss of a pregnancy without outside intervention < 20 wks gestation) − Threatened − Inevitable − Incomplete − Missed − Ectopic − Complete − Recurrent spontaneous (habitual) • Ultrasonography is very helpful in the diagnosis. • Chromosomal abnormalities are causative ~50% of the time. Pregnancy Loss

• 20% of pregnant women will have some bleeding < 20 wks gestation. − ~50% will end in a spontaneous pregnancy loss. • 20% of recognized pregnancies will end in miscarriage or loss. • No bed rest or drug therapy including progestogens will correct the common etiologies (Cochrane 2004). • At the time of onset of the bleeding, viability is most likely already determined. First-Trimester Bleeding

• Laboratory tests should include − KOH and wet prep of vaginal secretions − CBC − ABO and Rh testing − Quantitative hCG − GC and Chlamydia testing • Ultrasonography − Crucial in identifying status of the pregnancy − Location of the pregnancy − Transvaginal with hCG > 1,800 mIU/mL (should see IUP) − Transabdominal with hCG > 3,500 mIU/mL (should see IUP) CDC - “High-Risk” Sexual Behaviors

• Previous STDs • New or multiple sex partners • Inconsistent condom use • Engage in commercial sex work and drug use • Those living in communities with a high prevalence of disease Pregnancy Loss

• Biochemical predictors of “Successful Outcomes” − “Doubling” of the β-hCG level in 48 hours − Serum progesterone level > 25 ng/mL • Predictors of “Failure of Successful Outcomes” − Subchorionic hemorrhage on US • If present, continuation rate of pregnancy is decreased even if heartbeat is present. − Serum progesterone level < 5 ng/mL − Failure of the β-hCG to double at 48 hrs First-Trimester Bleeding and Preterm Delivery

• Compared with controls, women presenting with first-trimester bleeding or threatened pregnancy loss were more likely to deliver prematurely. • Bleeding group rate was 11.9%. − Most likely between 34-37 wks gestation • Control group rate was 5.6%. © 2020 American Academy of Family Physicians. All rights reserved.

All materials/content herein are protected by copyright and are for the sole, personal use of the user.

No part of the materials/content may be copied, duplicated, distributed or retransmitted

in any form or medium without the prior permission of the applicable copyright owner.

Endocrine Diseases

Benjamin Gilmer, MD, MS MAHEC Family Medicine University of North Carolina SOM Disclosure Statement

It is the policy of the AAFP that all individuals in a position to control content disclose any relationships with commercial interests upon nomination/invitation of participation. Disclosure documents are reviewed for potential conflicts of interest. If conflicts are identified, they are resolved prior to confirmation of participation. Only participants who have no conflict of interest or who agree to an identified resolution process prior to their participation were involved in this CME activity.

All individuals in a position to control content for this session have indicated they have no relevant financial relationships to disclose. Learning Objectives

1. Identify the diagnosis and management of the common types of hypothyroidism, hyperthyroidism & thyroid nodules. 2. Discuss the work-up of pituitary masses. 3. Review the diagnosis & work-up of parathyroid disorders, Addison’s disease and Cushing’s syndrome 17 yo with neck “swelling” for 2-3 wks, 5 lb weight gain and feeling increasingly tired. Otherwise has no symptoms. Exam significant for a diffusely enlarged, smooth and non-tender thyroid.

What initial test would you order for this patient? 1. What initial test would you order for this patient? A. T4 level B. T3 resin uptake (T3 uptake or T3RU) C. TSH D. Thyroid ultrasound Thyroid Dysfunction

HYPOTHYROIDISM EUTHYROID HYPERTHYROIDISM

TSH = HIGH TSH = LOW

T4 LOW T4 NORMAL T3/ T4 NORMAL T4 HIGH

OVERT SUBCLINICAL SUBCLINICAL OVERT

NOTE: You don’t need symptoms to be classified as overt. Screening for Thyroid Dysfunction & Cancer

• The USPSTF concludes that the current evidence is insufficient to assess the balance of benefits and harms of screening for thyroid dysfunction in nonpregnant, asymptomatic adults

•The USPSTF recommends against screening for thyroid cancer in asymptomatic adults.

Final Update Summary: Thyroid Dysfunction: Screening. U.S. Preventive Services Task Force. September 2016. https://www.uspreventiveservicestaskforce.org/Page/Document/UpdateSummaryFinal/thyroid-dysfunction-screening

Final Recommendation Statement: Thyroid Cancer: Screening. U.S. Preventive Services Task Force. May 2017. https://www.uspreventiveservicestaskforce.org/Page/Document/RecommendationStatementFinal/thyroid-cancer-screening1 Hypothyroidism

Is common Variety of causes •Primary : • Chronic autoimmune (Hashimoto) thyroiditis • Ablation • Females > Males (6:1) •Secondary •1:300 U.S. adults • Lithium • Interferon • Amiodarone •Transient • Silent, subacute, or postnatal thyroiditis •Central Risk Factors for Hypothyroidism

• Female sex • Goiter • Advancing age • Previous hyperthyroidism • White race • Possibly due in part to ablation • Type 1 diabetes therapy leading to iatrogenic thyroid dysfunction • Down syndrome • External-beam radiation in the • Family hx of thyroid disease head and neck area Hypothyroidism Signs & Symptoms

• Fatigue • Myalgia • Coarse hair • Dry skin • Macroglossia • Goiter • Hair loss • Lateral eyebrow • Constipation • Hoarseness thinning • Concentration loss • Slow DTRs • Weight gain • Hyperlipidemia* • Depression • Cold intolerance** • Bradycardia Diagnosing Hypothyroidism

• History ‾ TSH • Physical Exam ‾ T4 • Lab testing ‾ Thyroid antibodies ‾ Surrogate markers • High CPK, LDL, triglycerides, proteinuria, normocytic anemia 2. 6 weeks after initiating therapy with levothyroxine, you check a steady-state TSH level. The TSH decreased from 9.3 to 7.21. All of the following factors are barriers to reaching a therapeutic TSH level EXCEPT? A. Treating with desiccated thyroid supplementation B. Treating with levothyroxine (T4) rather than liothyronine (T3) C. Patient is also taking lithium D. Patient taking ferrous sulfate for iron deficiency Initiating Hypothyroidism Treatment • Use levothyroxine (T4) • Start at 1.6 mcg/kg/day • Start lower in the elderly (1.0-1.25) • Take on fasting stomach & wait 30 mins before eating • Re-evaluate @ steady state (5-6 weeks after dosage change) • Different products can have different bioavailability (e.g., generic brands; Armour) • Combining T3/T4 generally not superior to T4 alone Hypothyroidism Treatment Principles • Avoid desiccated thyroid – poorer quality control • Avoid triiodothyronine/ liothyronine (T3) • If supratherapeutic can develop osteoporosis; A Fib • Interactions: Iron, sucralfate, cholestyramine, antacids, anticonvulsants, grapefruit, amiodarone, lithium, SSRIs, retinoids Hypothyroidism Treatment Principles

• If TSH WNL but patient not feeling well, consider - Pushing TSH to <2.5. - Problems with conversion of T4 to T3 (see next slide) • Exposures & nutrient deficiencies - Heavy metals: selenium, chromium, zinc, iron, copper, mercury, lead - Iodine - Vitamins: A, B2, B6, B12, D, E Decreased T4 →T3 Conversion Medications

• OCPs • SSRIs • Beta blockers • Steroids • Fluoride • Phenytoin • Chemotherapy • Opiates • Lithium • Iodinated contrast • Estrogen agents • Theophylline Decreased T4 →T3 Conversion Other Factors • Stress • Receptor antibodies • Aging • Low ferritin • Alcohol use • Pesticides • Fasting • Soy (excess) • Radiation • Hemochromatosis • Excess cruciferous • Smoking vegetables • Kidney disease 3. 42 yo c/o fatigue, weight loss, voracious appetite, hand tremor, HAs, and worsening anxiety symptoms for 4 weeks. Exam significant for a diffusely enlarged non-tender thyroid & fine hand tremor. TSH is 0.12 and T4 is elevated. What is the next diagnostic step? A. CBC B. Radioiodine uptake study C. Thyroid antibodies D. Thyroid ultrasound HypERthyroidism

Is common Variety of causes - Graves disease - TSH receptor-stimulating : antibodies - Functional thyroid nodules - Multinodular goiter; - Thyroiditis - Hashimoto*; postpartum • Females > Males (8:1) - Ingestion - Amiodarone; Iodine • U.S. Prevalence: ~0.2 % - Metastatic thyroid cancer - Hyperemisis gravidarum Risk Factors for HypERthyroidism

• Female sex •Personal or family history of • Advancing age thyroid disease • Black race • Ingestion of iodine- • Low iodine intake containing drugs, such as amiodarone HypERthyroidism Signs & Symptoms

• Nervousness • Irritability • Palpitations • Weight loss • Heat intolerance • Increased appetite • Tremor • Hyperdefecation • Fatigue • Mental changes • Insomnia • Dyspnea on exertion • Headache Diagnosing HypERthyroidism

• History & Physical Exam • Labs & studies • TSH (A) • Free T4 & T3 (A) • Radioactive uptake scan (A) • CBC (B) • Consider: ESR, ultrasound, thyroid abs (C) Diagnosing HypERthyroidism

Radioactive thyroid uptake & scan Graves Disease: Treatment

• Radioactive iodine is the TOC: A Rec - Except perhaps in cases with ophthalmopathy: B Rec • Surgery is uncommon today • Medications - PTU or methimazole & beta blockers - Chinese herbal meds (I Rec, Cochrane 2007) Graves Disease: Treatment

• Methimazole safer than PTU • With PTU, risk of serious liver injury is - Adults: 1:10,000 - Peds: 1:2,000 • PTU now considered a 2nd-line agent - EXCEPT during pregnancy (1st tri) & lactation 4. 46 yo c/o painless neck mass for 6 wks. On exam you palpate a 2-cm firm mass in right lobe of thyroid. What is the initial diagnostic test? A. TSH B. Fine-needle aspiration C. Free T4 D. Thyroid ultrasound Thyroid Nodules

• Work up all nodules - 5% are malignant - 95% of solitary thyroid nodules are benign hyperplastic lesions • Start with TSH (SOR A) - TSH results determine further workup

Image from Shutterstock Thyroid Nodules

• Benign • Malignant - – Papillary carcinoma (60%) - Cysts – Follicular carcinoma (12%) - Hashimoto’s - Subacute thyroiditis - Riedel’s struma Thyroid Nodule Workup Other Causes of Hyperthyroidism

Hashimoto’s Thyroiditis Subacute Thyroiditis • Chronic lymphocytic thyroiditis • Acute inflammatory disease • Prevalence= 0.3-1.2% (viral) • Typically painless, nontender, • Fever and thyroid diffuse goiter tenderness - Can have pain, pressure • Initial hyperthyroidism → and , low grade transient hypothyroidism fever, high ESR or CRP • Clinical dx with thyroid function tests The Thyroid in Pregnancy

• Thyroid size can increase by 10% (more in iodine deplete countries) • 50% increase in thyroid hormone production & 50% in iodine needed • 10% of gravid women in 1st trimester will be + for thyroid peroxidase or thyroglobulin antibodies - 16% have hypothyroidism - 33-50% develop post partum thyroiditis

Erik K. Alexander, Elizabeth N. Pearce, Gregory A. Brent, Rosalind S. Brown, Herbert Chen, Chrysoula Dosiou, William A. Grobman, Peter Laurberg, John H. Lazarus, Susan J. Mandel, Robin P. Peeters, and Scott Sullivan.Thyroid.Mar 2017.ahead of printhttp://doi.org/10.1089/thy.2016.0457 The Thyroid in Pregnancy

• TSH goal in pregnancy <3.0 • Treatment not needed for isolated low T4 • Women already on levothyroxine should increase dose by 25-50% during pregnancy. • Antithyroid meds are NOT indicated for women with gestational hypERthyroidism • For Graves, use PTU in 1st trimester, then methimazole The Thyroid in Pregnancy

• Post partum thyroiditis • No anti-thyroid medication during toxic phase • Check TSH q 2 months after toxic phase for hypothyroidism • Can try to wean off levothyroxine @ 6-12 months after starting treatment • No radioactive iodine scanning during pregnancy 5. 58 yo with hypertension has an elevated calcium level on routine lab work. He denies any complaints & his exam is unremarkable. Additional labs are significant for an elevated calcium, elevated parathyroid level and low phosphate levels.

You suspect he has hyperparathyroidism. All of the following are common symptoms of hyperparathyroidism EXCEPT:

A. Bone pain B. Increased appetite C. Depression D. Renal Stones Causes of Hypercalcemia

• Parathyroid hormone-related • Granulomatous disease - Primary hyperparathyroidism* sarcoidosis, berylliosis, • Sporadic, familial, tuberculosis associated with multiple • Hodgkin's lymphoma endocrine neoplasia I or II • Malignancy - Tertiary hyperparathyroidism - Humoral hypercalcemia of - Associated with chronic renal malignancy* (mediated by failure or vitamin D deficiency PTHrP) • Vitamin D-related - Solid tumors, especially lung, - Vitamin D intoxication head, and neck squamous - Usually 25-hydroxyvitamin D2 , renal cell tumors in over-the-counter - Local osteolysis* (mediated by supplements cytokines) multiple myeloma, breast cancer Causes of Hypercalcemia

• Medications • Genetic disorders - Thiazide diuretics (usually mild)* - Familial hypocalciuric - Lithium hypercalcemia: mutated calcium- - Milk-alkali syndrome (from sensing receptor calcium antacids) • Other - Vitamin A intoxication (including - Immobilization, with high bone analogs used to treat acne) turnover (e.g., Paget's disease, • Other endocrine disorders bedridden child) - Hyperthyroidism - Recovery phase of - Adrenal insufficiency rhabdomyolysis - Acromegaly - Pheochromocytoma Hyperparathyroid

• Presentation • Treatment - Bone pain - Locate & remove tumor - Depression surgically - Frequent urination - Kidney Stones - Nausea - Loss of appetite Parathyroid

• Hypoparathyroid - Only 900 cases per year in U.S. - I would not expect any questions on such an uncommon disease - Treatment is to restore calcium & mineral balance thru Ca+++ & Vit D supplements • Hyperparathyroid - ~2 per thousand people affected - May be primary (most common) or secondary Hyperparathyroidism

Primary Secondary •Typically from single benign •Typically due to vitamin D adenoma (80%) deficiency, chronic kidney - Multiple benign tumors, rarely disease, or other causes of low parathyroid cancer blood calcium •Most people with primary disease •Manage underlying cause have no symptoms at the time of diagnosis •Remove the adenoma or overactive parathyroid 6. A 33 yo presents with irregular menses and galactorrhea for 3 months. The remainder of her PMH and PE are unremarkable. Lab studies are within normal limits, except for a prolactin level of 310 mg/mL. The most appropriate next step in her work-up is:

A. Dexamethasone suppression test B. PET scan of the brain C. MRI of the brain D. CT head with and without contrast – Diagnosis • Prolactin correlates with tumor size • Rule of 200’s • Prolactin level >200 is almost always a prolactinoma • <25 is normal • Pregnancy test • Thyroid function studies (TSH and Free T4) • MRI with contrast or CT scan with coronal cuts • Formal visual field examination if >10 mm in size • Evaluation of remainder of pituitary function, if indicated Pituitary Abnormalities

• Sx depend on hormone secreted • Prolactin – Amenorrhea, galactorrhea, impotence • Growth hormone – Gigantism and acromegaly • Corticotropin – Cushing’s disease • TSH – Hyperthyroidism Evaluation of a Sellar Mass

• MRI with and without gadolinium - Gadolinium contrast • Normal pituitary takes up gadolinium more than does CNS tissue • Microadenomas often take up gadolinium less than normal pituitary • CT - Calcification in a craniopharyngioma or a meningioma is seen better by a CT than by MRI • PET Scan - Uptake by adenoma was 2-3X greater than by craniopharyngiomas or meningiomas Differential Diagnosis for a Sellar Mass

• Benign Tumors • Malignant Tumors • Pituitary adenoma • Primary (most common mass) • Germ cell tumor • Craniopharyngioma • Sarcoma • Meningiomas • Chordoma • Pituitary carcinoma • Metastatic • Lung • Breast Cushing’s Syndrome (Think  Cortisol) Primary features Diagnostic steps - Central obesity - Ecchymoses - Confirm the elevated - Plethora cortisol level, then identify - Proximal weakness - Osteopenia/osteoporosis the source - Hypertension - Dexamethasone - WBC >11.0 Suppression Test - Purple striae >1cm wide Other features: Treatment -Myopathy, hirsutism, opportunistic infections, loss of - Reducing steroid use, libido (male) surgery, radiation, and medications Cushing’s Syndrome

• 10-15 per million in general population • Higher prevalence in patients with - Diabetes - Obesity - Hypertension - Osteoporosis • Peak Incidence in 25-40 yo Cushing’s Syndrome Clinical Characteristics of Cushing’s Syndrome • Psychological symptoms 40% • Bruising 36% • Congestive heart failure 22% • Edema 18% • Renal Calculi 16% • Headache 14% • Polyuria/Polydipsia 10% • Hyperpigmentation 6%

• Diagnosis is usually delayed because Sx are nonspecific Laboratory Diagnosis of Cushing’s Syndrome • 1st confirm the hypercortisolemia; 2nd determine the source • Cortisol levels (circadian cycling) • AM cortisol may be normal • Raised midnight cortisol • 24-hr urinary free cortisol • Not affected by obesity, drugs, medical conditions • Need to measure creatinine (ratio unreliable) • 4x normal unequivocal, lower uncertain • Midnight salivary cortisol • Raised in medical/psychiatric illness • Dexamethasone Suppression test Dexamethasone Suppression Test Dexamethasone acts like cortisol, lowers the amount of ACTH released by the pituitary Normal Cushing’s Syndrome Give Dexamethasone

Pituitary Adrenals makes less make Pituitary Adrenals ACTH less makes less still cortisol ACTH making cortisol Addison’s Disease (Think cortisol &  aldosterone) • Primary • Clinical presentation - Atrophy or destruction - Hyperpigmentation of adrenal glands - Low blood pressure • Secondary - Weight loss - Inadequate secretion - N&V of ACTH from pituitary - Muscle cramps gland - Irregular menses - Salt craving - Malaise, fatigue Addison’s Disease (Think cortisol &  aldosterone) • Diagnostic Testing • Treatment - Serum electrolytes - Replace cortisol - Blood glucose - Replace aldosterone - CBC - ACTH stimulation test - CT scan adrenals - MRI adrenals Summary Pearls • Thyroid - TSH to start diagnosis - Be vigilant for thyroid disease in pregnancy • Parathyroid - Focus on hyperparathyroidism (high Ca, high PTH) - Usually primary, remove the tumor • Pituitary Masses - Sx dependent on specific hormone • Addison’s Disease - Think low cortisol and aldosterone (Must Add some); electrolyte abnormalities • Cushing’s Syndrome - Think high cortisol; dexamethasone suppression testing • Supplementary slides – pituitary masses, thyroid ca Answers

1. C 2. B 3. B 4. A 5. B 6. C Bibliography

• Am Thyroid Assoc Management Guidelines For Thyroid Nodules. 2009 • Bahn, R et al. Approach to the Patient with Nontoxic Multinodular Goiter. J Clin Endocinol Metab, May 2011, 96: 1202-12. • Gaitonde DY, et al. Hypothyroidism: An Update. Am Fam Physician. 2012;86:244-51. • Barbesino G. Drugs affecting thyroid function. Thyroid. 2010;20:763-70. • Wiersinga WM. Thyroid disease manager. http://www.thyroidmanager.org/chapter/adult-hypothyroidism/ • Levy EG, et al. Algorithms for diagnosis & management of thyroid disorders. www.thyroidtoday.com/ExpertOpinions Bibliography

• Erik K. Alexander, Elizabeth N. Pearce, Gregory A. Brent, Rosalind S. Brown, Herbert Chen, Chrysoula Dosiou, William A. Grobman, Peter Laurberg, John H. Lazarus, Susan J. Mandel, Robin P. Peeters, and Scott Sullivan.Thyroid.Mar 2017.ahead of printhttp://doi.org/10.1089/thy.2016.0457 • Final Update Summary: Thyroid Dysfunction: Screening. U.S. Preventive Services Task Force. September 2016. https://www.uspreventiveservicestaskforce.org/Page/Document/UpdateSummaryFinal/ thyroid-dysfunction-screening • Final Recommendation Statement: Thyroid Cancer: Screening. U.S. Preventive Services Task Force. May 2017. https://www.uspreventiveservicestaskforce.org/Page/Document/RecommendationStat ementFinal/thyroid-cancer-screening1 Supplementary Slides Adenomas

•There is a separate webinar available covering adenomas Pituitary Masses

• 10-15% of all primary brain tumors • 20-25% of pituitary glands at autopsy found to have adenomas • 70% of adenomas are endocrinologically secreting • 25% of those with MEN-I develop pituitary adenomas • Etiology is unknown • Not associated with environmental factors Sonographic Evidence of Thyroid Cancer in a Nodule

• PPV (%) • Taller than wide 59.8 • Solid appearance 49.4 • Microcalcifications 38.6 • Irregular margins 28.2

• Purely cystic nodule is highly unlikely (< 2%) Workup of Multinodular Thyroid for Cancer

• Dynamic Contrast Medium-Enhanced MRI (DCE- MRI) is more accurate than FNA in detecting cancer in a multinodular gland: B Rec, Tezelman. Archives of Surgery, 2007 • Negative Predictive Value = 100% For FNA, it’s 58% PPV: DCE-MRI = 78.5%; FNA = 100% Diagnostic accuracy: DCE = 90%; FNA = 71% Subclinical Hypothyroidism

• Prevalence: 5%-17% • Risk for progression to overt dz: 8%-18% • Look for Sx • Treat if TSH > 10, attempting conception, or + thyroid peroxidase Ab • Be observant for overtreatment: Osteoporosis, A Fib • Treatment does NOT result in improved survival or morbidity, nor QOL nor Sx: A Rec, Cochrane, 2007 Subclinical Hyperthyroidism

• Subclinical hyperthyroidism: Any antithyroid drug is effective (A Rec) • Nygaard. AFP’s Clinical Evidence Concise. 2007;76:1014-7. • Prevalence: 0.1%-6% • Risk higher in women, age > 60, + antibodies • Higher osteoporosis, death from CV causes, A Fib • Joint decision-making for treatment or not © 2020 American Academy of Family Physicians. All rights reserved.

All materials/content herein are protected by copyright and are for the sole, personal use of the user.

No part of the materials/content may be copied, duplicated, distributed or retransmitted

in any form or medium without the prior permission of the applicable copyright owner.

Maternity Care II

David G. Weismiller, MD, ScM, FAAFP Department of Family and Community Medicine University of Nevada, Las Vegas School of Medicine Disclosure Statement

It is the policy of the AAFP that all individuals in a position to control content disclose any relationships with commercial interests upon nomination/invitation of participation. Disclosure documents are reviewed for potential conflicts of interest. If conflicts are identified, they are resolved prior to confirmation of participation. Only participants who have no conflict of interest or who agree to an identified resolution process prior to their participation were involved in this CME activity.

All individuals in a position to control content for this session have indicated they have no relevant financial relationships to disclose. Learning Objectives

1. Identify appropriate techniques to recognize and manage premature onset of labor. 2. Define the treatment of DM and the diagnosis and treatment of hypertension in pregnancy. 3. Describe management of induction and augmentation of labor including postterm pregnancy. 4. Recall the recognition and treatment of late pregnancy bleeding including abruptio placentae, placenta previa, and postpartum hemorrhage. 1. A 20 yo female is seen for follow-up 6 weeks after delivery. Her pregnancy was complicated by preeclampsia. Her examination is unremarkable. This patient will be at increased risk for which of the following in midlife?

A. Breast cancer B. Diabetes C. Hypothyroidism D. Ischemic heart disease Definitions

• Gestational hypertension • Nomenclature − > 140/90 mm Hg on 2 − Preeclampsia occasions at least 4 hours − Eclampsia apart • Preeclampsia PLUS Seizure − > 160 mm Hg systolic or > − Chronic hypertension 110 mm Hg diastolic − Chronic hypertension with • Hypertension can be superimposed preeclampsia confirmed within minutes to facilitate timely − Gestational hypertension antihypertensive therapy. • Transient hypertension of pregnancy • Chronic hypertension Epidemiology • Hypertension in pregnancy is the most common medical complication of pregnancy occurring in 5-10% of all cases • 6th leading cause of maternal mortality US 2006-2009, 9.9% of maternal death (CDC)

Condition By the numbers Chronic Hypertension 1-5% of pregnancies Superimposed Preeclampsia on Chronic 25% of chronic HTN Hypertension Gestational Hypertension 3-6% of all pregnancies Preeclampsia 3-5% of all pregnancies Severe Preeclampsia ~ 1/3 of preeclampsia Eclampsia 0.05%-0.2% of all pregnancies Preeclampsia – Eclampsia

Diagnosis • Gestational BP elevation after 20 weeks of gestation in a woman with previously normal BP • Proteinuria > 0.3 g protein in a 24-hour urine specimen − No longer REQUIRED to make the diagnosis of preeclampsia when hypertension occurs with one or more severe features. − When proteinuria IS used among other diagnostic criteria for preeclampsia, a protein:creatinine ratio of at least 0.3 is sufficient. ACOG Task Force on Hypertension in Pregnancy. Obstet Gynecol. 2013;122(5):1122-1133. Severe Features • SBP > 160 mm Hg; DBP > 110 mm Hg on 2 occasions at least 4 hours apart while the patient is on bed rest (unless antihypertensive therapy is initiated before this time) • Plt count < 100,000/mL • Impaired liver function −Abnormally elevated transaminases (> twice normal) −Severe persistent RUQ pain or epigastric pain unresponsive to medication and not accounted for by alternative diagnosis, or both • Progressive renal insufficiency (serum Cr > 1.1 mg/dL or a doubling of the serum creatinine concentration in the absence of other renal disease) • Pulmonary edema • New-onset cerebral or visual disturbances Preeclampsia – Eclampsia

• Diagnosis −Gestational BP elevation after 20 weeks of gestation in a woman with previously normal BP −Proteinuria > 0.3 g protein in a 24-hour urine specimen • No longer REQUIRED to make the diagnosis of preeclampsia when hypertension occurs with one or more severe features • When proteinuria IS used among other diagnostic criteria for preeclampsia, a protein:creatinine ratio of at least 0.3 is sufficient −Severe features • Patients who have had preeclampsia have a 4X increased risk of hypertension and a 2X increased risk of ischemic heart disease, stroke, and VE. −No association between preeclampsia and cancer

ACOG Task Force on Hypertension in Pregnancy. Obstet Gynecol. 2013;122(5):1122-1133. Risk Factors for Preeclampsia • Pregnancy-associated factors • Maternal-specific factors – Chromosomal abnormalities – Age < 20 and > 35 – Hydatidiform mole – Black race – Hydrops fetalis – Family history – Multifetal pregnancy – Nulliparity – Oocyte donation or donor insemination – Preeclampsia in previous pregnancy – Structural congenital abnormalities – Medical conditions – Urinary tract infection • Diabetes • Obesity • Paternal-specific factors • Chronic hypertension – First-time father • Renal disease – Previously fathered a preeclamptic pregnancy in • Thrombophilias another woman – Offspring of preeclamptic pregnancy Chronic Hypertension

• Gestational BP elevation before the 20th week of gestation • Hypertension that is diagnosed for the first time during pregnancy and does not resolve postpartum is also classified as chronic hypertension • Chronic hypertension is the most common cause of IUGR • 25% will develop superimposed preeclampsia Typical Laboratory Evaluation

• Uric Acid • CBC oHemoconcentration and platelet count • Liver Function Tests (AST or ALT) • Protein/Creatinine Ratio • 24-hour Urine Total Protein Management of Chronic Hypertension • Blood pressures persistently above 160/110 should be treated to goals between 120/80 and 160/110 −Labetalol −Nifedipine • Ultrasound screening for fetal growth restriction is appropriate. • Antenatal fetal surveillance* should be performed in women who: −Require antihypertensive therapy −Have superimposed preeclampsia −Have fetal growth restriction

ACOG Practice Bulletin No. 203. Chronic Hypertension in Pregnancy. Obstet Gynecol. 2019;133(1):e26-50. *Fetoplacental assessment: Biophysical profile or modified biophysical profile to include umbilical artery Doppler velocimetry Treatment of Hypertension in Pregnancy and Postpartum ACC/AHA - 2014

• Severe hypertension should be treated (Level A) −Methyldopa, labetalol, nifedipine • Consideration MAY be given to treatment of moderate hypertension given the evidence for POSSIBLY increased stroke at currently defined systolic and diastolic BP cutoffs, as well as evidence for decreased risk for the development of severe hypertension with treatment (although maternal-fetal risk-benefit ratios have NOT been established). (Level B) • After giving birth, continue medications in women with chronic hypertension – dose adjustment to reflect the decrease in the volume of distribution and GFR that occurs after delivery. Monitor for development of postpartum preeclampsia (Level C)

Bushnell C, McCullough LD, Awad IA, et. al. Guidelines for the Prevention of Stroke in Women: A Statement for Healthcare Professionals From the American Heart Association/American Stroke Association. Stroke. 2014;45. Summary of Recommendation and Evidence Low-Dose Aspirin for Prevention of Preeclampsia – September 2014 USPSTF • Recommends the use of low-dose aspirin (81 mg/d) as preventive medication after 12 weeks of gestation in women who are at high risk for preeclampsia. (Grade: B recommendation) • Concludes with moderate certainty that there is a substantial net benefit of daily low-dose aspirin use to reduce the risk for preeclampsia, preterm birth, and IUGR in women at high risk for preeclampsia. Clinical Risk Assessment for Preeclampsia – USPSTF

Risk Level Risk Factor Recommendation High1 • History of preeclampsia, especially when accompanied by an adverse Low-dose aspirin is the outcome patient has > 1 of these risk • Multifetal gestation factors • Chronic hypertension • Type 1 or 2 diabetes • Renal disease • Autoimmune disease (i.e., systemic lupus erythematous, the antiphospholipid syndrome) Moderate2 • Nulliparity Consider low-dose aspirin if • Obesity (body mass index 30 kg/m2) the patient has several of • Family history of preeclampsia (mother or sister) these moderate-risk factors3 • Sociodemographic characteristics (African American race, low socioeconomic status) • Age >35 y • Personal history factors (e.g., low birthweight or small for gestational age, previous adverse pregnancy outcome, 10-y pregnancy interval) Low Previous uncomplicated full-term delivery No low-dose aspirin 1. Single risk factors that are consistently associated with the greatest risk for preeclampsia. The preeclampsia incidence rate would be approximately 8% in a pregnant woman with 1 of these risk factors 2. A combination of multiple moderate-risk factors may be used by clinicians to identify women at high risk for preeclampsia. These risk factors are independently associated with moderate risk for preeclampsia, some more consistently than others 3. Moderate-risk factors vary in their association with increased risk for preeclampsia Prevention of Preeclampsia ACOG – 2013

• Women with histories of preeclampsia that is recurrent or has previously developed prior to 34 weeks SHOULD be offered daily low-dose aspirin (60-80 mg) commencing at the end of the first trimester • NOT recommended for primary prevention of preeclampsia −Bed rest/restriction of physical activity (not for treatment of POL either) −Vitamin C or E administration −Dietary salt restriction • Calcium may be useful to reduce the severity of preeclampsia in populations with low calcium intake; this finding is NOT relevant to a population with adequate calcium intake, e.g., United States • Bedrest is NOT recommended for prevention OR treatment of preeclampsia Prevention of Preeclampsia ACC/AHA – 2014

• Women with chronic primary or secondary hypertension or previous pregnancy-related hypertension should take low-does aspirin from the 12th week of gestation until delivery (Level A) • Calcium supplementation (of > 1g/d, orally) should be considered for women with low dietary intake of calcium (<600 mg/d) to prevent preeclampsia (Level A)

Bushnell C, McCullough LD, Awad IA, et. al. Guidelines for the Prevention of Stroke in Women: A Statement for Healthcare Professionals From the American Heart Association/American Stroke Association. Stroke. 2014;45. Prevention of Stroke in Women With a History of Preeclampsia • Increased risk of future hypertension and stroke 1 to 30 years after delivery in women with a history of preeclampsia (Level B) • Consider: −Evaluating all women starting 6 months to 1 year postpartum, as well as those who are past childbearing age, for a history of preeclampsia/eclampsia as a risk factor (Level C) −Evaluate and treat for CV risk factors including hypertension, obesity, smoking, and dyslipidemia (Level C)

Bushnell C, McCullough LD, Awad IA, et. al. Guidelines for the Prevention of Stroke in Women: A Statement for Healthcare Professionals From the American Heart Association/American Stroke Association. Stroke. 2014;45. Gestational Hypertension

• Gestational blood pressure elevation without proteinuria that is detected for the first time after 20 weeks gestation • Two types − Transient hypertension of pregnancy: Preeclampsia has not developed and BP has returned to normal by 12 weeks postpartum. − Chronic hypertension: BP elevation persists. Timing of delivery

• Women with gestational hypertension or preeclampsia without severe features should have planned delivery at 37 weeks gestational age (Level B) • Women with chronic hypertension and with no additional maternal or fetal complications supporting earlier delivery (Level B) − if not prescribed maintenance antihypertensive medications, delivery before 38 0/7 weeks of gestation is not recommended. − if prescribed maintenance antihypertensive medications, delivery before 37 0/7 weeks of gestation is not recommended.

ACOG Practice Bulletin No. 202 Gestational Hypertension and Preeclampsia. Obstet Gynecol. 2019;133(1):e1-25. ACOG Practice Bulletin No. 203. Chronic Hypertension in Pregnancy. Obstet Gynecol. 2019;133(1):e26-50. 2. 19 yo primigravida at approximately 40 weeks EGA comes to the hospital with painful contractions. She has received no prenatal care. Examination: Cervix is 4 cm dilated and 80% effaced at station -1. Blood pressure is 164/111 mm Hg and a urine dipstick shows 3+ protein. She reports that she has had severe headaches for 3 days and has noticed a lot of swelling in her hands and feet. Moments after her blood is drawn and IV access is obtained, she has a generalized tonic-clonic seizure and fetal heart tones drop to 60 BPM. Which of the following is the most appropriate immediate course of action?

A. Emergency cesarean section B. Lorazepam, 2 mg IV push, repeated in 2 minutes if necessary C. Magnesium sulfate, 4 g loading dose IV, followed by a drip at 2 g/hr D. Terbutaline, 0.25 mg subcutaneously Management

• Anticonvulsive therapy oMgSO4 oReduces the risk of first or subsequent seizures in women with severe preeclampsia ✔Now ONLY recommended for women with preeclampsia with severe features oReduces risk of subsequent seizures in women with eclampsia ✔Antidote for toxicity: Calcium gluconate 1 g IV over 3 minutes • Antihypertensive therapy • Vaginal delivery is preferred! ACOG Task Force on Hypertension in Pregnancy.Obstet Gynecol. 2013;122(5):1122-1133. Antihypertensive Therapy Acute Hypertension in Preeclampsia • BP dangerously high or rises suddenly in women with preeclampsia, especially intrapartum. • Antihypertensive medications should not be administered unless BPs are persistently > 160/110 Pharmacologic Agent Dose Arterial vasodilator 5 mg IV over 1-2 minutes, or IM e.g., hydralazine Beta-blocker 20-40 mg IV bolus or 1 mg/kg infusion (maximum e.g., labetalol 220 mg) Calcium antagonists 10 mg po and repeat in 30 minutes, if necessary; oral nifedipine lowered BP e.g., nifedipine more quickly than did IV labetaolol. (Shekhar S et al. Obstet Gynecol. 2013;Nov;122:1057.)

Sodium nitroprusside 0.25 ug/kg/min to a maximum of 5 ug/kg/min Postpartum Management

• In women with gestational hypertension and/or preeclampsia, BP should be monitored for at least 72 hours postpartum and again 7-10 days after delivery. • Greatest risk for postpartum hypertension o Antenatal preeclampsia – particularly with higher urinary protein, serum uric acid, and BUN

ACOG Task Force on Hypertension in Pregnancy. Obstet Gynecol. 2013;122(5):1122-1133. 3. Gestational diabetes has been associated with all of the following perinatal complications EXCEPT:

A. Increased frequency of maternal hypertensive disorders B. Increased risk of operative delivery C. Increased frequency of neonatal hyperglycemia D. Increased risk of intrauterine fetal death during last 4-8 weeks of gestation Why All the Fuss … Adverse Outcomes Maternal Fetal • Increased frequency • Excessive fetal growth (macrosomia) o Maternal hypertensive disorders o Increased risk for operative delivery o Cesarean delivery o Shoulder dystocia • Increased risk of intrauterine fetal o Birth trauma death during last 4-8 weeks of • Neonatal morbidity gestation o Hypoglycemia o Fasting hyperglycemia o Hypocalcemia (> 105 mg/dL) o Hyperbilirubinemia o Polycythemia The most common cause of neonatal death in children of mothers known to have DM before pregnancy is congenital anomalies. GD Treatment ACOG 2018 • Initial management (Level A) − Nutritional counseling by registered dietician − Advice on moderate exercise program (if possible); minimum of 150 minutes per week • No conclusive evidence for the threshold value at which clinicians should start pharmacologic therapy • Pharmacologic treatment − Insulin is considered first-line treatment in pregnancy (Level A) − Glyburide treatment should NOT be recommended as a first-line pharmacologic treatment because, in most studies, it DOES NOT yield equivalent outcomes to insulin (worse outcome including macrosomia and birth injury (Level B) − In women who decline insulin therapy or if unable to safely administer, metformin is a reasonable second-line choice (Level B) ACOG Practice Bulletin No. 190: Gestational Diabetes Mellitus. Obstet Gynecol. 2018;131(2):e49-e64. Gestational Diabetes Maternal Surveillance – Glucose Monitoring

• Glucose Target Levels − Fasting or preprandial < 95 mg/dL − 2-hour postprandial BG < 120 mg/dL (1 hour < 140mg/dL) − 1-2 times per week versus daily; review weekly • Pharmacologic Treatment (if on more than 3 occasions) − > 95 mg/dL fasting whole blood glucose or − > 120 mg/dL 2 h postprandial − Daily glucose monitoring Diabetes Timing of Delivery?

• Timing of delivery in women with GDM that is controlled with only diet and exercise (A1GDM) should NOT be before 39 weeks gestation, unless otherwise indicated. Expectant management up to 40 6/7 weeks of gestation in the setting of indicated antepartum testing is generally appropriate (Level C) • GDM well controlled on medications (A2GDM) or Type 2, delivery is recommended at 39 0/7 to 39 6/7 weeks of gestation (Level C)

ACOG – 2018 Diabetes Timing of Delivery?

• Poorly controlled – Expert guidance supports earlier delivery but data lacking regarding precise timing −Delivery between 37 weeks 0 days and 38 weeks 6 days may be justified −Delivery between 34 weeks 0 days and 36 weeks 6 days reserved for (1) failure of in-hospital glycemic control or (2) abnormal fetal testing • Council regarding risks/benefits of scheduled cesarean delivery when EFW > 4500 g (Level C)

ACOG – 2018 Long-Term Considerations

• Increased risk for recurrence of GD − 33%-50% likelihood • Increased risk for development of diabetes after pregnancy − Up to 1/3 will have diabetes or impaired glucose metabolism at postpartum screening. − 35% of women 5-10 years after parturition • Offspring – increased risk − Obesity − Glucose intolerance − Diabetes in late adolescence and young adulthood Postpartum

• Reclassification of maternal glycemic status at least 4 weeks after delivery (preferred 4-12 weeks, ACOG 2018) [Level C] − FPG or 2-hr OGTT • Reassessment of glycemia every one (USPSTF) to three years (ADA) [SOR:C], if above normal; yearly assessment (ADA) if impaired fasting glucose or impaired glucose tolerance at 6-12 weeks

ACOG – 2018 Management of Postpartum Screening Results

Gestational diabetes

FPG or 75-g, 2-hr OGTT at 4-12 weeks postpartum

FPG > 125 mg/dL or FPG 100-125 mg/dL or FPG < 100 mg/dL or 2-hr glucose > 199 mg/dL 2-hr glucose 140-199 mg/dL 2-hr glucose <140 mg/dL

Impaired fasting glucose or Diabetes mellitus Normal IGT or both

• Consider referral for management • Assess glycemic status Refer for diabetes • Weight loss and physical activity every 1-3 years. management counseling as needed • Consider metformin if combined • Weight loss and physical impaired fasting glucose and IGT activity counseling as • Medical nutrition therapy needed • Yearly assessment of glycemic status 4. A 26 yo G2P1 presents at 30 weeks gestation with a complaint of severe itching. She has excoriations from scratching in various areas. She says that she had the same problem during the last pregnancy, and her medical records reveal a diagnosis of intrahepatic cholestasis of pregnancy. Elevation of which of the following is most characteristic of this disorder? A. GGT B. Bile acids C. Direct bilirubin D. Prothrombin time Intrahepatic Cholestasis of Pregnancy (ICP)

• Most common pregnancy-related liver disorder • Etiology ambiguous − Abnormalities in the metabolism and disposition of sex hormones and/or bile acids • ? Genetic predisposition • ? Environmental factors Intrahepatic Cholestasis of Pregnancy Diagnosis • Historic − Pruritus at night, pruritus continually (with no rash) and mild jaundice during the 3rd trimester; elevation of serum level of total bile acids • Clinical jaundice in 50% of cases; pruritus worsens with onset of jaundice. − Tends to recur (60%-70%); severity varies. − Seldom exhibits before 25 weeks • Hepatitis − More than 20% of women known to be infected with hepatitis C develop ICP, so testing is indicated. • Liver biopsy − Pathologic findings are not specific, and a liver biopsy is rarely indicated. Laboratory Presentation

Parameter Finding Serum alkaline Increased 5-10 fold (hepatic > placental) phosphatase Bilirubin Elevated bilirubin (conjugated, < 5 mg/dL) Serum transaminases Usually normal

Serum bile acids > 3 times upper limit of normal (fasting) GGT Usually normal or modestly elevated – may help to differentiate this condition from other cholestatic liver diseases

PT Usually normal – but if elevated may reflect a vitamin K deficiency from malabsorption Pharmacologic Treatment

• Goal: Decrease maternal symptomatology and enhance fetal outcome. • Main drug prescribed: Most benefit for both mother and infant − ursodeoxycholic acid* − 10 mg/kg q day Pharmacologic Treatment

Drug Action Effect Pregnancy Risk Cholestyramine Anionic exchange resin binding bile Decreased pruritus, no effect on acids in the intestine for excretion fetal outcome; no effect on liver C enzymes Dexamethasone Suppression of fetal production of Alleviate pruritus and liver enzyme estrogen decreasing bile acid levels abnormalities C

Epomediol Terpenoid compound, prevents and Alleviate pruritus; no change in liver reverses cholestasis induced by ethinyl enzyme abnormalities C

Phenobarbital Induces hepatic enzymes to decrease Some benefit to decrease bile acids symptoms; no effect on fetal D outcome Ursodeoxycholic acid, Hydrophilic bile acid that replaces more Decreases pruritus, bile acids; Actigall, and Ursodiol cytotixic bile acids improved fetal outcomes B

Hydroxyzine Antihistamine activity Decreased pruritus C Outcomes – ICP

• Maternal – usually benign o Impaired quality of life o Pruritus and abnormal laboratory tests resolve within 1-2 weeks of delivery, no sequelae. • Fetal – may be serious o Higher bile acid levels and impaired transport of bile acids from the fetus through the uteroplacental circulatory system ✔Preterm labor, preterm delivery ✔Fetal compromise ✔Increased risk of cesarean delivery ✔Meconium-stained amniotic fluid ✔IUFD Bleeding in Late Pregnancy

• Placenta previa • Placenta abruption • Vasa previa • Cervical trauma (e.g., intercourse) • Vaginal infections • “Bloody show” 5. Which of the following statements is true regarding bleeding in late pregnancy?

A. The most serious complications of placental abruption are the result of hypovolemia B. Young maternal age is a risk factor for placenta previa C. Placenta previa typically presents with painful vaginal bleeding D. Placental abruption is diagnosed using ultrasonography Placenta Previa

• Occurs in 0.3%-0.6% of all births • Painless vaginal bleeding • Bleeding patterns – first bleeding episode: − Before 30 weeks 33% − Between 30 and 35 weeks 33% − After 36 weeks 33% • Each successive episode of bleeding may be heavier, more unpredictable. Placenta Previa Etiology • Usually none is found • Risks: − Prior C/S or history of uterine curettage • ? Damage to myometrium or endometrium − Cocaine − Advanced maternal age − Tobacco • Recurrence rate of 6-12x in subsequent pregnancy • Malpresentation is the result rather than the cause of placenta previa Placenta Previa Diagnosis • Antenatal by ultrasound • Before 24 weeks, only a cautionary significance is attributed to a placenta close to the os. − 90% have moved away from the os at term. − Need to confirm “movement” in 3rd trimester. − In cases where the bulk of the placenta is over the os after 24 weeks, it is less likely to be clear of the os at term. Placental Abruption

• Separation of the placenta from implantation site - Clinical diagnosis (pain and typically vaginal bleeding) • Occurs in approximately 1% of all deliveries − Incidence increases with gestational age. − Most serious complications are due to hypovolemia  acute renal failure. • Etiology − Trauma, cocaine abuse, acute decompression of amniotic fluid, preterm rupture, hypertensive disorders, tobacco use • Risk of recurrence high (20-30x) Placental Abruption Management • Assess fetal viability − Fetal demise – here placental detachment usually > 50% • Deliver the fetus. • Prepare to transfuse. − Live fetus + rigid uterus – again > 50% detachment • 90% fetal demise - C/S − Live fetus + soft uterus - induction of labor Antepartum Hemorrhage Summary

Symptom/Sign Placenta previa Placental Abruption

Pain Minimal Severe

Contractions Absent or mild Hard, rapid, often tetanic

Blood Bright red Port wine

Coagulation Normal Abnormal

Concealed Almost never > 20% of time hemorrhage Preterm Labor Diagnosis

• Gestation > 24 weeks, < 37 weeks • Contractions and cervical change 6. Which of the following is a risk factor for preterm labor?

A. Prior preterm birth B. Interpregnancy interval < 12 months C. Long cervical length on endovaginal ultrasound (> 25mm) D. Administration of 17-hydroxylprogesterone caproate to expectant mother Risk Factors for Preterm Labor

• Prior preterm birth • Non-Hispanic black race − Recurrence risk of • Interpregnancy interval < 6 17%-37% months • Multiple gestation • Tobacco use • Short cervical length on • Infection endovaginal U/S – Bacterial vaginosis − <25 mm at 18-25 weeks – Bacteriuria – Urinary tract infection Preterm Labor Symptomatic Management Intervention Recommendation Bed rest Not effective for the prevention of preterm birth and should NOT be routinely recommended (Level B) Hydration Not effective for the prevention of preterm birth and should NOT be routinely recommended (Level B) Tocolytic agents First line: Beta-adrenergic agonist, calcium channel blockers, NSAID for short-term prolongation of pregnancy; maintenance therapy is INEFFECTIVE for preventing preterm birth and improving neonatal outcomes (Level A) Corticosteroids Single course between 24 and 34 weeks who are at risk of preterm delivery with in 7 days (Level A) (Betamethasone 12 mg q 24hX2 OR dexamethasone 6 mg q 12hX4) Antibiotics Should NOT be used to prolong gestation or improve neonatal outcomes in women with POL and INTACT membranes (Level A) Magnesium Sulfate Reduces the severity and risk of CP in surviving infants if administered when birth is anticipated before 32 weeks of gestation (Level A) Prevention of Preterm Labor

• 17-hydroxyprogesterone caproate (IM) − Trade name: Makena − 250 mg IM weekly from 16-20 weeks through 36 weeks • Only one indication − Pregnant women with a history of at least one spontaneous preterm birth Prevention of Preterm Delivery

• In addition to 17-P −Check cervical length via U/S every 2 weeks from 16-22 weeks −Consider cerclage for cervical length <25 mm • Benefit of 17-P for women with history of preterm delivery −0.31 RR of birth < 34 weeks [CI=0.14-0.69] −0.50 RR of neonatal mortality [CI=0.33-0.75] 7. A 26 yo gravida 3 para 2 with mild chronic hypertension and an uncomplicated pregnancy has just delivered a vigorous female by spontaneous vaginal delivery. She is noted to have heavy vaginal bleeding and a bimanual examination reveals a soft, poorly contracted uterus. Her temperature is 37.1°C (98.8°F), blood pressure 168/97 mm Hg, pulse rate 105 beats/min, and oxygen saturation 95% on room air. Which one of the following uterotonic agents is CONTRAINDICATED in the management of this patient’s postpartum hemorrhage? A. Oxytocin (Pitocin) B. Carboprost tromethamine (Hemabate) C. Misoprostil (Cytotec) D. Methylergonovine Postpartum Hemorrhage (PPH) Definition

• Cumulative blood loss of greater than or equal to 1,000 mL or • Blood loss accompanied by signs or symptoms of hypovolemia −Early (or primary): First 24 hours after delivery −Late (or secondary): After 24 hours but before 6 weeks after delivery

ACOG Practice Bulletin: Clinical Management Guidelines for Obstetrician-Gynecologists Number 183, October 2017: Postpartum Hemorrhage, Obstetrics and Gynecology 2017;130(4):e168-e186. (Reaffirmed 2019) Early Postpartum Hemorrhage Etiology

• Uterine atony (Tone) • Retained placental − Uterine overdistention fragments (Tissue) • Hydramnios ‒ Including abnormal placentation • Multiple gestation • Accreta • Fetal macrosomia • Increta − Oxytocin use − High parity • Percreta − Rapid or prolonged labor − Intra-amniotic infection − Halogenated anesthetic Early Postpartum Hemorrhage Etiology

• Lacerations of vagina • Coagulopathy (Thrombin) and cervix (Trauma) − Hereditary − Forceps − DIC − Fetal macrosomia • Sepsis − Precipitous labor and • Abruption delivery − Hemolysis, Elevated Liver − Episiotomy Enzymes, Low Platelets • Uterine rupture (HELLP) • Uterine inversion Treatment Approach

• Uterine massage − Bimanual • Oxytocics (medical uterotonic therapy) • Inspect for lacerations • Surgical intervention

Anderson JM and Etches D. Prevention and Management of Postpartum Hemorrhage. Am Fam Physician. 2007;75:875-82. Postpartum Hemorrhage Treatment Medical Uterotonic Therapy • Oxytocin − IV, IM, or intramyometrial

• Misoprostol (Cytotec, PGE1) − Per rectum (400 mcg after placenta and 100 mcg at 4 and 8 hours postpartum, total 800-1000 mcg)* • Methylergonovine − Contraindicated in presence of hypertensive disease state

• 15-methyl PGF2-alpha (Carboprost or Hemabate) − IM or intramyometrial

• Dinoprostone (PGE2) * Caliskan E et al. Am J Obstet Gynecol. 2002; 187:1038-45. Postpartum Hemorrhage Prevention

• Correct anemia • Avoid routine episiotomy • Infant to breast • Routine use of medications after delivery of the placenta *Misoprostol is NOT approved by the U.S.FDA for use in prevention or treatment of PPH Medications for Prevention of PPH Medication Dose Contraindications/ Mechanism of Side Effects Cautions Action First Line Agent Oxytocin 10 IU IM or 5-10 Overdose or Stimulates the Rare IU IV bolus prolonged use upper segment of can cause water the myometrium to Intoxication. contract rhythmically, constricting Possible hypotension spiral arteries with IV use decreasing blood following cesarean flow through the section uterus Second Line Agent Misoprostol* 600 mcg oral; Caution in patients with Generalized smooth Nausea, vomiting, Use ONLY when CV Disease muscle contraction diarrhea, pyrexia Prostaglandin E1 oxytocin is NOT available. analog Postpartum Hemorrhage Prevention • Correct anemia • Avoid routine episiotomy • Infant to breast • Routine use of medications after delivery of the placenta • Active management of third stage Third Stage of Labor Management

Expectant Active • Await separation • Oxytocin with shoulder • Leave cord uncut delivery • Spontaneous placental • Cord clamped and cut early delivery (2-3 minutes) • Oxytocin/breast after placental • Controlled cord traction separation

Begley CM, Gyte GM, Devane D, Mcguire W, Weeks A. ActiveVersus Expectant Management for Women in the Third Stageof Labour. Cochrane Database Syst Rev 2011:CD007412. PPH Recommendations (SOR A)

• Active management of the third stage of labor should be utilized to decrease the risk of PPH, postpartum maternal hgb < 9 mg/dL, and the need for manual removal of the placenta. • Delayed cord clamping (one to three minutes) decreases neonatal risk of anemia and does not increase risk of PPH • Oxytocin remains first choice for prevention of PPH because it is as, or more, effective than ergot alkaloids or prostaglandins and has fewer side effects • Misoprostol is less effective for prevention of postpartum hemorrhage than oxytocin and has more side effects Postdates Pregnancy > 42 weeks • Accurately date everyone. − Early sonograms help (Level A) • Antenatal fetal testing between 41 and 42 weeks (Level C) − q 4-7 days depending on method − Most do MBPP q 3-4 days • Induction: Risk of routine induction (C/S) in the era of cervical ripening agents is lower than previously reported − Cervical ripening: Improves success of inductions (Level B) • Bishop score of 8 − Oxytocin − Nonpharmacologic methods of labor induction may be helpful Cervical Ripening Agents (Induction is indicated, cervix is unfavorable) • Total score > 8 (Max score 15); probability of vaginal delivery after labor induction is similar to that after spontaneous labor. Dilatation 0 cm 1-2 cm 3-4 cm > 5 cm (Points) 0 1 2 3 Effacement 0%-30% 40%-50% 60%-70% > 80% (Points) 0 1 2 3 Consistency Firm Medium Soft (Points) 0 1 2 Position Post Mid Anterior (Points) 0 1 2 Station* -3 -2 -1/0 +1/+2 (Points) 0 1 2 3

* Station reflects a -3 to +3 scale. Modified from Bishop EH. Pelvic scoring for elective induction. Obstet Gynecol. 194;24:267. Macrosomia ACOG - > 4500g (Morbidity increases sharply beyond this weight) • Level A − Prediction of birth weight imprecise by US or clinical measurement. For suspected macrosomia, the accuracy of estimated fetal weight using US biometry is no better than that obtained with abdominal palpation − No contraindications - encourage engagement in aerobic and strength conditioning exercises during pregnancy to reduce the risk of macrosomia − Control of maternal hyperglycemia reduces the risk of macrosomia; maternal glucose management recommended for pregnancies complicated by DM • Level B − Suspected fetal macrosomia or large for gestational age is NOT an indication for induction of labor before 39 0/7 weeks of gestation … insufficient evidence that benefits of reducing shoulder dystocia risk would outweigh the harms of early delivery • Level C − Suspected macrosomia is NOT a contraindication to labor after cesarean.

ACOG Practice Bulletin No. 216. Macrosomia,. Obstet Gynecol. 2020;135(1):246-248. Induction

• Level A Evidence − Postterm pregnancies with unfavorable cervix can undergo induction OR expectant management. − Prostaglandins (PG) can be used to promote cervical ripening AND induce labor. − Delivery should be effected if there is evidence of fetal compromise or oligohydramnios. • Level C Evidence − Reasonable to initiate fetal monitoring between 41-42 weeks. • Many practitioners use twice weekly. • NST and AFI (MBPP) should be adequate. − Many recommend induction or prompt delivery in patients with a favorable cervix and no other complications. Additional Resource

• Syllabus – Advanced Life Support on Obstetrics −First Trimester Complications −Medical Complications −Preterm Labor/PROM −Vaginal Bleeding in late pregnancy −Intrapartum Fetal Surveillance −Maternal Resuscitation/PPH −Shoulder Dystocia −Labor Dystocia

Answers 1. D 2. C 3. C 4. B 5. A 6. A 7. D 8. A 9. D Supplementary Slides 8. A 26 yo G2P1 at 36 weeks fell off of her bicycle 2 hours ago. She says that she has felt no movement since the fall. She has abrasions on the left forearm and left leg but otherwise appears uninjured. Which one of the following would be the most sensitive indicator of fetal compromise in this patient? A. Continuous electronic fetal monitoring B. Vaginal bleeding C. Uterine ultrasonography D. Kleihauer-Betke Management of the Pregnant Woman After Trauma • Stabilize the pregnant woman using “ABC” protocols • Supplemental oxygen • IV lactated Ringer’s solution in 3:1 ratio based on blood loss • Laboratory studies − CBC − Blood type and Rh − Kleihauer-Betke • Not predictive of fetal outcome − Injuries are severe – consider: • Coagulation studies • Chemistry panel • ABG • Deflect uterus off of great vessels if more than 20 weeks gestation. • Radiologic studies as needed Secondary Survey

• Examine for nonobstetrical injury. − Treat as needed. • Speculum examination to rule out spontaneous rupture of membranes, vaginal bleeding • Check for fetal heart tone.

Fetal heart tone Fetal heart tone ABSENT PRESENT

Maternal treatment only; no fetal Estimate gestational age (history, fundal height, Leopold’s resuscitation maneuvers, ultrasonography < 24 weeks of gestation*

Document fetal heart tone No fetal resuscitation

* Determination of fetal viability varies by institution. Some centers use estimated fetal weight limits typically 500 g; others use a gestational age of 24-26 weeks. > 24 Weeks of Gestation*

YE EFM for 24 hours, intervene Electronic fetal monitoring S if needed for NRFHR tracing. for four hours

• Vaginal bleeding • SROM D/C monitoring and discharge to • FH tone abnormality home, return to L and D: • Uterine contractions for > 4 hours • Vaginal bleeding • High-risk mechanism of injury • Decreased fetal movement NO • Uterine tenderness • Loss of fluid vaginally • Abdominal pain • Repetitive uterine contractions • Maternal anesthesia • Abdominal pain or tenderness 9. Following a prolonged second stage of labor, your primigravid patient at 42 weeks gestation delivers the fetal head, but the anterior shoulder does not release with a normal amount of downward traction. The statement that most accurately is associated with this complication is:

A. A history of shoulder dystocia in a previous pregnancy does not confer any greater risk of the complication in this pregnancy. B. It is important to stop and suction the newborn before attempting to deliver the anterior shoulder in cases where this complication is anticipated C. Routine induction of labor for macrosomia has been proven to improve perinatal outcomes. D. After delivery of the fetal head, pH can be expected to drop by 0.04 per minute due to cord compression. Shoulder Dystocia

• Caused by the impaction of the anterior shoulder above the symphysis pubis • Elective induction of labor or elective cesarean delivery for all women suspected of carrying a fetus with macrosomia is not appropriate (Level B). − Planned C/S to prevent SD may be considered (Level C) • EFW > 5000 g with no DM • EFW > 4500 g with DM Shoulder Dystocia: Complications • Maternal • Fetal − Lacerations Brachial plexus (10%-20%) • Perineum • Erb’s palsy • Vagina • Majority are bruising/stretching; complete • Rectum recovery in 6-12 months (80+ percent) − Hemorrhage Cervical spine • Uterine atony Asphyxia • Lacerations • Cord pH drops 0.04/minute Fractures • Clavicle • Humerus Shoulder Dystocia

• A true obstetric emergency; studies suggest the majority are relieved by maneuvers that are safe for fetus and mother. • Manual maneuvers will facilitate resolution of most of these cases. − Sequence in which maneuvers are performed will depend on the experience/preference of the individual provider. No evidence that one maneuver is superior to another in releasing an impacted shoulder or reducing the chance of injury. • Fundal pressure is counterproductive and not indicated. ACOG Committee Opinion Term Singleton Breech

• Decision regarding mode of delivery should depend on experience of healthcare provider. − Cesarean will be preferred mode for most physicians because of diminishing expertise in vaginal breech delivery. • Offer and perform ECV whenever possible. • Planned vaginal delivery may be reasonable. − Great caution, detailed patient informed consent • Inform woman that the risk of perinatal or neonatal mortality or short-term serious neonatal morbidity may be higher than if cesarean delivery is planned. © 2020 American Academy of Family Physicians. All rights reserved.

All materials/content herein are protected by copyright and are for the sole, personal use of the user.

No part of the materials/content may be copied, duplicated, distributed or retransmitted

in any form or medium without the prior permission of the applicable copyright owner.

The Surgical Abdomen

Robert Dachs, MD, FAAFP Clinical Associate Professor Ellis Hospital Family Medicine Residency Program Albany Medical College, Albany, New York Disclosure Statement

It is the policy of the AAFP that all individuals in a position to control content disclose any relationships with commercial interests upon nomination/invitation of participation. Disclosure documents are reviewed for potential conflicts of interest. If conflicts are identified, they are resolved prior to confirmation of participation. Only participants who have no conflict of interest or who agree to an identified resolution process prior to their participation were involved in this CME activity.

All individuals in a position to control content for this session have indicated they have no relevant financial relationships to disclose. Learning Objectives

1. Recognize the signs and symptoms of intra-abdominal conditions requiring surgical intervention. 2. Perform specialized maneuvers to narrow the differential diagnosis of acute abdominal pain. 3. Appropriately utilize labs and radiology tests in the evaluation of acute abdominal pain. 4. Appreciate that acute abdominal pain in children could be due to different etiologies than adult patients. Abdominal Pain

Non-surgical Surgical “acute” abdomen

I. II.obstruction

-Blood -Pus -Foreign material Location, Location, Location…

• RUQ------> hepatitis/GB • LUQ------> gastric/spleen • Periumbilical------> pancreatitis/early appy • RLQ------> appy/GYN (torsion, cysts, PID, ectopic) • LLQ------> diverticulitis/GYN • General/ill-defined--> ischemia, obstruction Location, Location, Location… with visceral pain – ill-defined location • RUQ------> hepatitis/GB • LUQ------> gastric/spleen • Periumbilical------> pancreatitis/early appy • RLQ------> appy/GYN (torsion, cysts, PID, ectopic) • LLQ------> diverticulitis/GYN • General/ill-defined--> ischemia, obstruction Location, Location, Location… when parietal peritoneum involved – localizes

• RUQ------> hepatitis/GB • LUQ------> gastric/spleen • Periumbilical------> pancreatitis/early appy • RLQ------> appy/GYN (torsion, cysts, PID, ectopic) • LLQ------> diverticulitis/GYN • General/ill-defined--> ischemia, obstruction Abdominal Pain and Analgesia: Give it!!

• Pain relief does not increase the risk of diagnostic and management errors

• Dose: morphine 0.1 - 0.2 mg/kg IV

Manterola C, et al. Cochrane Database, 2011 Gallagher EJ, et al. Ann Emerg Med, 2006 Peritonitis and Antibiotics

Regimens: Must cover Gr (-) aerobes and anaerobes Resistance of Bacteroides −PIP/TZ, AM-SB, TC-CL…………. Near zero −Ertapenem, tigecycline −FQ + metronidazole −Cefoxitin, cefotetan ……………… 4-25% (?add Metro?)

No one regimen is proven better than another… (40 trials, 5094 patients – Cochrane review, 2012) Nasogastric Tubes and Abdominal Surgery

• 37 randomized trials, postop patients • 2866 pts with NG tube, 2845 pts (-) NG tube • Results: No NG tube=====> ‾ Earlier return of bowel function ‾ Fewer pulmonary infections ‾ Trend toward fewer wound infections and ventral ‾ Trend toward improved patient comfort, less nausea and vomiting, and decreased length of stay ‾ No difference in anastomotic leaks ‾ Shorter length of stay

Cochrane Library 2010 Acute Abdominal Pain in the Elderly: Be Careful!!!

• More likely to require hospitalization • More likely to require surgery • More likely to be misdiagnosed • More likely to die …compared to younger patients

Lyon C, et al. AFP, 2006 Laurell H. et al. Gerontology, 2006 Abdominal Wall and Hernias

© Todd Buck Illustration, Inc. from Suleiman S, et al. AFP Aug 2001, 431-38 13 Abdominal Wall and Hernias

1. A 72 yo female presents with a 6-hour hx of a painful lump in her groin. Some nausea, no vomiting. No fever/chills. Exam: Abdomen: soft, non-tender, R groin with palpable mass below (inferior to) inguinal ligament. Despite gentle, firm pressure, the mass does not disappear.

The most likely diagnosis is: A. Strangulated indirect inguinal B. Reducible direct inguinal hernia C. Incarcerated femoral hernia D. Incarcerated Spigelian hernia Abdominal Wall and Hernias Epidemiology • 1 million hernia repairs are performed each year in the US • Inguinal hernia repairs constituting > 3/4 of these cases • Approximately 90% of all inguinal hernia repairs are performed on men Terminology – Reducible – Incarcerated: i.e., “Stuck!” (despite gentle firm pressure) – Strangulated: vascular compromise (surgical emergency) Abdominal Wall Hernias: Types •Indirect inguinal hernia: 75% of all hernias!!! -Most common hernia in both sexes!!! -Men are 5x more common -More likely to incarcerate/strangulate than direct •Direct inguinal hernia: 2nd most common Note: You can NOT differentiate between direct and indirect hernias on exam!!! •Femoral hernias: more common in women - Located below inguinal ligament, medial to femoral pulse Abdominal Hernias: Do You Remember These? • Spigelian hernia: rare, located along lateral border of rectus muscle, below umbilicus, at junction of arcuate line

• Richter hernia: When less than full circumference of bowel is trapped Groin Hernia – Is Watchful Waiting Appropriate?

• Yes – if... • Counseling points - Absent or minimal discomfort - Low chance of presenting as surgical emergency (1.8 per 1000 person/yrs) - Is completely reducible - But, 70% typically go onto surgery - Patient preference to forego because of pain - No evidence that physical activity operation results in incarceration on worsening of existing hernia

Montgomery J, et al. JAMA 2018; 320: 1029-30. 2. A 67 yo female with a history rheumatoid arthritis presents with 6 hours of sudden, severe diffuse abdominal pain. Medications: Prednisone 15 mg qd x 5 yrs, celecoxib (Celebrex). Vital Signs: BP=140/90, P=105, RR=30, afebrile. Her abdomen is diffusely tender, with guarding and rebound. The most cost-effective first test is:

A. An upright CXR B. Supine abdomen x-ray C. Abdominal ultrasound D. A CT of the abdomen and pelvis with contrast An Upright CXR is 80% Sensitive for Free Air

Can detect as little as 1-2 mL of gas under the diaphragm or lateral margin of the liver

Courtesy: Wikimedia - https://commons.wikimedia.org/wiki/File%3APneumoperitoneum_modification.jpg Pneumoperitoneum: Etiology • Perforated duodenal ulcer – The most common cause Especially of the anterior aspect of the first part of the duodenum

• Perforated peptic ulcer • Bowel obstruction • Ruptured diverticulum • Penetrating trauma • Ruptured inflammatory bowel disease (e.g., megacolon) • Necrotizing enterocolitis • Cancer • Ischemic bowel • After laparotomy; after laparoscopy 3. A 60 yo male presents to the ED/office with 3 days of worsening RUQ pain. The pain radiates to the right scapula. (+) nausea and vomiting. He is febrile (temp = 39 C, 102.2 F), P = 122, BP = 90/45 He is jaundiced (total bilirubin = 7.8), RUQ is tender. The most likely diagnosis is: A. Acute B. Ascending cholangitis C. Gallstone pancreatitis D. Gallstone ileus Gallstone Disease: Who Is at Risk? • First-degree relative with Hx of gallstones • Obesity, sedentary lifestyle, cyclic weight change

Think recent bariatric surgery!!! • Medical conditions: DM, hyperlipidemia • Surgical conditions: short bowel syndrome, terminal ileal resection (think Crohn’s disease) • Drugs: ceftriaxone, post-menopausal estrogen, TPN • Hemolytic diseases Sickle cell disease ------> black pigmented stones Gallstone Disease • Cholesterol stones most common (80%) • Due to bile supersaturation with cholesterol=====> Crystal formation • Asymptomatic stones (cholelithiasis) Do NOT recommend prophylactic surgery - 10% will go on to symptoms in 5 yrs • Symptomatic stones (biliary colic) Now consider surgery. Why? Randomized trial noted - Observation = 20% hospitalization rate with recurrent pain - After 67 months – Complication Rates Observation = 4% vs. Surgery = 1% Vetrhus M, et al. Scand J Gastroenterol 2002 Complications of Gallstone Disease

Liver .

-----Common hepatic GB . ------Cystic duct .----Common bile duct . Pancreas Duodenum-- Complications of Gallstone Disease

1. Cholecystitis: in 10% of patients with biliary colic Occluded cystic duct =➔ RUQ pain and tender, +/-fever, +/-WBC ***(+) Murphy sign: cessation of inspiration 2. Ascending (acute) cholangitis Result of occluded CBD (choledocholithiasis) 50-70% (+) Charcot’s triad: fever, abd pain, and jaundice Reynolds' pentad---Charcot’s + confusion + shock 3. Gallstone pancreatitis: obstructed ampulla of Vater ===> reflux of bile 4. Gallstone ileus: biliary-enteric fistula, 6/1000 SBO Gallstone Disease: Evaluation • Clinical Suspicion: Classic history • Ultrasound – 99% specific – If high degree of suspicion, but negative US… Think Functional Gallbladder Disorder!!! • Functional Gallbladder Disorder (Dyskinesia, dysmotility): gallbladder ejection fraction < 40% is abnormal (HIDA scan) in conjunction with typical clinical symptoms (Rome III criteria)* 70-94% of patients with dyskinesia, symptoms improve or the disease is cured after cholecystectomy1,2 1Canfield AJ et al. J Gastrointest Surg 1999 2Singhal V, et al J Gastrointest Surg 2012 •Develops in 6-7% of population •It is the most common cause of surgical abdomen •Due to intraluminal (e.g., appendicolith) OR extraluminal obstruction (e.g., lymphadenopathy) Rovsing’s sign: LLQ palpation==> RLQ pain : RLQ pain on thigh extension while lying in left lateral decubitus position : RLQ pain with internal rotation of the flexed right thigh Carnett sign: perform abdominal crunch (+) if pain persists/worsens--> abdominal wall pathology Absence of These Signs Does Not “Rule Out” Appendicitis

Psoas sign: suggests inflamed appendix is retrocecal irritating iliopsoas muscle

Obturator sign: suggests inflamed appendix is in pelvis irritating obturator internus muscle

Hardin DM. Am Fam Physician.1999 Nov1;60(7):2027-2034. Two More Signs… • Dunphy sign: cough elicits pain (also known as “cough sign”) • Markle sign: patient drops from standing on toes to the heels with a jarring landing ===> pain Both suggest peritoneal inflammation And the rectal exam: is useless!!! Study: 1024 pts with possible appy + rectal exam Physicians asked opinion before and after exam Result: made no difference1

1 Dixon JM et al. BMJ 1991;302: 386-388. Appendicitis Labs: Worthless! • WBC – Elevated in 80-85% of pts – But only 60-65% in children and elderly!!!

• CRP – elevates within 6-12 hrs What happens when you combine: WBC + CRP??? • Sensitivity: 85-99% • Specificity: very poor!!!! 4. You are evaluating a 6 yo for abdominal pain. You have a moderate degree of suspicion that the patient may have appendicitis. The American College of Radiology Appropriateness Criteria recommends which of the following as the first choice to evaluate this patient? A. Abdominal x-ray B. Abdominal ultrasound C. CT scan of abdomen and pelvis D. MRI of abdomen and pelvis

Appendicitis: Diagnostic Imaging • X-ray: rarely useful –Unless appendicolith seen • US: first choice: age <14 yrs and pregnancy –26 studies (as of 2006) of US in pediatrics –Sensitivity: 88%; specificity of 94%1 • CT: study of choice in adults –No advantage to using contrast • MRI: good sensitivity and specificity –greater cost, longer acquisition time, and less clinical

availability 1Doria AS, et al. Radiology. 2006;241(1):83-94 Intestinal Obstruction

5. The most common cause of large bowel obstruction is: A. Adhesions (from previous surgery) B. Neoplasm C. Herniation D. Volvulus Intestinal Obstruction

Small bowel (85%) Large bowel (15%)

• Adhesions from previous • Neoplasm (60%) surgery (60%) • Diverticulitis (20%) • Malignant tumor (20%) • Volvulus (10-15%) • Herniation (10%) • Intussusception (<5%) • Inflammatory bowel • Misc. disease (5%) • Volvulus (3%) • Misc. (2%) Small Bowel Obstruction

A. Symptoms B. Signs • Colicky abdominal pain •Distention • Nausea and vomiting •Tympany, high-pitch BS • Diarrhea (early finding)

C. Late Findings •Lack of flatus and constipation •Fever/tachycardia (with strangulation) •Abdominal tenderness and peritoneal signs (with strangulation) Small Bowel Obstruction: Radiography

• Dilated loops of bowel ------> • Paucity of air in colon • Air fluid levels

Jackson PG, et al. AFP 2011 Small Bowel Obstruction: Radiography • Dilated loops of bowel • Paucity of air in colon • Air fluid levels ------May------be “normal”: i.e., false neg (-) early or in high jejunal or duodenal obstruction High clinical suspicion and neg (-) X-ray should Non-contrast CT evaluation From: Wikipedia Small Bowel Obstruction: Management A. Clinically stable: no suggestion of strangulation • NGT, IV hydration, analgesia • Nonoperative trial up to 3 days is warranted. ➔Successful 40-70%

• B. If unstable/peritonitis or (+) suggestion of strangulation… Go to surgery!!!! Large Bowel Obstruction Symptoms • More common in elderly • Crampy abdominal pain –Sudden onset: think acute obstruction (e.g., volvulus) • Nausea, vomiting, distension Signs • Hypertympanic to percussion • Bowel sounds normal early, then become quiet • Fever/tenderness/rigidity – is a bad sign • Cecum is region most likely to perforate Large Bowel Obstruction

- A “closed loop” obstruction (with a competent ileocecal valve)

- Cecum preferentially dilates

- Dilation > 12-14 cm increases risk of perforation

www.practicalhospital.com Sigmoid Volvulus

A long mesentery with a narrow base of fixation to the retroperitoneum and elongated, redundant bowel predisposes to the formation of volvulus Sigmoid colon most common Result of chronic constipation At-risk patients •Neuropsychiatric disorders •Institutionalized/NH patients •Parkinson disease •Multiple sclerosis Image reprinted with permission from Medscape Reference(http://emedicine.medscape •Spinal cord injury .com/), 2014, available at: http://emedicine •Excessive laxatives, enemas .medscape.com/article/2048554-overview. Sigmoid Volvulus

Average age: 8th decade

Treatment: endoscopic decompression - Then semi-elective surgery - If no surgery: up to 50% will have recurrence

12-15% mortality rate Image reprinted with permission from Medscape Reference (http://emedicine.medscape.com/), 2014, available at: http://emedicine.medscape.com/article/2048554-overview. Cecal Volvulus

Hereditary condition: congenital incomplete dorsal mesenteric fixation of the cecum or ascending colon associated with an abnormally elongated mesentery distal to this area of absent mesentery Women > men

Most common: 6th decade of life Treatment: Surgery Image reprinted with permission from Medscape Reference Right hemicolectomy (http://emedicine.medscape.com/), 2014, available at: http://emedicine.medscape.com/article/774045-overview. Vascular Disease and the Abdomen: 4 Cases

A. Chronic mesenteric ischemia (“Abdominal angina”) B. Acute mesenteric arterial thrombosis C. Ischemic colitis SMA D. Acute mesenteric IMA venous thrombosis Vascular Disease and the Abdomen: 4 Cases

A. Chronic mesenteric ischemia (“Abdominal angina”) B. Acute mesenteric arterial thrombosis C. Ischemic colitis SMA D. Acute mesenteric IMA venous thrombosis Chronic Mesenteric Ischemia Also Known as – “Abdominal Angina” • Etiology: atherosclerotic disease - Involves proximal celiac, SMA, IMA arteries - Because of collateral circulation, 2 of 3 involved • Mean age: 60 yrs, women > men; 3:1 • Presentation: postprandial pain • Workup: Duplex US, CT-Angiography, MRA or conventional angiography • Treatment: angioplasty, stenting, surgery Case #1: 60 yo female 4-6 month hx of worsening postprandial abdominal pain. Described an “ache,” in midepigastric-central location. Pain starts 15 min. after eating, increases over the next 1-2 hrs., then abates. Worse with large meals. Subsequently, she has lost 20 lbs in 6-8 weeks. PMHx: HTN, hyperlipidemia, tobacco abuse Physical exam: unremarkable Labs, CT scan, upper and lower endoscopy – all “normal” She has been told she has “IBS.” A.Chronic mesenteric ischemia B. Acute mesenteric arterial thrombosis C.Ischemic colitis D.Acute mesenteric venous thrombosis Vascular Disease and the Abdomen: 4 Cases

A. Chronic mesenteric ischemia (“Abdominal angina”) B. Acute mesenteric arterial thrombosis C. Ischemic colitis SMA D. Acute mesenteric IMA venous thrombosis Acute Mesenteric Arterial Occlusion

• Etiology: embolus > thrombosis - SMA preferentially involved • Presentation: “Pain out of proportion to physical findings” • Mortality: very high Case #2: 78 yo male presents to ED with 90-minute history of severe diffuse abdominal pain. Started suddenly at 7:30 p.m. (+) nausea, vomiting. No change in bowel/bladder habits. PMHx: CAD, HTN, Afib, frequent falls, early dementia Meds: metoprolol, ASA, donepezil VS: BP = 115/70, P = 110, irreg. RR = 28, afebrile PE: The patient is writhing in bed, very restless, little to no relief with opiate analgesia. Abdomen: soft, non-tender, absent bowel sounds, no rebound or guarding. A. Chronic mesenteric ischemia B.Acute mesenteric arterial thrombosis C.Ischemic colitis D.Acute mesenteric venous thrombosis Vascular Disease and the Abdomen: 4 Cases

A. Chronic mesenteric ischemia (“Abdominal angina”) B. Acute mesenteric arterial thrombosis C. Ischemic colitis D. Acute mesenteric SMA venous thrombosis IMA Ischemic Colitis • Etiology: “low-flow” state - Nongangrenous – 80-85%; gangrenous – 15-20% • Age: 90% over the age of 60 yrs • Location: left colon 75% • Presentation: varies on severity - Mild case: crampy abd pain, diarrhea, heme (+) stool - Severe: peritonitis • Work-up: CT, MRA or colonoscopy • Treatment: - Mild case: liquid diet, antibiotics - Severe: IV fluids, antibiotics, surgery Case #3: 82 yo NH resident with one week of progressive weakness. Crampy abdominal pain, (+) diarrhea. Rare vomiting, decreased po intake. Outpatient identified UTI - Rx with nitrofurantoin (Macrodantin). Vitals: BP = 90/45, P = 115, RR = 28, T = 100.6 PE: Ill-appearing, restless. Mucus membranes: very dry. The abdomen is distended and tympanic; absent BS, Mild left-sided tenderness, but no rebound or guarding. Rectal exam = brown stool, heme (+) Labs: Na+ 146, K+ 2.7, Cl 115, HCO3- 15, BUN 66/Cr 2.2 WBC = 15.5, H/H = 10.5/31.5 U/A = WBC - TNTC A. Chronic mesenteric ischemia B. Acute mesenteric arterial thrombosis C.Ischemic colitis D.Acute mesenteric venous thrombosis Vascular Disease and the Abdomen: 4 Cases

A. Chronic mesenteric ischemia (“Abdominal angina”) B. Acute mesenteric arterial thrombosis C. Ischemic colitis D. Acute mesenteric venous thrombosis Acute Mesenteric Venous Thrombosis • Frequency: 10-15% of all mesenteric ischemia • At-risk population: hypercoagulable state • Presentation: insidious over 7-10 days • Workup: - X-rays: nonspecific findings (ileus) •Late findings: thumbprinting, pneumatosis or portal venous gas - Duplex US: useful if used early - CT: may==> enlarged mesenteric or portal vein and thrombus within the vein •Gas in the wall of the bowel, fat streaking, and thickened bowel wall have also been noted • Treatment: anticoagulation, thrombolytics - Surgery: for bowel infarction, peritonitis Case #4: 42 y/o female with 7-10 day hx of vague, midabdominal pain. (+) nausea and vomiting (+) anorexia and occasional diarrhea. No fever. PMHx: HTN FHx: Mother PE age 40, Father: Unknown Meds: HCTZ, OCP SHx: married, (+) smokes 1pk/day Vitals: BP = 130/70, P = 105, RR = 28, T = 99.4 PE: Appears uncomfortable, non-toxic. Abdomen: soft, slightly distended and tympanic, rare BS, no rebound/guarding. Rectal exam = brown stool, heme (+) Labs: WBC = 15.5, H/H = 10.5/31.5, HCO3 = 18. A. Chronic mesenteric ischemia B. Acute mesenteric arterial thrombosis C. Ischemic colitis D. Acute mesenteric venous thrombosis Intestinal (Mesenteric) Ischemia: Remember Mechanisms A. Low-flow state: ischemic colitis B. Mesenteric venous thrombosis: Think – hypercoaguable states C. Arterial occlusion 1. Atherosclerosis:“intestinal angina” 2. Thromboembolic phenomenon: - “Pain out of proportion to physical findings” Pediatric Surgery

A.Intussusception B.Meckel’s diverticulum C.Hypertrophic pyloric stenosis D.Necrotizing enterocolitis (NEC) Intussusception • Sex: Male > female; 3:1 • Etiology: unclear. Likely lead point => invagination • Peak age: 5-10 months • Presentation: Classic triad (Note: triad only present 30% of cases) - A) vomiting, B) abdominal pain, C) “currant jelly” (bloody) stool - Abdominal pain: colicky, severe, intermittent • Abdominal exam - R-sided “sausage”-like mass, Empty RLQ (Dance sign) • Diagnosis: U/S, barium enema • Treatment: Barium, water-soluble or air-contrast enema Lead Points: Adults = polyps, cancer Child = idiopathic - ? Peyer’s patch Meckel’s Diverticulum: “The Rule of 2s” • 2% of the population harbor it – Complications occur in minority (4-16%) • 2x more common in males • Within 2 feet of ileocecal valve • Presentation: with one of 2 complications 1.Children: Painless bleeding due to heterotopic gastric tissue 2.Adults: Obstruction

• Diagnosis: technetium-99m pertechnetate scan – Known as a Meckel’s scan (80-90% sensitive) Hypertrophic Pyloric Stenosis

• Etiology: Marked hypertrophy and hyperplasia of the muscular layers of the pylorus --> leading to narrowing of the gastric antrum. • Sex: Male > female; 4:1 • Peak age: 3 weeks (Range: 1-18 weeks) • Risk factor: erythromycin, azithromycin • Presentation: nonbilious vomiting/regurgitation – after which the infant is still hungry – Dehydration and poor weight gain can occur • Abdominal exam: RUQ “olive” • Diagnosis: Ultrasound • Treatment: Surgery Necrotizing Enterocolitis (NEC) • Incidence: increases with prematurity • Time of onset – ≤ 30 weeks EGA: avg. time of onset = 20.2 days – 31-33 weeks EGA: avg. time of onset = 13.8 days – ≥ 34 weeks EGA: avg. time of onset 5.4 days – Term infants: usually develop in first 1-3 days of life • Presentation: starts with feeding intolerance, then--> vomiting, diarrhea – Clinical triad may ultimate develop a) , b) bloody stools, and c) *** (pathognomonic) • Treatment: NGT decompression, antibiotics, TPN Thank you! Answers

1.C 2.A 3.B 4.B 5.B Supplemental Slides Pneumoperitoneum: Mimics

• Subphrenic abscess • : Bowel interposed between liver and diaphragm

• Pneumoperitoneum: Benign etiology S/P Laparoscopy/laparotomy: air remains up to 6 days

Image reprinted with permission from Medscape Reference (http://emedicine.medscape.com/), 2014, available at: http://emedicine.medscape.com/article/372053-overview Table 1: Diagnostic Criteria from the Rome III Guidelines for Functional Gallbladder Disorder Must include episodes of pain located in the epigastrium and/or right upper quadrant and all of the following: • - Gallbladder is present • - LFT’s, bilirubin, amylase/lipase are normal • - Episodes last longer than 30 minutes • - Recurrent symptoms occur at different intervals (not daily) • - The pain builds up to a steady level • - The pain is moderate to severe enough to interrupt daily activities or lead to ED visit • - The pain is not relieved by bowel movement • - The pain is not relieved by positional change • - The pain is not relieved by antacids • - Other structural diseases that would explain the symptoms are excluded Acute Colonic Pseudo-Obstruction (ACPV): Olgilvie Syndrome

• Population at risk: older, hospitalized – With a variety of medical and surgical conditions • Perforation (cecum) reported 3-40% • Treatment: – NPO, IV fluids, rectal tube, correct electrolytes – Neostigmine is effective in 85-90% of cases •Lasts 1-2 hours, multiple doses may be needed •Cardiac monitor, atropine at bedside – Colonoscopy if medical therapy fails © 2020 American Academy of Family Physicians. All rights reserved.

All materials/content herein are protected by copyright and are for the sole, personal use of the user.

No part of the materials/content may be copied, duplicated, distributed or retransmitted

in any form or medium without the prior permission of the applicable copyright owner.

This Photo by Unknown Author is licensed under CC BY Selected Issues in Women’s Health

David G Weismiller, MD, ScM, FAAFP Department of Family and Community Medicine University of Nevada, Las Vegas School of Medicine Disclosure Statement It is the policy of the AAFP that all individuals in a position to control content disclose any relationships with commercial interests upon nomination/invitation of participation. Disclosure documents are reviewed for potential conflicts of interest. If conflicts are identified, they are resolved prior to confirmation of participation. Only participants who have no conflict of interest or who agree to an identified resolution process prior to their participation were involved in this CME activity.

All individuals in a position to control content for this session have indicated they have no relevant financial relationships to disclose. Learning Objectives

1. Discuss what is known about risk factors for common malignancies. 2. Recommend methods of screening for cancer in women. 3. Discuss the management of vasomotor instability and atrophic vaginitis. 4. Describe principles of patient selection regarding contraceptives. 5. Discuss the contraindications for various contraceptive methods. 6. State the indications regarding methods of emergency contraception. 7. Summarize the basic steps for the initial evaluation of the infertile couple. 8. Review the diagnosis and management of common breast issues. Cancer • Endometrial • Ovarian • Cervical 1. A 59 yo postmenopausal female presents with a recent onset of painless vaginal bleeding. Her last menses occurred 7 years ago and she has had no bleeding until 1 week ago. She reports that her Papanicolaou tests have always been normal, with the most recent obtained a year ago. A pelvic examination today is normal. Which one of the following management options is the preferred next diagnostic step? A) Colposcopy with endocervical curettage B) Transvaginal ultrasonography C) Saline infusion sonohysterography D) Hysteroscopy Endometrial Cancer - Key Facts

• Most common gynecologic malignancy in the USA () • Abnormal uterine bleeding is the presenting sign in 85% of women with endometrial cancer. − Only 10-20% of postmenopausal women who are evaluated for uterine bleeding are diagnosed with endometrial cancer − Most common cause of postmenopausal bleeding is endometrial atrophy − Ensure that women understand the importance of reporting ANY postmenopausal bleeding. (There is no screening test.) • Strongest association with reduced risk: Combined hormonal contraception use −50% reduction in risk −Protection for 10-15 years after discontinuation Histopathology •Types −Type I, Endometroid adenocarcinoma – 70% of cases • Associated with unopposed estrogen stimulation (endogenous or exogenous) • Generally low-grade tumors −Type II, Papillary serous or clear cell; tend to be high grade – 10% of cases • Poor prognosis • High risk of relapse and metastasis • Associated with 40% of related deaths −Familial tumors – 10% of cases • Commonly found in association with Lynch Syndrome (hereditary nonpolyposis colorectal cancer) −Hyperplasia • Endometrial, 1-3% risk of progression • Atypical, 30-40% of patients with concomitant adenocarcinoma Risk Factors for Type I Uterine Cancer

Factors Influencing Risk Estimated RR

Older age 2-3 Early age at menarche 1.5-2 Residency in North America or Long-term use of opposed 10-20 Northern Europe estrogen 3-18 Tamoxifen use 2-3 Higher level of education or income Obesity 2-5 1.5-2 Estrogen-producing tumor >5 White race 2 History of type 2 diabetes, hypertension, Nulliparity 3 gallbladder disease, or thyroid disease History of infertility 2-3 1.3-3 Menstrual irregularities 1.5 Lynch syndrome 6-20 Late age at natural menopause 2-3 Younger women

•Most common risk factors for development of endometrial cancer in young women: −Increasing BMI −Nulliparity −Irregular menses • Chronic anovulation (e.g., PCOS) •Risk may increase as much as 22X in women younger than 45 whose BMIs are > 35 Unopposed endogenous estrogen

•Adipose tissue −Excessive peripheral conversion of androgens to estrone −Case-control studies have demonstrated a 200- 400% linear increase in risk of endometrial cancer in individual with BMI > 25 Endometrial Cancer in Premenopausal Women? • 20% of cases • ACOG Recommendation - Evaluate oAUB > 45 years oAUB < 45 years and have a history of unopposed estrogen exposure SERMs and Endometrial Cancer

• Although raloxifene has estrogen-like effects on the uterus, it has NOT been shown to increase the risk of endometrial cancer. (SOR A) • Tamoxifen is a selective estrogen receptor modulator that has estrogen-like effects. While it has a protective effect on breast tissue, its effect on the uterus INCREASES the risk of endometrial cancer. (SOR A) Diagnostic Studies

•Type of initial study depends on availability of options and their level of invasiveness, and patient and physician preference −Transvaginal Ultrasonography −Endometrial Sampling −Saline Infusion Sonohysterography −Hysteroscopy Postmenopausal Bleeding: An Evidence-Based Workup

H&P, Pap ±CBC, STD testing* Preferred initial test for a patient with painless postmenopausal bleeding Perform EMB or TVUS

EMB Abnormal TVUS results

Normal Refer for ET ≤ 4 mm results treatment (˃ 95% ET ˃ 4 Uterine sensitivity) mm pathology

Bleeding continues? Atrophic Perform SIS, Refer for endometrium EMB, or treatment hysteroscopy with biopsy No Yes

Begin Can the “content” of the endometrial stripe be Perform TVUS, SIS, observation or hysteroscopy reliably assessed by TVUS? period What Are We Looking for on the Endometrial Sampling? • Cytologic atypia is the SINGLE most important histologic finding. • Only ATYPICAL hyperplasia has a significant risk of developing into endometrial cancer. − 29% progresses to invasion. − Need to rule out cancer if atypia is present. • Endometrial hyperplasia is a BENIGN condition, not a cancer precursor. Triage Guidelines Reproductive Age Women • No cytologic atypia − Simple EMHP with abnormal bleeding • Progestin withdrawal for 6 months, then rebiopsy − Complex (adenomatous) EMHP • Progestin withdrawal, then rebiopsy • Cytologic atypia − High-dose progestins, megestrol, or medroxyprogesterone (Depo-Provera) for 3 months, then rebiopsy Triage Guidelines Postmenopausal Women

• No cytologic atypia − Progestins for 6 months, then rebiopsy − TAH for recurrent EMHP or bleeding • Cytologic atypia (substantial risk of deeply invasive or poorly differentiated cancer) − Hysterectomy Treatment

•Surgery −Hysterectomy −Debulking •Adjuvant radiotherapy •Systemic adjuvant therapy −Chemotherapy −Hormone Therapy Best Practice Recommendations

Clinical Recommendation Evidence Rating

Women older than 65 years should be informed of the risks and symptoms of endometrial cancer and advised to seek evaluation C if symptoms occur. Women with abnormal uterine bleeding should be evaluated for endometrial cancer if they are older than 45 years or if they have C a history of unopposed estrogen exposure.

In postmenopausal women, the endometrial thickness on transvaginal ultrasonography should be less than 4 to 5 mm. With thickness above this level, biopsy should be considered to rule C out endometrial hyperplasia or cancer. Ovarian Cancer

• The most lethal of the gynecologic malignancies − Epithelial ovarian is the most common type (90%) − Most commonly diagnosed in those 55 to 64 years of age − 10-12% of cases are genetically predisposed • Majority have no identifiable risk factors (90%) - Lifetime risk of 1.3% • Highest risk factor: genetic predisposition − 90% are inherited mutations in either BRCA1/2 genes • Assess response to treatment by serial Ca-125 levels − 90% correlation with disease progression Ovarian Cancer Risk Factors Summary • Increased Risk • Decreased Risk • Delayed childbearing • Breastfeeding > 18 months • Early menarche • Early menopause • Endometriosis • Multiparity (risk decreases with each additional pregnancy) • ERT > 5 years • Salpingectomy • Family history suggesting genetic predisposition • Late menarche • Genetic syndromes • Low-fat diet • High-fat diet • OCP use • Late menopause • DMPA • Low parity • Tubal ligation • Obesity or weight gain BEST EVIDENCE Ovarian Cancer Prevention • OCPs and a decreased incidence − 40% reduction among all users − 50% reduction with use > 5 years − Up to 80% reduction with use > 10 years − Protective effect persists up to 15 years after discontinuation. − 50% in BRCA-mutation carriers with long-term use (> 4 years) • DMPA use • Breastfeeding − Each month of breastfeeding results in a 1%-2% decrease. • Tubal sterilization − Decreases risk by ~ 18%-40%; mechanism for the protective effect is unknown; some experts theorize it stops carcinogens from reaching the ovaries after they enter the body via the vagina. • Avoid talc powders in genital hygiene. • Greater than one full-term pregnancy prior to age 35 • Prophylactic oophorectomy Pathogenesis

• Epithelial ovarian cancer o 90% of ovarian cancers o 90% of deaths from ovarian cancer • Traditional view – epithelial ovarian cancer derives from elements of the ovary • New view – epithelial ovarian cancer derives from elements of the fallopian tube and endometrium • Research has also pointed to a protective effect of tubal ligation against endometroid and clear cell , suggesting these tumors may arise from retrograde menses Pathogenesis

• Lim and Oliva – 2013 − Lining of the adjacent fallopian tubes – especially their fimbriated ends – can give rise to pre- and then fully malignant transformation with sloughing, seeding, invaginations, and formation within the adjacent ovary − Bilateral salpingectomies prior to the full fruition of this fallopian epithelial process, most serous ovarian cancers can be prevented before they begin

Lim D, Oliva E. Precursors and pathogenesis of ovarian carcinoma. Pathology. 2013;45:229-242. ACOG Opinion Committee on Gynecologic Practice Obstet Gynecol. 2015;125:279-281 •Opinion specifically addresses women at “population risk” for ovarian cancer – women without a genetic risk – but who are having routine pelvic surgery for benign disease •“Salpingectomy at time of hysterectomy or as a means of tubal sterilization appears to be safe, without an increase in complications…compared with hysterectomy alone or tubal ligation.” ACOG Recommendations - 2015

• In women at population risk for ovarian cancer, surgeons should discuss the potential benefits of salpingectomy • In women considering laparoscopic sterilization, physicians can discuss the fact that bilateral salpingectomy provides effective contraception, while pointing out that this procedure eliminates the option of tubal reversal • Prophylactic salpingectomy may prevent ovarian cancer in some patients • More RCTs are needed to support the use of salpingectomy in reducing ovarian cancer Evaluation

• Transvaginal ultrasonography − Test of choice for suspected adnexal mass − Looking for: evidence of metastatic disease, suspicious or persistent complex adnexal mass, increased ovarian volume (>20 mL in premenopausal; >10 after menopause) • Consider CA 125 − > 200 U per mL in premenopausal women − Any elevation in postmenopausal women Treatment

• Surgery • Chemotherapy oPostsurgical adjuvant chemotherapy (intraperitoneal and intravenous) for late-stage disease; not indicated for disease confined to the ovary oNeoadjuvant chemotherapy has no advantage over postsurgical initiation • Evidence does not support routine maintenance chemotherapy following the primary course Screening

• USPSTF recommends AGAINST screening − (Ultrasound and CA-125) • High-risk family history (two or more first- or second- degree relatives with a history of ovarian cancer or a combination of breast and ovarian cancer, or a woman of Ashkenazi Jewish descent with a first-degree relative or two second-degree relatives on the same side of the family) with breast OR ovarian cancer − Offer referral for genetic counseling and, if appropriate, genetic testing Best Practice Recommendations Clinical Recommendation Evidence Rating Women should undergo diagnostic imaging with transvaginal ultrasonography if there is strong clinical suspicion for ovarian cancer C based on clinical presentation or a pelvic mass. The USPSTF and the AAFP recommend against routine screening for ovarian cancer in asymptomatic women. C The USPSTF and the AAFP recommend that women with a family history associated with an increased risk of harmful BRCA A mutations be referred for genetic counseling. The American College of Physicians recommends against routine screening pelvic examinations in asymptomatic women. C Cervical Cancer Cervical cancer is an STD caused by HPV!

• Worldwide – the fourth leading cause of cancer deaths in women − Developing countries – second most common cause of cancer deaths in women • Screening programs have dramatically reduced the rate in screened population. − 70% reduction in the US over the past five decades • Condoms appear to be an effective barrier against HPV transmission. − HPV infection is primarily transmitted through contact with infected skin or mucosal surfaces. 2. As part of routine care for a 33 yo female, you obtain a Pap test for cervical cancer screening. The cytology results are negative for intraepithelial neoplasia, and the sample is positive for the presence of HPV but negative for serotypes 16 and 18. Which one of the following is the most appropriate management for this patient? A. Immediate colposcopy B. Repeat HPV testing in 6 months C. Repeat Pap and HPV testing in 12 months D. Repeat Pap and HPV testing in 36 months Screening for Cervical Cancer ACS, ASCCP, ACCP – 2012*; Group USPSTF – 2018* ACOG 2013* Women ages 21-29 Screen with cytology q 3 years. (A) Screen with cytology q 3 years. (A)

Women ages 30-65 Screen with cytology alone q 3 years Screen with co-testing OR HPV testing alone q 5 years. (A) (cytology/HPV) q 5 years (preferred) or cytology q 3 y. (A) Women < age 21 Do not screen. (D) Women > 65 who have had Do not screen. (D) adequate prior screening and are The ACS/ASCCP/ASCP/ACOG guidelines define adequate prior screening: 3 NOT at high risk consecutive (-) cytology results or 2 consecutive (-)HPV results within 10 y before cessation of screening, with the most recent test occurring within 5 y.

Women after hysterectomy with Do not screen. (D) removal of cervix AND no history of HGSIL or cervical CA Women < age 30 Do not screen with HPV testing (alone or with cytology) (D) *This recommendation statement applies to all women who have a cervix, regardless of sexual history Best Practices in Oncology: Recommendations from the Choosing Wisely Campaign Society of Gynecologic Oncology • Do not perform Papanicolaou tests for surveillance of women with a history of endometrial cancer

Source: http://www.choosingwisely.org. For supporting citations and to search Choosing Wisely recommendations relevant to primary care, see http://www.aafp.org/afp/recommendations/search.htm. Cytology Negative, but HPV Positive FDA Panel: HPV DNA Test Before Pap for Cervical Cancer Screening (24 April 2014) •Approval for the cobas human papillomavirus (HPV) test (Roche) alone for primary cervical cancer screening of women aged > 25 years. −Using a sample of cervical cells, the cobas HPV Test detects DNA from 14 high-risk HPV types; including HPV 16 and 18. −Previously, FDA-approved (2011) for use along with cervical cytology in women aged > 30 years to screen for the presence or absence of high-risk HPV types, 16 and 18; also used as a follow-up test in patients aged 21 years and older with abnormal cytology results. •Recommended indication is for first-line primary cervical screening to detect high-risk HPV, including genotyping for genotypes 16 and 18. …and what is the follow-up?

•Women who test negative for high-risk HPV types should have follow-up in accordance with the physician's assessment of screening and medical history, other risk factors, and professional guidelines. •Women who test positive for HPV genotypes 16 and/or 18 by the cobas HPV Test should be referred to colposcopy. •Women who test high-risk HPV positive and 16/18 negative by the cobas HPV Test (12 other [high-risk] HPV positive) should be evaluated by cervical cytology to determine the need for colposcopy. More Sensitive Than Cytology? Ronco G et. al., http://www.thelancet.com/journals/lancet/article/PIIS0140-6736(13)62218-7/fulltext

•Prospective Cohort Study – 47,208 women - Cobas HPV test (candidate) and Cytology at baseline (comparator) •Disease evaluation - Abnormal cytology - Positive cobas HPV test - Randomly assigned subset of patients with high-risk HPV negative and normal cytology results •The candidate was also tested against the currently recommended cervical cancer screening algorithm, which includes cytology testing on all women and HPV testing on a subset of women according to their age and cytology results Results

Candidate (95% CI) Comparator (95% CI) Sensitivity for > CIN3 58.26% (44.02-74.37) 42.63% (31.75-55.41) PPV > CIN3 in women referred to 12.25% (10.69-13.91) 6.47% (5.54-7.50) colposcopy Risk for > CIN3 in those not 0.42% (0.20-0.74) 0.59 (0.36-0.92) referred for colposcopy False (+) rate for > CIN3 4.09% (3.89-4.28) 6.04 (5.81-6.27) ✓3 very different options that have very potentially different intervals, different triage trees ✓The challenge is now there for professional societies...to put together data- driven, evidence-based algorithms to the extent that there is data, and then go forward with robust education for providers and for patients Next Step…

•ASCCP and SGO have developed a a guidance document – based on the final ATHENA study data being published Guidance Report • http://www.asccp.org/Portals/9/docs/News/HPV%20Guidance%2 0Doc%20Article_main.pdf ATHENA Study • http://www.asccp.org/Portals/9/docs/News/ATHENA- article_YGYNO_975716.pdf Use of primary high-risk human virus testing for cervical cancer screening: Interim clinical Guidance Huh WK, et. al. Gynecologic Oncology. 2015;136:178-182

• Rescreening after a negative primary hrHPV screen should occur no sooner than every 3 years • Primary hrHPV screening should begin 3 years after the last negative cytology and should not be performed automatically at 25 if screening is up to date. Gynecological Conditions: Screening With the Pelvic Examination USPSTF – MARCH 2017 What this statement DOES NOT mean… • The USPSTF has clarified that it is recommending neither for nor against screening with pelvic examination for gynecologic conditions other than cervical cancer, gonorrhea, or chlamydia. American College of Physicians July 2014

• Recommendation: The American College of Physicians recommends against performing screening pelvic examination in asymptomatic, nonpregnant, adult women (strong recommendation, moderate-quality evidence). • Current evidence shows that harms outweigh any demonstrated benefits associated with the screening pelvic examination. • Indirect evidence showed that screening pelvic examination does not reduce mortality or morbidity rates in asymptomatic adult women, as 1 trial showed that screening for ovarian cancer with more sensitive tests (transvaginal ultrasonography and CA-125) also did not reduce mortality or morbidity rates. Because CA-125 and transvaginal ultrasonography found all cancer detected by the screening pelvic examination as well as additional cancer and this earlier detection did not lead to a reduction in morbidity or mortality rates, the guideline authors conclude that the screening pelvic examination alone would also not reduce morbidity or mortality rates. • No studies assessed the benefit of pelvic examination for other gynecologic conditions, such as asymptomatic pelvic inflammatory disease, benign conditions, or gynecologic cancer other than cervical or ovarian cancer. • Low-quality evidence that screening pelvic examination leads to harms, including fear, anxiety, embarrassment, pain, and discomfort, and possibly prevents women from receiving medical care. • False-positive screening results can lead to unnecessary laparoscopies or laparotomies. • Note that this guideline is focused on screening asymptomatic women; full pelvic examination with bimanual examinations is indicated in some non-screening clinical situations. • This guideline does not address women who are due for cervical cancer screening. However, the recommended cervical cancer screening examination should be limited to visual inspection of the cervix and cervical swabs for cancer and human papillomavirus and should not entail a full pelvic examination. Qaseem A, Humphrey LL, Harris R, Starkey M, Denberg TD, Clinical Guidelines Committee of the American College of Physicians. Screening pelvic examination in adult women: a clinical practice guideline from the American College of Physicians. Ann Intern Med. 2014 Jul 1;161(1):67-72. Resources

• American Society for Colposcopy and Cervical Pathology: www.asccp.org • Algorithms • Applications Essential Changes From 2006 Guidelines

• Cytology reported as negative but lacking endocervical cells − managed without early repeat. • CIN 1 on endocervical curettage should be managed as CIN 1, NOT as a positive ECC. • Cytology reported as unsatisfactory requires repeat even if HPV negative. • Genotyping triages HPV-positive women with HPV type 16 or type 18 to earlier colposcopy only after negative cytology; colposcopy is indicated for all women with HPV and ASC- US, regardless of genotyping result. Essential Changes From 2006 Guidelines

• For ASC-US cytology, immediate colposcopy is NOT an option. The serial cytology option for ASC-US incorporates cytology at 12 months, NOT 6 months and 12 months, and then if negative, cytology every 3 years. • HPV-negative and ASC-US results should be followed with co- testing at 3 years rather than 5 years. • HPV-negative and ASC-US results are insufficient to allow exit from screening at age 65 years. • The pathway to long-term follow-up of treated and untreated CIN 2+ is more clearly defined by incorporating co-testing. Essential Changes From 2006 Guidelines

• More strategies incorporate co-testing to reduce follow-up visits. Pap-only strategies are now limited to women younger than 30 years, but co-testing is expanded even to women younger than 30 years in selected circumstances. • Women aged 21-24 years are managed conservatively. Prevention

• Condom use − May reduce the risk for HPV-associated diseases (eg, genital warts and cervical cancer) − Use associated with higher rates of regression of CIN and clearance of HPV infection in women; regression of HPV-associated penile lesions in men − Limited number of prospective studies have demonstrated a protective effect of condoms on the acquisition of genital HPV • Tobacco cessation − Quit • Vaccination – Gardasil 9 HPV Vaccine - 2019

• Begin series BEFORE age 15 (well known - antibody response STRONGER in young children) −Two dose vaccine series −Time zero and 6-12 months • Routine vaccination at age 11-12 −Can begin as young as age 9 REGARDLESS of whether they have a history of sexual assault or abuse (Starting at a younger age helps take the question of sexual activity out of the discussions?) • To be considered immunized, 5 or more months MUST have passed between the first and second doses, other wise third dose should be given at 6 months • Immunocompromised persons (regardless of age) and ANYONE starting series AFTER age 15, 3 doses (Time 0, 1-2 months, six months) Dobson SR, McNeil S, Dionne M, et. Al. Immunogenicity of 2 doses of HPV vaccine in younger adolescents vs 3 doses in young women: a randomized clinical trial. JAMA. 2013;309:1793-1802. Best Practice Recommendations ✓ Data does not support cervical cancer screening in girls and young women < 21 years of age. ✓ Do not screen women for cervical dysplasia after hysterectomy with removal of cervix AND no history of HGSIL or cervical CA. ✓ Do not perform Papanicolaou tests for surveillance of women with a history of endometrial cancer. ✓ Adolescent prevention programs should focus on prevention of HPV infection through universal HPV vaccination. ✓ The nonavalent human papillomavirus vaccine may be considered in males and females 9 to 26 years of age to prevent genital warts and cervical and anal cancers. Menopause •Vasomotor Instability •Atrophic vaginitis Hot Flashes

• After menopause, up to 85% of • Probably hypothalamic women experience hot flashes as a origin result of vasomotor instability. o Menopause o Thyroid disease o Panic or anxiety disorder o o Autoimmune disorders o Pheochromocytoma o syndrome o Tamoxifen and raloxifene Influences on Hot Flashes

•Cultural − More prevalent in African American and Latin American women than in white women − Less common in Chinese and Japanese women •Other variables associated with increased reporting of hot flashes − Cigarette smoking − Potential risk factors with inconsistent association • Maternal history • Early age of menarche and menopause onset • History of irregular menses • Higher BMI • Alcohol use • Hot/humid weather HORMONAL MEDICATIONS EFFECTIVE IN TREATING VMS • Systemic HT with estrogen alone or in combination with progestin, is THE MOST EFFECTIVE therapy for vasomotor symptoms related to menopause −Orally or transdermally • Data DO NOT SUPPORT the use of the following: −Progesterone-only medications −Testosterone −Compounded bioidentical hormones NAMS Position Statement on HT - 2017

• Most effective treatment for vasomotor symptoms (VMS) and the genitourinary syndrome of menopause (GSM) • Has been shown to PREVENT bone loss AND fracture • Risks differ depending on type, dose, duration of use, route of administration, timing of initiation, AND whether a progestogen is used • Treatment should be individualized to identify the most appropriate HT type, dose, formulation, route of administration, and duration of use, using the best available evidence to maximize benefits and minimize risks, with periodic reevaluation of the benefits and risks of continuing or discontinuing HT. NAMS Position Statement on HT - 2017

•Women aged < 60 years OR who are within 10 years of menopause onset and have no contraindications, the benefit-risk ratio is most favorable for treatment of bothersome VMS and for those at elevated risk for bone loss or fracture. •For women who initiate HT > 10 or 20 years from menopause onset or are aged >60 years, the benefit-risk ratio appears less favorable because of the greater absolute risks: coronary heart disease, stroke, venous thromboembolism, and dementia. •Longer durations of therapy should be for documented indications such as persistent VMS or bone loss, with shared decision making and periodic reevaluation. FDA Indications • Indications for hormone therapy approved by the U.S. Food and Drug Administration (FDA) in menopausal women are limited to the treatment of menopausal symptoms and the prevention of postmenopausal osteoporosis • An FDA-issued black box warning indicates that estrogen therapy, with or without progestin, should be prescribed at the lowest effective dose and for the shortest duration consistent with the patient's treatment goals and risks.

Pfizer Inc. Premarin [package insert]. http://labeling.pfizer.com/showlabeling.aspx?id=131. 2014. Accessed July 29, 2018 Hormonal Treatment Options Treatment Dosage/Regimen Evidence of FDA Approved benefit* Estrogen alone or combined with progestin • Standard Dose Conjugated estrogen 0.625 mg/d Yes Yes Micronized estradiol-17β 1mg/d Yes Yes Transdermal estradiol-17β 0.0375-0.05 mg/d Yes Yes

• Low Dose Conjugated estrogen 0.3-0.45 mg/d Yes Yes Micronized estradiol-17β 0.5mg/d Yes Yes Transdermal estradiol-17β 0.025 mg/d Yes Yes

• Ultra-Low Dose Micronized estradiol-17β 0.25 mg/d Mixed No Transdermal estradiol-17β 0.014 mg/d Mixed No Estrogen combined with estrogen Conjugated estrogen 0.45 mg/d and Yes Yes agonist/antagonist bazedoxifene 20mg/d Progestin Depot medroxyprogesterone acetate No No Testosterone No No Tibolone (Synthetic steroid) 2.5 mg/d Yes No Compounded bioidentical hormones No No *Compared with placebo Selected Estrogen and Progestin Preparations for the Treatment of Menopausal Vasomotor Symptoms Preparation Generic Name Brand Name Doses (mg/day) Estrogen ¶ Oral Conjugated estrogens Premarin 0.3, 0.45, 0.635, 1.25 17β-Estradiol Estrace 0.5, 1.0, 2.0 Transdermal 17β-Estradiol Alora 0.025, 0.05, 0.075, 0.1 (patch applied twice weekly) Climara 0.025, 0.0375, 0.05, 0.075, 0.1 (patch applied weekly) Vaginal Estradiol acetate Femring vaginal ring ± 0.05, 0.1 (inserted every 90 days)

Progestogen Oral MPA Provera 2.5, 5.0, 10.0 Micronized progesterone Prometrium 100, 200 (in peanut oil) Vaginal Progesterone Prochieve 4% 45 Combination preparation Oral sequential € Conjugated estrogens and MPA Premphase 0.625 conjugated estrogens plus 5.0 MPA Oral continuous ¥ Conjugated estrogens and MPA Prempro 0.625 conjugated estrogens plus 2.5- 5.0 MPA; 0.45 conjugated estrogens plus 2.5 MPA; or 0.3 or 0.45 conjugated estrogens plus 1.5 MPA; Transdermal continuous ¥ 17β-estradiol-norethindrone Activella 1.0 estradiol plus 0.5 norethindrone acetate 17β-estradiol-levonorgestrel Climara Pro 0.045 estradiol plus 0.015 levonorgestrel (patch applied weekly)

17β-estradiol-norethindrone CombiPatch 0.05 estradiol plus 0.14 or 0.25 acetate norethindrone (patch applied twice weekly) Bioidentical Hormones

• Plant derived, chemically similar or structurally identical to those produced by the body −FDA Approved • Micronized progesterone • Estradiol • Estrone −Non-FDA regulated • Compounded preparations; purity, potency, and quality are concern • Overdosage and underdosage possible because of variable bioactivity and bioavailability NAMS Position Statement 2017 (Bioidentical HT) • Compounded bioidentical HT presents safety concerns such as minimal government regulation and monitoring, overdosing or underdosing, presence of impurities or lack of sterility, lack of scientific efficacy and safety data, and lack of a label outlining risks. • Salivary hormone testing to determine dosing is unreliable. • Prescribers of compounded bioidentical HT should document the medical indication for compounded HT over government-approved therapies, such as allergy or the need for dosing or a formulation not available in FDA-approved products Testosterone? • In combination with HT has been investigated for treatment of menopausal symptoms −No benefit −Potential adverse effects • Detrimental effects on lipid parameters • Clitoromegaly • Hirsutism • Acne • In combination with HT for postmenopausal women −Improved sexual function scores −Improved number of satisfying sexual episodes • Alone, NOT FDA-approved for use in women

Somboonporn W et al., Testosterone for peri and postmenopausal women. Cochrane Database of Systematic Reviews 2005, Issue 4. Art. No. CD004509. Menopausal Hormone Therapy for the Primary Prevention of Chronic Conditions – 2017 • The USPSTF recommends against the use of combined estrogen and progestin for the prevention of chronic conditions in postmenopausal women. Grade: D Recommendation • The USPSTF recommends against the use of estrogen for the prevention of chronic conditions in postmenopausal women who have had a hysterectomy. Grade: D Recommendation Risks * -Women aged 65 years and older (Majority of trials – conjugated equine estrogen alone or in combination with medroxyprogesterone acetate) Estimated Event Rate Difference Associated with Combined Estrogen and Progestin Use vs. Placebo in Postmenopausal Women

Outcome Absolute event rate difference per 10,000 woman-years Harms Breast Cancer (invasive) 9 Coronary Heart Disease 8 Dementia (probable)* 22 Gallbladder disease 21 Stroke 9 Venous thromboembolism 21 Urinary incontinence 876 Risks (Majority of trials – conjugated equine estrogen alone or in combination with medroxyprogesterone acetate)

Estimated Event Rate Difference Associated with Combined Estrogen and Progestin Use vs. Placebo in Postmenopausal Women

Outcome Absolute event rate difference per 10,000 woman-years Benefits Diabetes -14 All fractures (hip and vertebral) -44 Colorectal cancer -6

Gartlehner G, Patel S, Viswanathan M, et al. Hormone Therapy for the Primary Prevention of Chronic Conditions in Postmenopausal Women: An Evidence Review for the US Preventive Services Task Force. Evidence synthesis no. 155. AHRQ publication no. 15-05227-EF-1. Rockville, Md.: Agency for Healthcare Research and Quality; 2017. Risks (Majority of trials – conjugated equine estrogen alone or in combination with medroxyprogesterone acetate)

Estimated Event Rate Difference Associated with Estrogen Use Alone vs. Placebo in Postmenopausal Women

Outcome Absolute event rate difference per 10,000 woman-years Harms Dementia (probable)* 12 Gallbladder disease 30 Stroke 11 Venous thromboembolism 11 Urinary incontinence 1,261 * -Women aged 65 years and older Risks (Majority of trials – conjugated equine estrogen alone or in combination with medroxyprogesterone acetate)

Estimated Event Rate Difference Associated with Estrogen Use Alone vs. Placebo in Postmenopausal Women

Outcome Absolute event rate difference per 10,000 woman-years Benefits Breast cancer (invasive) -7 All fractures (hip and -53 vertebral) Diabetes -19

Gartlehner G, Patel S, Viswanathan M, et al. Hormone Therapy for the Primary Prevention of Chronic Conditions in Postmenopausal Women: An Evidence Review for the US Preventive Services Task Force. Evidence synthesis no. 155. AHRQ publication no. 15-05227-EF-1. Rockville, Md.: Agency for Healthcare Research and Quality; 2017. 3. A 52 yo patient currently takes no prescribed or over- the-counter medications and declines estrogen replacement therapy. Which one of the following would be most effective for relieving this patient’s menopausal symptoms?

A. Venlafaxine (Effexor) B. Propranolol C. Soy protein D. Vitamin E Nonhormonal Medications Therapy Evidence of Benefit in treating VMS FDA Approved for treatment of VMS SSRIs and SSNRIs Yes, but less effective than HT for vasomotor Paroxetine mesylate (7.5 mg/d) is the ONLY symptoms nonhormonal therapy FDA approved (6/2013) Clonidine Small benefit compared to placebo; less benefit No compared to HT Gabapentin Yes; but HT more effective Phytoestrogens (plant- No No derived substances with [Lethaby A et al., Phytoestrogens for VMS. estrogenic biologic activity; Cochrane Database of Systematic Reviews e.g., isoflavones, genistein, 2007, Issue 4. Art. No.:CD001395.] daidzein) found in soybeans, soy products, red clover Herbal remedies e.g. Insufficient data to support use of herbal No ginseng, black cohosh, remedies ginko biloba Vitamins Limited data; one less hot flush per day has No been reported with Vitamin E (800 IU/d) Menopausal Symptoms – Nonhormonal Treatment Options Treatment Dose (orally daily) Side Effects Superior to Placebo Conventional Clonidine 0.1-0.4 mg Drowsiness, dry mouth, constipation, Yes; no long-term studies hypotension, insomnia, postural hypotension, reaction to skin patch; to discontinue slowly reduce dose to avoid rebound hypertension, headaches, and agitation SSRIs Dry mouth, insomnia, sedation, decreased Fluoxetine 20-30 mg appetite, constipation Inconsistent effect; few studies, side effects profile significant, no long-term studies ParoxetineXR 12.5-25 mg Yes; no long-term studies Yes; efficacy increases with higher doses but so do Venlafaxine XR 37.5-75 mg side effects. No long-term studies. Gabapentin 900 mg in divided doses Somnolence, fatigue, dizziness, and Yes; no long term studies palpitations Alternative Black Cohosh 2-4 mg of terpene glycoside or 4 mL of tincture Diarrhea and vomiting, skin rash, Yes, but inconclusive results, long-term safety not dizziness, headaches, weight gain. Case known; caution with estrogen-dependent tumors reports on autoimmune hepatitis and asthenia Red Clover 40-80 mg Headache, myalgia, arthralgia, nauseam Inconsistent effect; few studies, long-term safety diarrhea. Contraindicated in bleeding unknown disorders Phytoestrogens 34-100 mg Mastalgia, weight gain, a case report of Inconclusive hypertensive crisis Ginseng 200mg Insomnia, mastalgia, skin eruptions, GI, Few studies interacts with warfarin

Evening Primrose 500 mg Gastrointestinal, headache, lowers Only one randomized study seizure threshold in epileptics

Dong quai 450 mg Photosensitization, warfarin interaction Only one randomized study

Vitamin E 50-100 mg Diarrhea, abdominal pain Only one randomized study Alternative Techniques

• Acupuncture −No benefit over placebo • Reflexology −No benefit compared with nonspecific foot massage • Local injection of anesthetic into the stellate ganglion1 −May reduce VMS in women with contraindications to HT −More studies needed 1Kontos M, et al. Climacteric 2010;13:4-21 Best Practice Recommendations

• Systemic HT, with estrogen alone or in combination with progestin, is the most effective therapy for vasomotor symptoms related to menopause. (SOR: A) • Given the variable response to HT and the associated risks, it is recommended that health care providers individualize care and treat women with the lowest effective dose for the shortest duration that is needed to relieve symptoms. (SOR: C) • The decision to continue HT should be individualized and be based on a woman’s symptoms and the risk-benefit ratio, REGARDLESS OF AGE. (SOR: C) • Combined estrogen/progesterone therapy, but NOT estrogen alone, increases the risk of breast cancer after three to five years of use (SOR: B) • Data do not support the use of progestin-only medications, testosterone, or compounded bioidentical hormones for the treatment of vasomotor symptoms. • Vaginal estrogen therapy preferred over systemic therapy for women treated solely for vaginal atrophy symptoms. (SOR: B) Atrophic Vaginitis Definition

• Atrophic vaginitis (also known as vaginal atrophy or urogenital atrophy) - inflammation of the vagina (and the outer urinary tract) due to the thinning and shrinking of the tissues, as well as decreased lubrication. Symptoms are due to lack of estrogen. • Cause of vaginal atrophy is the decrease in estrogen − Naturally (perimenopause, and increasingly so in post-menopause) − Breastfeeding − Medications intended to decrease estrogen to, for example, treat endometriosis Signs and Symptoms • Genital symptoms: −Dryness, itching, burning, soreness, pressure, white discharge, malodorous discharge due to infection, painful sexual intercourse, bleeding after intercourse −In addition, sores and cracks may occur spontaneously −Atrophic vaginitis is one possible cause of postmenopausal bleeding • Shrinkage of the tissues and loss of flexibility can be extreme enough to make intercourse impossible • Urinary symptoms: −Painful urination, blood in the urine, increased frequency of urination, incontinence, and increased likelihood and occurrence of infections Mechanical Measures

• Sexual activity − Mechanical stretching; increased vaginal blood flow • Dilators − Contraindications to estrogen AND who desire vaginal intercourse − Particularly effective in women who avoid intercourse due to pain − Instruction by physician or pelvic physical therapist Moisturizers and Lubricants • Improve coital comfort and increase vaginal moisture; do not reverse atrophic changes • Moisturizers – use one or more times per week, not just during sexual activity −e.g., Replens, Vagisil Feminine Moisturizer, Feminease, K-Y Silk-E • Lubricant at time of coitus to decrease irritation −Water-based, silicone based, oil-based (find the one that meets your needs) Estrogen Therapy

• Vaginal preferred over systemic for women treated solely for vaginal atrophy symptoms • If receiving systemic estrogen therapy for vasomotor symptoms, low dose vaginal therapy MAY be added if relief of atrophic symptoms is insufficient −Low dose = <50 mcg estradiol or <0.3 mg conjugated estrogens/0.5 g cream Atrophy of Vagina (Dryness) Suckling et al. (Cochrane – 2010)

• Local estrogens equally effective −Vaginal estradiol ring (worn 90 days) −Vaginal estradiol tablets • Can be used in place of somewhat higher doses if there is concern about small amounts of systemic absorption −Vaginal cream (e.g. Premarin) • More uterine bleeding, breast pain than tablets • Significantly more endometrial stimulation than estradiol ring • Recommended (+) uterus use additional progestagen protection when using conjugated equine estrogen cream if dose used results in systemic estrogen absorption (usually doses of estradiol > 0.5 mg/d) • (+) postmenopausal bleeding with local estrogen use - should have endometrial investigation Vaginal Estrogen Preparations

Preparation Available Strengths Regimen Vaginal Ring Estring* 7.5 mcg estradiol/day released over 90 days Inserted into vagina by patient or clinician. Removed and replaced with new ring every 90 days Vaginal Tablet Vagifem* 5-10 mcg estradiol/tablet Insert 1 tablet intravaginally daily for two weeks, followed by twice weekly Vaginal Cream Premarin 0.625 mg conjugated estrogens/g of cream 0.5 g of cream intravaginally administered twice weekly. Estrace 100 mcg estradiol/g of cream 2 to 4 g of cream intravaginally administered daily for 1 or 2 weeks, then gradually reduce to half the initial dosage for a similar period. A maintenance dosage of 1 g of cream, 1 to 3 times per week may be used * Lowest dose options for vaginal estrogen therapy Note: Ring, tablet, or “minidose” of cream (0.25g twice weekly) – no routine opposing progestin to prevent endometrial neoplasia; If needed with use of vaginal cream - opposing progestin – typical regimen – 10-12 days/month; e.g., 10 mg medroxyprogesterone Vaginal estrogen preparations

• Some vaginal preparations administer higher doses of estrogen and some are intended to deliver systemic doses e.g. Femring 50-100 mcg/day • Start with cream if symptomatic vulvar atrophy (e.g., fissures – so that cream may be applied to areas of vulva affected by atrophy – when atrophy improves, switch to vaginal tablet or ring, depending upon patient preference NAMS Position Statement - 2017

• For bothersome GSM symptoms not relieved with over-the-counter therapies and without indications for use of systemic HT, low-dose vaginal estrogen therapy or other therapies are recommended • Vaginal estrogen (and systemic if required) or other nonestrogen therapies may be used at any age for prevention or treatment of GSM. Nonpharmacologic is not sufficient… • Cannot (severe arthritis, obesity, vulvodynia) or prefer not to use a vaginal product o Ospemifene • Women with breast cancer - o First Line – lubricants and moisturizers o On aromatase inhibitors (stop the production of estrogen in postmenopausal women e.g., Arimidex, Aromasin, Femara) – avoid vaginal estrogen therapy o Low risk of recurrence – low dose, in consultation with patient’s oncologist and after explanation of potential risk to the patient • There is no evidence that using low-dose vaginal estrogen increases the risk of breast cancer recurrence Ospemifene • SERM that acts as an estrogen agonist in the vagina and appears to have no clinically significant estrogenic effect in the endometrium or breast. • Approved by FDA 2/2013 for the treatment of moderate to severe dyspareunia caused by vulvovaginal atrophy - 60 mg po daily • Disadvantages compared with vaginal estrogen − Need for daily use − Systemic side effects – hot flashes, potential risk of VTE • Potential benefit – reduction in bone turnover Bachmann GA, et al. Ospemifene effectively treats vulvovaginal atrophy in postmenopausal women: results from a pivotal phase 3 study. Menopause. 2010;17:480. Portman DJ, et al. Ospemifene, a novel SERM for treating dyspareunia associated with postmenopausal vulvar and vaginal atrophy. Menopause. 2013;20:623. Treatment Summary • Regular sexual activity helps maintain vaginal health • Regular use of vaginal moisturizing agents supplemented by water-based lubricants during vaginal intercourse (Grade 2B) • Low dose vaginal estrogen – if treating solely for vaginal atrophy (not for other menopausal symptoms) • Choice of vaginal delivery system (tablet, ring, cream) depends on patient preference • Women using intravaginal estrogenic preparations who have postmenopausal bleeding should have endometrial investigation. • It is recommended that women with a uterus use additional progestagen protection when using conjugated equine estrogen cream if the dose used results in systemic estrogen absorption (usually associated with doses of estradiol greater than 0.5 mg daily). Breast Disease • Mass • Mastitis • Paget disease of the breast 5. A 51 yo woman presents to the office with the complaint of a left . She reports she was in an automobile accident 5 months ago and first noted the mass at that time. She thought it was due to the seat belt she was wearing. On examination you note a 1 cm hard mass in the upper outer quadrant of the left breast. It is mobile and nontender. There is no overlying change in the skin, discharge, or nipple retraction. You review her chart and note she had a negative screening mammogram 6 months ago. Which of the following is the first diagnostic imaging study that should be performed in evaluating this new palpable breast mass? A. Ultrasound B. MRI C. Mammogram D. CT Breast Mass

• May be benign or malignant o ~90% of palpable breast masses in women 20s to early 50s are benign • Benign mass may be solid or cystic • Malignant mass is TYPICALLY solid • Cystic mass with solid components (complex cyst) can also be malignant Differential Benign Malignant • • Infiltrating ductal • Cyst • Accounts for 70-80% of invasive breast cancers • Fibrocystic changes • Infiltrating lobular • Galactocele • Mixed ductal/lobular • Fat necrosis Evaluation of Breast Mass

• Characterization of mass - Location, size, margin irregularity, relationship to the chest wall, density of the breast tissue; evaluation of the skin overlying the mass and nipple • Supraclavicular, cervical, axillary nodes • Breast imaging Imaging • Palpable mass o Unilateral diagnostic mammogram - First imaging study for a woman with a new, palpable breast mass, and should be performed even if recent mammogram was negative - A normal mammogram DOES NOT eliminate the need for further evaluation of a suspicious mass [even though the false (-) rate of mammograms is <5% for clinically palpable breast cancers]. o Ultrasound - Always perform in setting of new palpable abnormality – help differentiate benign cyst from a benign or malignant solid mass - For young women with a clinically benign mass e.g. fibroadenoma and no family history of premenopausal breast cancer, US is a useful initial diagnostic imaging study o MRI is NOT indicated for the work-up of an undiagnosed mass – reserved for diagnostic dilemmas Biopsy • Diagnosis of benign or malignant mass is confirmed by a breast biopsy • Core needle biopsy using image guidance (or FNA with experienced cytopathologist) for ANY mass not identified as a simple cyst • Image guidance ensures adequate localization of the mass and placement of localizing clip for future identification of the mass if required for surgical intervention (open biopsy) 6. A 32 yo breastfeeding female presents to the office with redness over her right breast and a fever of 101F. The redness started 3 days ago and the fever within the past 12 hours. She delivered via cesarean 14 days ago and remained in the hospital for 7 days secondary to the development of endomyometritis. She is non-toxic in appearance. Her exam reveals a hard, red, tender, swollen area over the right breast measuring 4 by 6 cm. There is no axillary adenopathy. The remainder of the examination is normal. Which one of the following would be the most appropriate management option? A. Dicloxacillin B. Clindamycin C. Cephalexin D. Augmentin Mastitis • Majority is lactational; usually secondary to breastfeeding problems • Ultrasound is the most effective method of differentiating mastitis from breast abscess • Most lactation associated breast infections are caused by staphylococcus aureus. MRSA is becoming an increasingly important pathogen in cases of lactational mastitis • Manage initially with systematic emptying of the breast, anti-inflammatory agents and symptomatic treatment to reduce pain and swelling. If symptoms do not improve (48-72h), treatment with antibiotics • Breastfeeding continues during treatment for lactation-associated breast infections. Breast emptying is essential throughout the course of treatment. Mastitis

• Hard, red, tender, swollen area • Fever >38.3º (typically) Clinical Manifestations

• Lactational mastitis typically presents as a hard, red, tender, swollen area of one breast associated with fever >38.3º C in a nursing mother. • Other systemic complaints may variably include myalgia, chills, malaise, and flu-like symptoms. • In the early stages of breast infection the presentation can be subtle with few clinical signs, while patients with advanced infection may present with a large area of breast swelling with overlying skin changes (e.g., erythema). • Reactive lymphadenopathy can also cause axillary pain and swelling. Differential

• Plugged ducts • Galactoceles • Inflammatory breast cancer - In inflammatory breast cancer the skin examination will demonstrate thickening, erythema, peau d'orange, and there is often associated axillary lymphadenopathy. - It is important to rule out inflammatory breast cancer if a suspected breast infection does not respond to antibiotics. Treatment

• Non severe infection, absence of risk factors for MRSA • Dicloxacillin – 500 mg po QID • Cephalexin – 500 mg po QID • Clindamycin – 300 mg po QID • Nonsevere infection with risk for MRSA… Risk factors for methicillin-resistant Staphylococcus aureus (MRSA) ✓ Recent hospitalization • Incarceration • Residence in a long-term • Military service care facility • Sharing needles, razors, or ✓ Recent antibiotic therapy other sharp objects • HIV infection • Sharing sports equipment • Men who have sex with men • Diabetes • Injection drug use ✓ Prolonged hospital stay • Hemodialysis Treatment

• Non severe infection, absence of risk factors for MRSA • Dicloxacillin – 500 mg po QID • Cephalexin – 500 mg po QID • Clindamycin – 300 mg po QID • Non severe infection with risk for MRSA • Clindamycin – 300 mg po QID • Trimethoprom-Sulfamethoxazole – 1-2 tabs po BID • Linezolid – 600 mg po BID • Setting of severe infection (eg, hemodynamic instability, progressive erythema) • Empiric inpatient therapy with vancomycin (15 to 20 mg/kg/dose every 8 to 12 hours, not to exceed 2 g per dose), therapy tailored to culture and sensitivity results • Gram stain results (+) for gram negative rods – empiric antibiotic therapy against these organisms with a third-generation cephalosporin or a combination beta-lactam/beta- lactamase agent Paget Disease of the Breast (PDB)

• 1-3% of new cases of female breast cancer • Presentation - Scaly, raw, vascular or ulcerated lesion on the nipple then spreads to ; (+/-) bloody discharge - Pain, burning, pruritis – may be present before the development of clinically apparent disease • An underlying breast cancer is present in 85% of cases – although often without an associated breast mass or mammographic abnormality Paget disease of the breast presentation

• Various presentations • Paget disease is typified by erythematous, scaly, and weeping "eczema" that involves the nipple. • Discoloration, depigmentation and desquamation of the nipple and areola are sometimes seen. Differential

Benign Malignant • Eczema • Bowen’s disease • Contact dermatitis (Squamous carcinoma of • Nipple adenoma the epidermis) • Basal cell carcinoma • Superficial spreading malignant melanoma Evaluation

• Full thickness wedge or punch biopsy of the nipple - Pathologic hallmark is malignant, intraepithelial adenocarcinoma cells (Paget cells) within the epidermis of the nipple • Bilateral to identify associate mass as well as rule out synchronous cancers • Prognosis is dependent upon the presence of an underlying invasive or axillary node metastasis • PDB presenting with a palpable mass is usually associated with more advanced disease than cases without a palpable mass CONTRACEPTION Combined Hormonal Options

• Pill − Monophasic pills recommended as the first choice for women starting combined OCPs (Cochrane 2005). − 25-year mortality from all causes same for OCP users vs nonusers. • Ring • Patch − Highly efficacious in women < 90 kg − Safety warning (FDA) • ~ 60% more estrogen per cycle than 35 mcg pill (11/2005) • > 3x risk of VTE compared to combined OCP • Inhibit ovulation at pituitary and hypothalamus • Estrogen − Ethinyl estradiol • Progestins − Drospirenone – no risk of hyperkalemia Descending order of androgenic potency Progestins First Second Third Fourth “Original Levonorgestrel Norgestimate Drospirenone pills” Alesse Ortho-Cyclen Yasmin Levlite Ortho-Tri-cyclen Yaz Nordette Desogestrel Anti-androgenic Levlen Mircette Norgestrel Desogen LoOvral Orthocept Norethindrone acetate Cyclessa Loestrin Norethindrone Ortho-novum 1/35 Ovcon 35 Ethynodiol diacetate Demulen 1/35 JAMA, 2001; 285:2232-2239 Prerequisite Preventive Services? Stewart et al. Systematic Review • Consensus statement reviewing and summarizing relevant medical literature and policy statements • No evidence supporting necessity of CBE and pelvic examination • The available evidence supports prescribing hormonal contraception based only on: − Blood pressure measurement − Review of medical history Medical History

• Chronic diseases • Previous contraceptive methods, −Hypertension successes and failures −DM • VTE −Hyperlipidemia −Migraine • Tobacco use −Immune deficiency states −Cancer • Gynecologic history −Infections −Pap Bottom Line

•Requiring prerequisite preventive services, such as cervical cytology; breast examination; or evaluation for sexually transmitted infections, diabetes mellitus, dyslipidemia, liver disease, or thrombophilia, can introduce unnecessary barriers to contraceptive care.

Gavin L, Moskosky S, Carter M, et al.; Centers for Disease Control and Prevention (CDC). Providing quality family planning services: recommendations of CDC and the U.S. Office of Population Affairs. MMWR Recomm Rep. 2014; 63(RR-04): 1-54. Best Practices in Oncology: Recommendations from the Choosing Wisely Campaign American Academy of Family Physicians

• Do not require a pelvic exam or other physical exam to prescribe oral contraceptive medications.

American Academy of Family Physicians. Choosing Wisely: Pelvic exam or physical exams to prescribe oral contraceptive medications. 2014. http: //www.aafp.org/patient-care/clinical-recommendations /all / cw-oral-contraceptives.html. Combined Contraceptives Benefits

• Less risk of ectopic pregnancy • Increases bone mass − Reduces risk of postmenopausal hip fractures • Relieves dysmenorrhea • Improves symptoms of PCOS − High estrogen/progestin ratio • Low-dose pills useful for management of perimenopause Combined Hormonal Contraceptives Decrease • Iron deficiency anemia • Fibrocystic breast disease • Functional ovarian cysts (use high estrogen content/fibroids) • Pelvic inflammatory disease − Cervical mucus/reduced menstrual blood flow − Less retrograde menstruation • Cancer − Ovarian and endometrial (OR 0.57; NNT 60)* cancers • Protective effects persist up to 20 years after discontinuation. − Colorectal cancer (OR 0.86; NNT 132)* • Endometriosis (use strong progestin component) * Gierisch JM et al. Cancer Epidemiol Biomarkers Prev. 2013 Sep 6 [e-pub ahead of print]. (http://dx.doi.org/10.1158/1055-9965.EPI-13-0298) Combined Contraceptives Side Effects (Excess) • Estrogen • Progestin −N/V - Acne −/edema • (Ortho-Tri-Cyclen approved for treatment) −Hypertension - Increased appetite −Migraine HA - Hypertension - Fatigue −Breast tenderness - Depression −Decreased libido - Hirsutism −Weight gain - Vaginal yeast infection −Heavy bleeding −Leukorrhea Combined Contraceptives Side Effects (Deficiency)

• Estrogen • Progestin − Spotting/breakthrough − Amenorrhea − Amenorrhea − Late breakthrough or − Vaginal dryness heavy bleeding Drug Interactions With Combined Hormonal Contraception

• Drugs likely to lead to contraceptive failure - Rifampin – otherwise little effect from antibiotics [SOR:A] - Anticonvulsants (significant effect) – except valproic acid - Antifungals (griseofulvin) - HIV medications Extended vs. 28-day Cycle OCPS Cochrane 2014

• Similar pregnancy rates, safety profiles, compliance • The continuous or extended-cycle and traditional regimens appeared similar, as judged by bleeding, discontinuation rates, and reported satisfaction. • Extended regimens fared better: headaches, genital irritation, tiredness, bloating, and menstrual pain. Progestin Only Options • Pill − Must be taken q day at SAME time (2 h window) • Back-up if > 3 h late • Implant − q 3 years − < 125% IBW • IUS − q 3-5 years • DMPA − q 3 months • No effect on BP; risk of VTE, CVA, MI [SOR: B] • Most common side effect: Irregular bleeding Management of Unscheduled Bleeding in Women Using Contraception Contraceptive Preferred Treatment DMPA • Expectant management • 7-14 days oral estrogen (1.25 mg conjugated estrogen or 2 mg micronized estradiol • Transdermal patch (0.1 mg estradiol/24 h) • 10-20 days of low-dose combined OCP Etonogestrel implant • Expectant management • Low-dose combined OCP for 10-20 days (not studied) • NSAID for 5-7 days

Progestin pills • Take at same time each day and minimize missed doses.

Levonorgestrel IUD • NSAID for 5-7 days (eg, ibuprofen 400 mg, naproxen 250 mg, or mefanamic acid 500 mg TID) Edelman A and Kaneshiro B. Management of unscheduled bleeding in women using contraception. www.uptodate.com, 2013. Depot medroxyprogesterone acetate (DMPA) • Typical failure rate - ~ 6% • Side effects −Weight gain, amenorrhea, hair loss, bone loss • ACOG −No longer recommends limiting injections to 2 years −No routine monitoring of bone density −Recommend • 1300 mg calcium and 600 IU of vitamin D3 daily • Participate in weight-bearing exercise Depot medroxyprogesterone acetate and bone effects. Committee Opinion No. 602. American College of Obstetricians and Gynecologists. Obstet Gynecol 2014;123:1398–402. DMPA •One week to take effect if given AFTER first five days of the period cycle •Return to fertility −3-18 months (avg. 9-10 months) •Benefits −Reduced endometrial cancer – 80% • Thought to be due to both the direct anti-proliferative effect of progestogen on the endometrium and the indirect reduction of estrogen levels by suppression of ovarian follicular development −No increased risk of DVT, PE, CVA, MI −Decreased risk • PID • Ectopic pregnancy • Primary dysmenorrhea • Ovulation pain • Functional ovarian cysts Progestin-Only Methods More Appropriate Than Combined

• Smoking or obesity AND over age 35 [SOR B, A; respectively] • Hypertension with vascular disease or > age 35 [SOR B] • Lupus with vascular disease, nephritis [SOR A] • Migraine with focal aura [SOR B] • Current or personal history of VTE associated with pregnancy or estrogen unless on anticoagulation [SOR A] • Coronary artery/cerebrovascular disease [SOR C] Types of IUDs IUD (Copper) Available Years Use and FDA Approved Possible side effects Since Effective Copper (Paragard) 1988 10 • Approved only in parous women, but • Abnormal menstrual available to all women regardless of bleeding parity • Higher frequency or • Can be used as Emergency intensity of cramps/pain Contraception when inserted within 5 days IUD (Hormonal) Mirena 2001 5 Approved only in parous women, but • Inter-menstrual spotting in available to all women regardless of the early months parity • Reduces menstrual blood Skyla (slightly smaller 2013 3 Approved for women regardless of parity loss significantly than Mirena) • Hormone-related: headaches, nausea, breast Liletta* 2015 3 Approved for women regardless of parity tenderness, depression, cyst formation. Kyleena (lower hormone 2016 5 Approved for women regardless of parity levels than Mirena)

*Actavis in conjunction with Medicines360, a non-profit women’s pharmaceutical company, developed Liletta specifically to be low cost and available to public health clinics enrolled in the national 340B Drug Pricing Program, which provides reduced cost pharmaceuticals to providers that serve low-income populations. Intrauterine Contraceptives Mechanisms of Action

https://commons.wikimedia.org/wiki/File:IUD_with_scale.jpg Levonorgestrel-Releasing Intrauterine Copper-Releasing System (LNG-IUS, Mirena® Skyla® Intrauterine Contraceptive Kyleena®) (ParaGard® T380A)

❖Inhibits fertilization ❖ Inhibits fertilization ❖Thickens cervical mucous ❖ Releases copper ions (Cu2+) that reduce ❖Inhibits sperm function sperm motility ❖Thins and suppresses the endometrium ❖ May disrupt the normal division of oocytes and the formation of fertilizable ova

Jonsson B, et al. Contraception. 1991;43:447-458; Videla-Rivero L, et al. Contraception. 1987;36:217-226; Kulier R, et al. Cochrane Database Syst Rev. 2006;3: CD005347. Considerations for IUDs

• IUD insertion, not IUD use, is associated with PID - Cochrane - Systematic Review (Grimes, Mohllajee) - ACOG Practice Bulletin 2011 • DO NOT cause future infertility • Nulliparas can use an IUD; Uterus sounds to depth of a minimum 6 cm • The USMEC guidelines state that the advantages of using the IUD in adolescents generally outweigh the risks. • Risk of uterine perforation Guidelines for IUDs

Organization Recommendation ACOG 2007 Asymptomatic women may use an IUD within 3 months of treated pelvic infection or septic abortion. ACOG 2007 All adolescents should be screened for GC and chlamydia prior to insertion. Cochrane 2007 No benefit from doxycycline or azithromycin prior to insertion.

CDC 2010 Evidence is insufficient to recommend the removal of IUDs in women diagnosed with acute PID. However, caution should be exercised if the IUD remains in place, and close clinical follow-up is mandatory. The rate of treatment failure and recurrent PID in women continuing to use an IUD is unknown, and no data have been collected regarding treatment outcomes by type of IUD (eg, copper or levonorgestrel). Intrauterine Contraceptives Noncontraceptive Benefits •Intrauterine contraceptives decrease the risk for endometrial cancer •The levonorgestrel-releasing intrauterine system (LNG-IUS) can be used as a first-line option to treat menorrhagia − May be used in the presence of fibroids, unless they significantly distort or enlarge the uterine cavity − Produces a 97% decrease in menstrual blood loss − In a retrospective study, 80% of women who were prescribed the LNG-IUS for menorrhagia chose not to undergo a hysterectomy, as opposed to 9% of women who received normal care for the condition • Levonorgestrel system may be an acceptable alternative to hysterectomy in women with AUB-O Hubacher D, Grimes DA. Obstet Gynecol Survey. 2002;57:120-128; Castellsague X, et al. Int J Cancer. 1993;54:911-916. Strategies to Reduce Barriers and Increase Use of Implants and IUDS

• Encourage implants and IUDS for all appropriate candidates – including nulliparous women and adolescents. • Adopt same-day insertion protocols. − Screening for chlamydia, gonorrhea, and cervical dysplasia SHOULD NOT be required before implant or IUD insertion but may be obtained on the day of insertion, if indicated.

ACOG Committee Opinion No. 450. Increasing use of Contraceptive Implants and Intrauterine Devices to Reduce Unintended Pregnancy. Obstet Gynecol. 2009;114(6):1434-1438 Key Recommendations for Practice Clinical Recommendation Evidence Rating Nulliparous women and adolescents can be offered an IUD, although the 20-mcg per 24 hours levonorgestrel-releasing IUD (Mirena) is not approved by the U.S. Food and Drug Administration for use C in nulliparous women Women who are at high risk of STIs but have no active signs or symptoms of genital tract STI should be tested for STIs at the time of IUD insertion. Insertion of the IUD may occur on the same day as STI testing, without waiting for test results. If results are subsequently found to be positive, treatment can be C administered at that time and the IUD left in place. For women with a known STI that causes cervical infection, it is recommended that IUD insertion be delayed for at least three months after resolution of the infection. C Prophylactic antibiotics should not routinely be administered before IUD insertion. Antibiotic prophylaxis does not have a major effect on reducing the risk of pelvic infection, and does not B alter the need for IUD removal in the months after insertion. Misoprostol (Cytotec) should not be administered before IUD insertion. Although an earlier study showed easier insertion with misoprostol, subsequent studies showed no benefit and increased B side effects. If a woman with an IUD becomes pregnant, the IUD should be removed. C Barrier Methods – Key Points

• Sponge − Does not protect against STIs (according to manufacturer) − Nonmenstrual toxic shock syndrome (sponge, diaphragm, cap) • 2 cases/100,000 users per year • Diaphragm − Increased incidence of UTI • Latex condom − Consistent use results in 80% reduction of HIV − Use only water-based lubricants

Natural Family Planning

• Pluses − No adverse drug effects − No medication or device cost; cannot “run out” of method − Immediately reversible − Acceptable to all major religions − Expands couples’ communication and forms of sexual expression • Minuses − No noncontraceptive benefits of some other methods − Requires periodic abstinence − Requires intensive education ACOG Breastfeeding: Maternal and Infant Aspects Committee Opinion

• All family planning choices are available to the postpartum lactating woman. • Choice and clinical ramifications merit additional counseling. • Support women in choosing breastfeeding − Accurate information − Problems arise • Early discussion of contraception and follow-up − Options to be explained in detail • Nonhormonal methods • Hormonal Methods • Lactational Amenorrhea Method WHO Medical Eligibility Criteria Duration of BF Progestin- Progestin- Progestin- Combined Low-dose method only pills only depots only implants/ patch or ring combined IUD < 6 weeks PP 3 3 3 4 4

> 6 w to < 6 m PP (primarily breastfeed) 1 1 1 3 3

> 6 m PP 1 1 1 2 2

1. No restriction 3. Not usually recommended 2. Generally use 4. Not to be used So How Does Choice Impact Lactation?

• LAM • IUD • Progestin-only • Combined pill • Abstinence/ – Copper – Pills • Patch – Periodic • Sterilization – Injectables • Ring Abstinence/ • Implants – NFP Methods • Levonorgestrel • Barrier Methods IUD No known impact Little to no known Some reports of Expected to have on lactation impact on negative impact negative impact lactation on lactation on lactation 7. According to ACOG, use of emergency contraception is recommended no longer than how many hours after inadequately protected or unprotected intercourse in women who do not desire pregnancy? A. 48 hours B. 72 hours C. 96 hours D. 120 hours Postcoital Treatments for Preventing Pregnancy

Regimen Formulation Timing of use after Access FDA LABELED unprotected sexual FOR USE AS EC intercourse1 Selective 1 tablet, containing 30 Up to 5 days Requires a prescription Yes progesterone receptor mg of ulipristal modulator acetate Progestin only 1 tablet containing 1.5 Up to 3 days(5)5 Available OTC without Yes mg of levonorgestrel age restriction Combined progestin- A variety of Up to 5 days Requires a prescription No3 estrogen pills formulations can be used2 Copper IUD N/A Up to 5 days Requires office visit and No4 insertion by a clinician 1. EC is BEST used ASAP after unprotected sex 2. Variety of formulations of combined OCPs can be used for EC. List of appropriate formulations: http://ec.princeton.edu/questions/dose.html#dose 3. Although not FDA labeled for use as EC, found to be safe and effective when used as EC and can be used off-label for this indication. 4. Most effective method of EC. 5. Off-label Emergency Contraception (EC)

• Prevent fertilization by inhibition of ovulation (hormonal). • Use after implantation does not interrupt an established pregnancy.

Grimes et al. Ann Intern Med. 2002:137 EC Indications ACOG - 2015

• Inadequately protected or unprotected intercourse in women who do not desire pregnancy (SOR A) • No evidence that EC is unsafe for women with contraindications to OCPs or for those with medical conditions • Should be offered up to 120 hours after unprotected intercourse (SOR B) What does the evidence say?

• ELLA is more effective than the levonorgestrel-only regimen and A maintains its efficacy for up to 5 days

• Levonorgestrel-only regimen is more effective than the combined A hormonal regimen and is associated with less nausea and vomiting

• Insertion of a copper IUD is the most effective method of EC A

ACOG Practice Bulletin #152. Obstet Gynecol 2015:126(3):e1-e11. What does the evidence say?

• No clinical examination or pregnancy testing is necessary before B provision or prescription of EC

• EC should be made available to patients up to 5 days after B unprotected or inadequately protected sexual intercourse

• Body weight influences the effectiveness of oral EC. Consider a copper IUD as an alternative in obese women. Oral EC SHOULD B NOT be withheld from women who are overweight or obese

ACOG Practice Bulletin #152. Obstet Gynecol 2015:126(3):e1-e11. What does the evidence say?

• Any EC regimen may be made available to women with contraindications to the use of conventional oral hormonal C contraceptive preparations

• To maximize effectiveness, women should be educated about the C availability of EC in advance of need

• Oral EC may be used more than once, even within the same C menstrual cycle.

ACOG Practice Bulletin #152. Obstet Gynecol 2015:126(3):e1-e11. Clinical Follow-up?

• No scheduled follow-up is required • Clinical evaluation for women who have used EC IF ✓Menses are delayed by a week or more AFTER the expected time OR ✓If lower abdominal pain or persistent bleeding develops (spontaneous pregnancy loss or ectopic) Best Practice Recommendations

• Clinicians should consider a tiered approach to contraceptive counseling, whereby the most effective and appropriate options are presented before less effective options. • Requiring prerequisite preventive services, such as cervical cytology; breast examination; or evaluation for sexually transmitted infections, diabetes mellitus, dyslipidemia, liver disease, or thrombophilia, can introduce unnecessary barriers to contraceptive care. • If a patient's pregnancy status is uncertain, clinicians may consider same-day start of a nonintrauterine method to provide immediate coverage, and should order follow-up pregnancy testing two to four weeks later. • Family planning services should be offered to adolescents with assurances of confidentiality, in the context of relevant law. • Intrauterine devices and contraceptive implants are safe and effective for postmenarchal adolescents. Infertility 8. Infertility is defined as an inability to conceive in what time period of attempted conception?

A. 6 months B. 12 months C. 18 months D. 24 months Infertility Am Soc Reprod Med Practice Comm 2000 • Defined: 1 year of attempted conception without successful pregnancy • 85% of fertile couples would have been successful by this time. • Earlier evaluation (6 months) − Oligomenorrhea/amenorrhea − Age > 35 years − Known or suspected pelvic pathology Etiology

Factors Percentage Combined factors 40 Male factors 26-30 Ovulatory dysfunction 21-25 Tubal factors 14-20 Other (e.g., cervical factors, peritoneal factors, 10-13 uterine abnormalities) Unexplained 25-28 Lindsay TJ and Vitrikas KR. Am Fam Physician. 2015;91(5):308-214. Essential History

• Sexual − Frequency of intercourse − Use of lubricants, etc. − Erectile dysfunction − Dyspareunia • Drug or alcohol use − Particularly has a “toxic” effect on sperm • Caffeine − Interferes with muscle contraction of fallopian tube • Medications including nonprescription • Chronic disease 9. A couple with 12 months of infertility presents to the office for evaluation. A semen analysis is completed and found to be normal. What is the first step in evaluating the female?

A. Progesterone level B. TSH level C. FSH Level D. Estradiol level Primary Objectives at Workup

Rule Out Procedure Azoospermia Semen analysis Anovulation Urinary LH and mid-luteal phase progesterone levels (day 21 of cycle for latter) Tubal obstruction Hysterosalpingogram (HSG) or laparoscopy

Uterine cavity anomalies HSG or sonohysterogram Decreased ovarian reserve Serum FSH on day 3 of cycle ↑ FSH (> 15-29 IU/L) are associated with: • Poor ovarian response to exogenous gonadotropins • Reduced likelihood of successful conception Male Evaluation If normal, pursue other etiologies Semen Analysis If abnormal, refer to male fertility specialist WHO 2010 Semen Analysis Reference Guidelines Characteristic Normal reference Morphologically normal 4% Motility (progressive) 32% Motility (total) 40% Sperm count 39 million per ejaculate; 15 million per mL Vitality 58% Volume At least 1.5 mL

Cooper TG, Noonea E, von Eckardstein S, et. al. World Health Organization reference values for human semen characteristics. Hum Reprod Update. 2010;16(3):231-245. Male Factor

History: Paternity, surgery, alcohol use, smoking, marijuana, medications

Physical Tests Treatment Testicular volume, hernia, Sperm analysis; (+/-) Intrauterine insemination, prostate, penile discharge Testosterone and FSH IVF, donor Male Evaluation

• Smoking causes damage to sperm; decreases sperm count; is associated with impotence. • Marijuana – decreases seminal fluid, lowers total sperm; contributes to abnormal sperm “behavior.” • If oligospermia or azoospermia is noted, hypogonadism should be suspected. Obtaining morning levels of total testosterone can help differentiate between primary and secondary disorders. • A decreased testosterone level with an increased FSH level points to primary hypogonadism. • A low testosterone level with a low FSH level signals a secondary cause. Some causes, such as hyperprolactinemia, are reversible with proper treatment. Varicocele Surgery or Embolization • Meshwork of distended blood vessels in the scrotum • Result of dilatation of spermatic vein • No evidence that treat- ment improves couples’ chance of conception when compared to expectant management

Source: OpenI/NIH Female Evaluation

Progesterone level TSH, Prolactin, FSH, < 5 ng/mL; evaluate Estradiol level for cause Ovulation Evaluation (day 21 progesterone level) Assess for tubal Progesterone level patency/uterine > 5ng/mL, indicates abnormalities ovulation (hysterosalpingography vs. laparoscopy) Female Evaluation

Progesterone level TSH, Prolactin, FSH, < 5 ng/mL; evaluate Estradiol level for cause Ovulation Evaluation (day 21 progesterone level) Assess for tubal Progesterone level patency/uterine > 5ng/mL, indicates abnormalities ovulation (hysterosalpingography vs. laparoscopy) If Not Ovulating …

• TSH, Prolactin • Check FSH – to ensure that patient is not menopausal. − Premature ovarian failure – consider donor oocytes • Look for: − Systemic disease − Anorexia nervosa, low body fat − PCOS/chronic hyperandrogenic anovulation − Hypothalamic dysfunction − Stress WHO Ovulatory Disorders

Group Disorders Percent Comments I Hypothalamic pituitary 10 Present with amenorrhea and low failure gonadotropin levels, most commonly from low body weight or excessive exercise II Dysfunction of 85 Include those with PCOS, hypothalamic-pituitary- hyperprolactinemia ovarian axis III Ovarian failure 5 Conceive ONLY with oocyte donation and in vitro fertilization National Collaborating Centre for Women’s and Children’s Health. Fertility: assessment and treatment for people with fertility problems. London, UK: National Institute for Health and Clinical Excellence (NICE); February 2013:1-63. (Clinical guideline no. 156). Abnormal TSH, Prolactin etc.

Consider ovulation Assess need for Treat underlying induction for Assisted causes WHO Group II Reproductive Disorders* with Technology Clomiphene§

*WHO Group II – Overweight and have PCOS; can benefit from weight loss, exercise, and lifestyle modifications to restore ovulatory cycles and achieve pregnancy. §Clomiphene has proven effective for ovulation induction in women with PCOS. The addition of 1500-1700 mg of metformin daily MAY increase ovulation and pregnancy rates, but it DOES NOT significantly improve live birth rates over clomiphene alone. Female Evaluation

Progesterone level TSH, Prolactin, FSH, < 5 ng/mL; evaluate Estradiol level for cause Ovulation Evaluation (day 21 progesterone level) Assess for tubal Progesterone level patency/uterine > 5ng/mL, indicates abnormalities ovulation (hysterosalpingography vs. laparoscopy) Tubal Patency/Uterine Abnormalities

• Surgical correction of tubal obstruction or uterine adhesions • Assess need for Assisted Reproductive Technology Other Female Factors

Factor Recommendation Tubal Tubal disease diagnosed by HSG, confirm with laparoscopy; treatment – typically IVF Cervical Previous cryotherapy, LEEP, conization; in-utero DES exposure Endometrial HSG: fibroids, polyps, anomalies; may need sonohysterogram/hysteroscopy Peritoneal Accounts for majority of infertility in young women. (endometriosis) Surgical ablation preferred over medical treatment IF pregnancy desired Lifestyle Factors

• To improve chances of natural conception or using assisted reproductive technology • Abstain from tobacco use • Limit alcohol consumption • Aim for BMI < 30 • Because anxiety over infertility may cause increased stress and decreased libido, further compounding the problem, formal counseling is encouraged for couples experiencing infertility National Collaborating Centre for Women’s and Children’s Health. Fertility: assessment and treatment for people with fertility problems. London, UK: National Institute for Health and Clinical Excellence (NICE); February 2013:1-63. (Clinical guideline no. 156). SORT: Key Recommendations for Practice Evidence Clinical Recommendation Rating Confirmation of ovulation should be obtained with a serum progesterone level on day 21 of a 28-day cycle or one week before presumed onset of menses C Hysterosalpingography should be offered to screen for uterine and tubal abnormalities in women with infertility who have NO history of pelvic infections, C endometriosis, or ectopic pregnancy Women with UNEXPLAINED infertility should NOT be offered ovulation induction or intrauterine insemination because these have NOT been shown to increase C pregnancy rates Women with a BMI > 30 should be counseled to lose weight because this may restore ovulation. B

Lindsay TJ and Vitrikas KR. Am Fam Physician. 2015;91(5):308-214. Recommendations From the Choosing Wisely Campaign Recommendation Sponsoring Organization Do not perform immunological testing as American Society for part of the routine infertility evaluation Reproductive Medicine Do not routinely order thrombophilia testing American Society for on patients undergoing a routine infertility Reproductive Medicine evaluation

Lindsay TJ and Vitrikas KR. Am Fam Physician. 2015;91(5):308-214. Answers 1. B 2. C 3. A 4. C 5. C 6. B 7. D 8. B 9. A References

1. Braun MM, Overbeek-Wager EA, and Grumbo RJ. Diagnosis and Management of Endometrial Cancer. Am Fam Physician. 2016;93(0):468-474. 2. Practice Bulletin No. 149: Endometrial cancer. Obstet Gynecol. 2015; 125(4):1006-1026. 3. Centers for Disease Control, Human, Papillomavirus. http://www.cdc.gov/hpv/ 4. 2015 Sexually Transmitted Disease Treatment Guidelines. http://www.cdc.gov/std/tg2015/default.htm 5. Human Papillomavirus ACIP Vaccine Recommendations. http://www.cdc.gov/vaccines/hcp/acip-recs/vacc- specific/hpv.html 6. American Society for Colposcopy and Cervical Pathology. http://www.asccp.org/Portals/9/docs/ASCCP%20Management%20Guidelines_August%202014.pdf 7. Doubeni CA, Doubeni A, Myers AE. Diagnosis and Management of Ovarian Cancer. Am Fam Phys. 2016;93(11):937-944. 8. Buys SS, Partridge E, Black A, et al. Effect of screening on ovarian cancer mortality: the Prostate, Lung, Colorectal and Ovarian (PLCO) Cancer Screening Randomized Controlled Trial. JAMA. 2011;305(22):2295-2303. 9. ACOG Practice Bulletin 152. Emergency Contraception. Obstet Gynecol 2015;126:e1-11. 10. Centers for Disease Control and Prevention. U.S. Medical eligibility criteria for contraceptive use, 2010. MMWR Recomm Rep. 2010;59(RR-4):1-86. 11. Klein DA, Arnold JJ and Reese ES. Provision of Contraception: Key Recommendations from the CDC. Am Fam Physician. 2015;91(9):625-633. © 2020 American Academy of Family Physicians. All rights reserved.

All materials/content herein are protected by copyright and are for the sole, personal use of the user.

No part of the materials/content may be copied, duplicated, distributed or retransmitted

in any form or medium without the prior permission of the applicable copyright owner. THANK YOU